MU MCQs
MU MCQs
MU MCQs
Preface ....................................................................................................................................................... 4
Final MCQ 6th year 2019 ............................................................................................................................. 5
Dr. Rami questions : ............................................................................................................................................ 5
Dr. Basil questions: ............................................................................................................................................. 5
Dr. Noha questions : ........................................................................................................................................... 6
Dr. Samah questions : ......................................................................................................................................... 6
Dr. Jeries questions : ........................................................................................................................................... 7
Dr. Mdallal questions : ........................................................................................................................................ 8
Dr. AL3ani questions : ......................................................................................................................................... 8
Final Exam 2019 4th year (Ihsan)
By:Abdelrahman Bdeir, Abdulrahman Alwardat & Abdallah Alazzam ........................................................... 8
DR. Rami – GI : .................................................................................................................................................... 9
DR. Samah – RS : ............................................................................................................................................... 11
DR. Ahmad – NEOHRO : .................................................................................................................................... 13
DR. Mohammed – HEMATOLOGY .................................................................................................................... 14
DR. Nuha – ENDO + INFECTUS : ........................................................................................................................ 16
4th year Final exam 2018
By: Ammar Adaileh & Tareq Abu Lebdah ................................................................................................... 17
GIT & Liver :....................................................................................................................................................... 18
Hematology :..................................................................................................................................................... 19
Infectious : ........................................................................................................................................................ 20
Endocrinology : ................................................................................................................................................. 20
CVS : .................................................................................................................................................................. 21
Rheumatology : ................................................................................................................................................. 22
Nephro & electrolytes :..................................................................................................................................... 23
Pulmonary : ....................................................................................................................................................... 24
4th year Final exam 2017............................................................................................................................ 25
4th year Final exam 2016 .......................................................................................................................... 29
6th year Final exam 2016............................................................................................................................ 36
Final Exam 2013 6th year............................................................................................................................ 42
Final Exam 2012 4th year .......................................................................................................................... 54
Final Exam 2011 4th year .......................................................................................................................... 60
Final Exam 2008 X year ............................................................................................................................. 75
Final exam 2007 6th year ........................................................................................................................... 94
Final Exam past years 2004 - 2007 / Q. of unknown origin ........................................................................ 119
Part 1............................................................................................................................................................... 119
Part 2............................................................................................................................................................... 125
Part3................................................................................................................................................................ 128
Part 4............................................................................................................................................................... 152
Part 5............................................................................................................................................................... 156
Part 6............................................................................................................................................................... 160
Part7 .............................................................................................................................................................. 165
Part 8 ............................................................................................................................................................ 171
Part 9............................................................................................................................................................... 181
Cardiology .............................................................................................................................................. 185
Part 1 (top secret) ........................................................................................................................................... 185
part 2............................................................................................................................................................... 196
Gastroenterology .................................................................................................................................... 266
Haematology .......................................................................................................................................... 274
Rheumatology and bone disease ............................................................................................................. 279
Nephrology ............................................................................................................................................. 287
Respiratory medicine .............................................................................................................................. 420
RS MCQs Dr. Samah 2019 ............................................................................................................................... 420
Preface
The file contains all the questions for the Internal Medicine
department at Mu'tah University from the year 2004 to the
current year.
DR. Samah – RS :
1-Not side effect of B2 agonist :
A. Tachycardia
B. Arrythmias
C. Tremor
D. Hyperkalemia
E. hypokalemia
2-most common clinic presentation of PE ?
A. acute onset dyspnea
B. chest pain
C. cough
D. syncope
E. hemoptysis
3-67 years old male patient complain from chest pain, productive cough and fever,
on examination the patient confused and RR=33, BP=100, on laboratory
investigation blood urea = 9 mmol/L (not mg/dl as in slides so to convert mmol/l to
mg/dl = mmol/l X 18), calculate CURP-65 score?
A. 2
B. 3
C. 4
D. 5
E. 6
4-A medical student while taking the internal medicine exam suffered from
tachypnea and anxiety, in the emergency laboratory investigation Ph=7.52, co2=22
, HCO3=24, which of the fooling is correct ?
A. Acute Metabolic alkalosis
B. Chronic Respiratory alkalosis
C. chronic Metabolic alkalosis
D. Acute Respiratory alkalosis
11-Wrong regarding to TB ?
answer: microscope used to differentiate between resistance and sensitive bacteria
for anti-TB
explanation: Cultures on L J media used as Sensitivity tests for anti-TB drugs
12-interstitial lung fibrosis ?
Answer: CT ????
13-Not part of hyperventilation syndrome:
Answer: PE
DR. Ahmad – NEOHRO :
1- Patient come with loin pain & fever and costovertebral angle tenderness what’s
your diagnosis :
A. Pyelonephritis
B. kidney stones
C. Nephrotic syndrome
D. Nephritic syndrome
2-Drugs that Cause hyperkalemia except :
A. Salbutamol
B. Insulin
C. ACEI
D. ARBs
E. SPIRONOLACTONE
3 - Pt with ventricular tachycardia and after few days serum creatinine increased
what type of kidney injury :
a. pre-renal
b. renal
c. post renal
5- 35 year old male complaining from fatigue. He denied hx of melena, trauma ...
cbc shows decrease in Hb, MCV, .. normal RDW ; next step :
A. Occult stool test
B. Iron measurment
C. Hb electrophoresis
D. Lead level
7- Pt have serum ca 14.3 and serum creatinine is 2.3 and on X-Ray have
compression Fracture of vertebrae what is next step :
Answer: Serum immune electrophoresis
8- Drug act on thrombin ?
Answer: Argatroban (direct thrombin inhibitor)
9- 1st choice treatment for ITP ?
Answer: IVIG
3-diabetic patient type 1 had kussmaul breathing , what most likely happen to him
?
Answer: DKA
4-patient present with perioral numbness & carpopedal spasm what most likely
diagnosis ?
Answer: Hypocalcemia
5-diabetic patient type 1 missed a meal what will happen to him ?
Answer: DM type 1 patient take insulin so missed a meal will cause
hypoglycemia
6- patient present with nausea ,vomiting & Bradycardia + continuous fever for 4
weeks what most likely causative organism ?
answer: Salmonella typhi
6- female with ECG showed : narrow QRS complex with regular HR of 170bpm
what is next step ?
Answer: IV adenosine
2- 59 year old patient with upper abdominal and nausea with vomiting , least essential
tool for dx :
CT scan ( Cardiac enzymes , ECG , Amylase , other ?? ).
3- Patient complaining of jaundice and the urine analysis revealing positive bilirubin :
( obstructive jaundice cause ) so it is Dubin–Johnson syndrome
4- Patient diagnosed with crohn's disease complaining of multiple perianal fistulas and
the treatment that given is infliximab which is :
Anti TNF monoclonal antibodies
5- Patient with liver cirrhosis come with tense ascitis and bilateral lower limb edema and
diffuse abdominal pain , ascetic fluid analysis was : 750 wbcs with 90%
polymorophonuclear cells and 3 g/dl albumin , what is treatment and negative gram stain
? Ceftriaxone IV (spontenous bacterial peritonitis
7- Patient with liver cirrhosis complaining of esophageal varicies and the banding
therapy is given , what drug we should put patient on : Propranolol
9- 20 year old female diagnosed as Irritable bowel syndrome with pain predominance ,
what is treatment not used for her :
Narcotic analgesia ( the other choices : tricyclic antidepressant , high fiber diet ,
antispasmotic )
10- At liver cirrhosis the level of which substance with low level :
urea ( impairment of urea cycle which occur at liver )
12- Drug of choice for patient with ulcerative colitis that reach segmoid colon :
Oral 5-ASA ( suppository if it is limited to rectum )
3- Father with his two sons come to ER at cold weather . They are complained of
dizziness and headache and confusion what is most likely diagnosis :
Mono oxide toxicity
4- yound adult complaint of sudden headache with neck rigidity and without fever , most
likely diagnosis : Subarachnoid hemorrhage (approved by dr, waleed )
Hematology :
1- wrong regarding CLL : It is disease of adults
4- True regarding Sickle cell disease : Hand-foot syndrome is associated with swelling
and pain at fingers and toes for children and cause disability ???!!!!
.
Infectious :
3- Patient with history of cough and sputum with fever and SOB , xray reveals non
homogeneous opacity at middle lung , after few days complained of Spontaneous
pneumothorax what is most likely organism :
This is scenario of narcotizing pneumonia >> Staph aurues
8- Egyptian farmer with scenario of mass at junction between bladder and ureter , true
regarding this patient :
He has schistosomiasis ??!!!!
9- true regarding typhoid fever : rose spot comes with fever ???!!
Endocrinology :
6- Fasting blood sugar : 120 , postprandial blood sugar : 180 , what is true : impaired
FBS & postprandial blood sugar tolerance
10- Wrong regarding SIADH : there is signs of overload in that patient ??! actually there
is no significant signs ??!!!
CVS :
1- young complained of repetitive attack of syncope in the morning after he shaves his
beard : ( He pressed on carotid body during shaving ) So it is Carotid body syncope
2- Patient come to clinic due to attack of syncope and doctor approved that his new drug
is the real cause of these attack of syncope , which less likely drug :
Aminoglycoside ( all of them are HTN drugs + procainamide )
5- Young female patient with palpitation and regular pulse with 120 Beat per minute ,
Treatment : Adenosine IV ??!!
8- patient with early diastolic murmur at left sternal area with high volume pulse (water
hammer pusle) : Aortic regurgitation
9- One is presentation of hypertrophic cardiomyopathy : sudden death
11- Shifted apex beat to anterior axillary line to 6 or 7th intercostal space indicative of :
Left ventricular dilatation
Rheumatology :
5- Antistreptolysin Q ??!!!!
7- 60 year old patient with headache and tenderness over temporal region and she suffers
from tiredness during mastication and talking , what is the diagnostic test
10- signs and symptoms of glomerulnephritis after upper respiratory tract infection, on
histopathology of biopsy what is the finding ( )ذكروني بالكيس: IgA nephritis
11- case with hemoptysis and hematuria with proteinase 3 positive (c-ANCA positive ) :
Wegner granulomatosis
4- Hyperkalemia chages on ecg : PR prolongation and QRS widening which one ???!
10- cause of papillary necrosis due to tubulointerstitial disease : elderly using chronic
analgesia
7- True regarding ARDS : شايفهن كلهن غلط (الويدج برشر دايركتلي ريليتد للبولموناري برشر فأي جواب
وال مش صحيح ؟، بأكد الثاني2)من ال
5-Inheritance bleeding ??
Vwf
6-Systolic murmur ??
Between s1 and s2
11-gottorn rash>
dermatomyositis
18-DM , Correct :
Fasting blood glucose < 126
35-a patient come with pain in left upper quadrent for with a history of infectious
mononucliosis what is the diagnosis ?
Splenic rapture
(hyposplenism )
58-.ECG in hypercalemia
60-.SS In ...
SLE +sjougren "not sure"
66--.commonest ca in thyroid..
papillary
71 HLAB27 ...
SPA
72- SE Of statine
( HmgCoa reductase)??
73-. DM ..
insulin not use firstly for type 2DM
13) History of a man with retrosternal chest pain. ECG reveals ST segment
depression in I and aVL, diagnosis:
a. Posterior MI
b. Inferior MI
c. Lateral MI
21) Young male, 20 years, fever, weight loss 10 kg, x-ray show anterior
mediastinal mass , diagnosis is:
a. Lymphoma
b. Coccidiomycoma
c. Histoplasmosis
25) A patient who presented with typical signs and symptoms of PMR, next step
in management:
a. Low dose steroids
b. High dose steroids
a. Surgery
b. Postural drainage and CPT
c. Maintenance of normal BP
d. Prophylactic antibiotics
50) Patient present with Hb of 8… Blood film shows polychromasia. Best next
step in management . patient is on hydoxychloroquine therapy for SLE :
a. IV corticosteroids
b. Plasmapharesis
Ans: A (She has Evan’s syndrome)
51) All are correct about the mortality in ARF, except:
a. Mortality depend on the cause
b. Prognosis is generally good with reversible causes like drugs and blood loss
c. Mortality in complicated ARF reaches 15-30%
52) History: A female with history of long travel, then develops unilateral lower
limb swelling with redness and hotness. She was diagnosed with DVT. She was
started on Unfractionated heparin. 10 days later, she was found to have a platelet
count of 60,000. Next step in management:
a. Stop unfractionated heparin and no longer anticoagulation
b. Stop unfractionated heparin and start low-molecular heparin.
c. Stop unfractionated heparin and start her on leperudine
53) Alcoholic patient was found to have macrocytic anemia, the most likely
cause:
a. Vitmain B12 defeciency
54) 30 year old male patient, presented with chest pain one week after an URTI,
most likely diagnosis:
a. Pericarditis
b. Tension pneumothorax
55) 3 weeks after MI, a patient presented with chest pain. ECG showed elevated
ST segment in anterior chest leads, diagnosis is:
a. Re-infarction
b. Pericarditis
c. Ventricular aneurysm with superimposed VT
Answer: C (Cause of persistent ST segment elevation is ventricular aneurysm)
56) A 22 year old male patient, with central chest pain, radiating to back, he is
found to have murmur on exam. Also, he is 2m tall. Most likely diagnosis:
a. MI
b. Pericarditis
c. Aortic dissection
Answer: C (tall ➔ Marfan syndrome. Plus, he is too young for an MI. Also, chest pain
that radiates to back is characteristic of aortic dissection)
58) 45 year-old male patient presents with retrosternal chest pain, radiating to
left arm. Troponin is negative. CK is 320. Next step in management:
a. Reassure and send home
b. Start heparin
c. Start thrombolytics
d. Observation
e. Ask for the CK-MB fraction.
59) Patient with history typical of MI. Found to have ST segment depression in
leads aVF, V2, and V3. Pulmonary capillary wedge pressure is normal. Right
ventricle pressure is 65/20. Right atrial pressure is 25/10. Neck veins are distended.
Diagnosis is:
a. Mitral valve prolapse
b. Right ventricular infarction
c. ?
Horner syndrome can be caused by any interruption in a set of nerve fibers that start in the part of the brain called
the hypothalamus and travel to the face and eyes.
Source: http://www.nlm.nih.gov/medlineplus/ency/article/000708.htm
25)A disease modifying anti rheumatic drugs (DMARD) include all f the following
except for :
A)Salazo pyrine.
B)Hydroxychloro quine.
C)Colchicine.
D)Methotrexate
E)leflenamide
26)All of the following are criteria for Behcet disease except for :
A)Mouth ulcer's.
B)Arterial Anuyresm .ِ
C)Hypopyron..
D)Pethergy test.
E)Acne-like lesion
29)All of the following are indications for the treatment of Gouty arthritis except
for:
A)Chronic Gouty arthritis.
B)Renal stones.
C)Renal failure.
D)Serum uric acid more than 8mg in men.
E)All of the above.
Ans:
- About 55% of UGIB is due to peptic ulcer disease.
Abstract
The relation between peptic ulcer and stomach cancer has long been disputed, but there is accumulating
evidence that gastric ulcer disease is positively associated and duodenal ulcerations negatively associated with the risk of
developing stomach cancer.
World J Surg. 2000 Mar;24(3):315-20. Risk of stomach cancer in patients with peptic ulcer disease. Hansson LE.
Department of Surgery, Mora Hospital, S-792 85 Mora, Sweden.
5. Case history: Patient presents to ER with K of 7.9 mmol/L and… the best
initial management:
A. Calcium gluconate
B. Calcium resonium
C.
6. One of the following does not metastasize to the CNS:
A. ALL
8. Worst prognosis in a patient diagnosed with ALL is with the following at the
presentation:
A. Neurological involvement
B. Philadelphia chromosome
C. Male
D. Age
Answer: Increasing age\ Philadelphia chromosome\ WBC >30,000
9. Wrong about iron defeciency anemia:
A. Low TIBC
B. Low retics response
Answer: (A)➔ High TIBC
Corrected reticulocyte count = %reticulocyte X (Patient's Hct/Expected normal Hct of 40)
Less than 2% = hypoproliferative type. This means that her anemia is due to underproduction of red cells by the bone marrow.
11. A plumber who didn’t improve after taking ranitidine for multiple ulcers he
had. He then developed steatorrohea:
A. Zollinger Ellison syndrome
Diarrhea — Diarrhea is usually characterized as frequent loose stools of small to moderate volume. Stools generally occur
during waking hours, most often in the morning or after meals. Most bowel movements are preceded by lower abdominal
cramps and urgency even to the point of fecal incontinence and may be followed by a feeling of incomplete evacuation.
Approximately one-half of all patients with IBS complain of mucus discharge with stools [16]. Large volume diarrhea,
bloody stools, nocturnal diarrhea, and greasy stools are NOT associated with IBS and suggest an organic disease. A
subgroup of patients describe an acute viral or bacterial gastroenteritis which then leads to a subsequent disorder
characteristic of diarrhea-predominant IBS, called post-infectious IBS. (See "Pathophysiology of irritable bowel
syndrome".)
14. 17 year old african american girl presented to ER with a one week history of
painful lesions on the legs. X-ray showed bilateral hilar masses, diagnosis is:
A. Sarcoidosis
16. A woman who developed dyspnoea over 3 weeks… she presented to ER… X-
ray showed large left pleural effusion, your next step is:
A. Aspiration of fluid to dryness and examination of fluid
B. Only examination of fluid
Answer: A
NEW TERMINOLOGY — In January 2011, the Boards of Directors of the American College of Rheumatology, the
American Society of Nephrology, and the European League Against Rheumatism recommended that the name Wegener’s
granulomatosis be changed to granulomatosis with polyangiitis (Wegener’s), abbreviated as GPA [1-3]. This change
reflects a plan to gradually shift from honorific eponyms to a disease-descriptive or etiology-based nomenclature. The
parenthetic reference to Wegener’s will be phased out after several years as the new name becomes more widely known.
18. X-ray showed a mass in the lung, you suspect this patient to have non-small
cell lung cancer. What finding would be against this diagnosis:
A. High ADH
Small cell carcinoma is associated with Eaton-Lambert syndrome (spares ocular muscles), SIADH, and other
paraneoplastic syndromes.
a generalized disorder of neuromuscular transmission caused by a defect in the release of acetylcholine quanta from the
presynaptic nerve terminals; often associated with small cell carcinoma of the lung, particularly in elderly men with a
long history of cigarette smoking. In contrast to myasthenia gravis, weakness tends to affect solely axial muscles, girdle
muscles, and less often the limb muscles; autonomic disturbances, e.g., dry mouth and impotence, are common; the
deep tendon reflexes are unelicitable; on motor conduction studies, responses on initial stimulation are quite low in
amplitude, but they show marked post-tetanic facilitation after a few seconds of exercise. Lambert-Eaton syndrome is
due to loss of voltage-sensitive calcium channels located on the presynaptic motor nerve terminal. See: myasthenic
syndrome. Syn: carcinomatous myopathy, Eaton-Lambert syndrome, Lambert syndrome, myasthenic syndrome.
Despite the list of characteristics that typically accompany PCOS, the exact mechanism(s) responsible for hypertension in women with
PCOS is controversial. Many of the symptoms associated with PCOS have been shown to also be associated with increases in blood
pressure, such as increases in body mass index and the presence of metabolic syndrome, with its accompanying insulin resistance and
type 2 diabetes.
http://hyper.ahajournals.org/content/49/6/1220.full
Numerous uncontrolled trials and one controlled trial of epoprostenol therapy have demonstrated improved survival
compared to historical controls [61,71-73]. As an example, one trial demonstrated that survival among patients
receiving epoprostenol was better than historical controls at one year (85 versus 58 percent), three years (63 versus 33
percent), and five years (55 versus 28 percent) [61].
A meta-analysis of 21 randomized trials (3140 patients) found that therapy with a prostanoid, an endothelin receptor
antagonist, or a phosphodiesterase-5 inhibitor improves mortality compared to controls (1.5 versus 3.8 percent, RR
0.57, 95% CI 0.35 - 0.92) [69]. The average duration of the trials was 14 weeks.
Answer: ????
23. A diabetic patient was diagnosed with new hypertension, best management:
A. Thiazide
B. Enalapril
C. Furosmide
Answer: B.
Summarized briefly, no benefit from statin therapy has generally been demonstrated in patients with moderate to severe
heart failure due to systolic dysfunction with or without coronary artery disease. Limited data suggest that statins may
benefit patients with diastolic dysfunction.
SUMMARY AND RECOMMENDATIONS — Despite strong evidence of benefit for statins in most subsets of patients
with established cardiovascular disease, two large randomized trials found no benefit from initiating statin therapy in
patients with symptomatic systolic heart failure (ischemic or nonischemic) and a mean left ventricular ejection fraction
≤33 percent. (See 'Statins in systolic HF' above.)
Pulmonary involvement — Pulmonary involvement is seen in more than 70 percent of patients with SSc. The two
principal clinical manifestations are interstitial lung disease (also called fibrosing alveolitis or pulmonary fibrosis) and
pulmonary vascular disease, leading to pulmonary arterial hypertension (table 5). These issues are discussed in detail
separately but will be briefly reviewed here. (See "Clinical manifestations of systemic sclerosis (scleroderma) lung
disease".)
Neuromuscular involvement — Neuromuscular involvement in SSc is discussed in more detail elsewhere. (See
"Neuromuscular manifestations of systemic sclerosis (scleroderma)".) The following is a brief summary of the types of
neurologic and muscle disorders that have been noted in case reports and series:
30. In kidney biopsy, linear deposits of IgG were found along basement
membrane, diagnosis is:
A. Good pasture’s syndrome
Ans: B
32. A patient with RUQ pain, fever, chills, rigors, clay colored stool and dark
urine. Diagnosis is:
A. Acute hepatitis
B. Ascending cholangitisi
C. Acute cholecystisi
Ans: A.
CLINICAL MANIFESTATIONS — The classic triad of The incubation period averages 30 days (range 15 to 49
Charcot — fever, right upper quadrant pain, and jaundice — days), after which the illness begins with the abrupt onset
occurs in only 50 to 75 percent of patients with acute of prodromal symptoms including, fatigue, malaise,
cholangitis [8]. Confusion and hypotension can occur in nausea, vomiting, anorexia, fever, and right upper
patients with suppurative cholangitis, producing Reynold's quadrant pain. Within a few days to one week, patients
pentad, which is associated with significant morbidity and note dark urine, acholic stool (light-colored stools lacking
mortality [9]. Hypotension may be the only presenting bilirubin pigment), jaundice, and pruritus. The prodromal
symptom in elderly patients or those on corticosteroids,
symptoms usually diminish when jaundice appears. The
while septic shock in severe cases can lead to multiorgan
most common physical findings are jaundice and
failure.
hepatomegaly. (See 'Clinical evaluation' above.)
The average fluid loss is 3 to 6 liters in DKA and up to 8 to 10 liters in HHS, due largely to the glucose osmotic diuresis
(table 2) [1,2,8,10]. In addition to inducing water loss, glucosuria results in the loss of approximately 70 meq of sodium
and potassium for each liter of fluid lost. The aim of therapy is to replete the extracellular fluid volume without inducing
cerebral edema due to too rapid reduction in the plasma osmolality. (See 'Cerebral edema' below and "Treatment and
complications of diabetic ketoacidosis in children", section on 'Cerebral edema'.)
Fluid repletion is usually initiated with isotonic saline (0.9 percent sodium chloride). This solution will replace the fluid
deficit, correct the extracellular volume depletion more rapidly than one-half isotonic saline, lower the plasma osmolality
(since it is still hypoosmotic to the patient), and reduce the serum glucose concentration both by dilution and by
increasing urinary losses as renal perfusion is increased [16,19]
Intravenous regular insulin — After an initial infusion of isotonic saline to increase insulin responsiveness by
lowering the plasma osmolality [17,18], the only indication for delaying insulin therapy is a serum potassium
below 3.3 meq/L, since insulin will worsen the hypokalemia by driving potassium into the cells. (See
'Potassium depletion' below.)
Rheumatic disorders — Patients may have detectable serum RF in a variety of rheumatic disorders, many of which share
similar features, such as symmetric polyarthritis and constitutional symptoms. These include [36]:
Nonrheumatic disorders — Nonrheumatic disorders characterized by chronic antigenic stimulation (especially with
circulating immune complexes or polyclonal B lymphocyte activation) commonly induce RF production (table 1).
Included in this group are [36]:
• Indolent or chronic infection, as with SBE or hepatitis B or C virus infection. As an example, studies have
demonstrated that hepatitis C infection, especially when accompanied by cryoglobulinemia, is associated with a positive
RF in 54 to 76 percent of cases [44-47]. RF production typically ceases with resolution of the infection in these disorders.
These molecules may be produced by activated hepatic lymphocytes [48]. (See "Clinical manifestations and diagnosis of
essential mixed cryoglobulinemia".)
• Inflammatory or fibrosing pulmonary disorders, such as sarcoidosis.
• Malignancy.
• Primary biliary cirrhosis
CLASSIFICATION CRITERIA — There is no specific test or combination of tests that can be used to establish
the diagnosis of ASD. As a result, at least seven sets of diagnostic criteria have been proposed [15-21].
• Sore throat
• Lymphadenopathy
• Hepatomegaly or splenomegaly
• Abnormal liver function studies, particularly elevations in aspartate and alanine aminotransferase and lactate
dehydrogenase concentrations
• Negative tests for antinuclear antibody and rheumatoid factor
• Hypovolemia
• Gastrointestinal bleeding
• Hypokalemia and/or metabolic alkalosis
• Hypoxia
• Sedatives or tranquilizers
• Hypoglycemia
• Infection (including SBP)
• Rarely, hepatoma and/or vascular occlusion (hepatic vein or portal vein thrombosis)
Answer: B.
40. 26 year old female presented to ER with petechiae, everything else is normal:
A. ITP
B. Septic meningitis
Ans: A?
Many of the older studies examining treatment in RPGN with pulse corticosteroids, cyclophosphamide, and
plasmapheresis are difficult to interpret because they were performed at a time before it was possible to distinguish
among the different types of RPGN. Nevertheless, these studies demonstrated that conventional doses of oral
prednisone, given alone or in combination with azathioprine, usually had little beneficial effect [1].
As a result, the therapy of most patients with RPGN involves pulse methylprednisolone followed by daily oral
prednisone, oral or intravenous cyclophosphamide, and, in some settings, plasmapheresis. Early diagnosis with renal
biopsy and serologic testing and early initiation of appropriate therapy is essential to minimize the degree of irreversible
renal injury.
Empiric therapy may be begun with the above modalities in patients with severe disease, particularly if either renal
biopsy or interpretation of the biopsy will be delayed. Empiric initial therapy consists of intravenous pulse
methylprednisolone (500 to 1000 mg/day for three days) and consideration of plasmapheresis, especially if the patient
has hemoptysis. This regimen will not alter the histologic abnormalities observed with a renal biopsy that is performed
soon after initiating empiric therapy.
49. A patient with palpitations and heat intolerance… technocium sacn revealed
uniform increased uptake in the thyroid, diagnosis is:
A. Graves’ disease
We generally agree with the 2005 American College of Cardiology/American Heart Association (ACC/AHA) guidelines
on peripheral artery disease, which were produced in collaboration with major vascular medicine, vascular surgery, and
interventional radiology societies [4].
Medical therapy for control of hypertension is indicated in all patients with bilateral renal artery stenosis (or unilateral
stenosis in a single viable kidney) [4,6]. Revascularization, usually by percutaneous angioplasty with stenting, may be
considered in patients with persistent hypertension who have one or more of the clinical features that suggest that the
stenosis plays an important role in the elevation in blood pressure (table 1). Surgery is primarily warranted for correction
of complex lesions.
The following discussion will review the antihypertensive response to therapy in patients with bilateral renal artery
stenosis. The use of angioplasty or surgery to preserve renal function in such patients is discussed separately. (See
"Chronic kidney disease associated with atherosclerotic renovascular disease".)
Skin necrosis — Skin necrosis is a well-described complication of treatment with unfractionated or LMW heparin. Affected patients
develop heparin-dependent antibodies but most do not experience thrombocytopenia. (See "Heparin-induced thrombocytopenia",
section on 'Skin necrosis'.)
Thrombocytopenia — Heparin-induced thrombocytopenia (HIT) is a well-recognized and potentially fatal complication of heparin
therapy, usually occurring within 5 to 10 days after the start of heparin therapy. The pathogenesis, clinical manifestations, diagnosis,
and treatment of HIT are discussed in detail separately. (See "Heparin-induced thrombocytopenia".)
Ans: A
66. The following drugs and their side effects are correct, except:
A. Thiazide diuretics: thrombocytosis
B. Enalapril: Dry cough
68. Women with multiple tender areas, all investigations are normal:
A. Fibromyalgia
81. All are indications for transfusion therapy in sickle cell anemia, except:
A. Stroke
B. Pain
C. Pain with occulusive ???
D. ???
Transfusion therapy for individuals with SCD can be categorized as therapeutic or prophylactic. Accepted indications for
transfusion therapy in individuals with SCD include [62,63]:
• Therapeutic — Acute use of transfusions for acute stroke, acute chest syndrome, acute multi-organ failure,
acute symptomatic anemia (eg, onset of heart failure, dyspnea, hypotension, marked fatigue [9]), reticulocytopenia (most
commonly associated with Parvovirus B19 infection, but can occur with any infection), or following hepatic or splenic
sequestration.
• Prophylactic — Use of periodic red cell transfusions for primary or secondary stroke prevention.
82. Patient with pancytopenia, splenic vein thrombosis, and ???. Diagnosis is:
A. Promyelocytic???
83. Osteoporosis, most common site of fracture is:
A. Hip
B. Vertebrae
C. Femur
The answer is not FEMUR ===> Neck of femur ===> So, the the most common location is "Hip"... Right? Because "fracture of
the neck of femure" is a type of hip fracture.
"Hip fractures are classified as intracapsular, which includes femoral head and neck fractures, or extracapsular, which includes
trochanteric, intertrochanteric, and subtrochanteric fractures. The location of the fracture and the amount of angulation and
comminution play integral roles in the overall morbidity of the patient, as does the preexisting physical condition of the individual.
Fractures of the proximal femur are extremely rare in young athletes and are usually caused by high-energy motor vehicle
accidents or significant trauma during athletic activity. Other causes may be an underlying disease process such as Gaucher
disease, fibrous dysplasia, or bone cysts."
http://emedicine.medscape.com/article/87043-overview
From the paper entitled “Which fractures are most attributable to osteoporosis?” : "the fractures rated most likely because of
osteoporosis were the femoral neck, pathologic fractures of the vertebrae, and lumbar and thoracic vertebral
fractures."http://www.sciencedirect.com/science/article/pii/S0895435610002635
Acetazolamid (Carbonic anhydrase inhibitos cause normal anion gap metabolic acidosis)
Answer: C (not sure… Because Wikipedia: A less frequent occurrence results from a vomiting
of intestinal contents, including bile acids and HCO3-, which can cause metabolic acidosis.)
12) All of the following are associated with hypokalemia and alkalosis, except:
a. Bartter syndrome (???) [Yes ➔ Hypokaemia + alkalosis➔ a disorder due to a defect in
active chloride reabsorption in the loop of Henle; characterized by primary juxtaglomerular cell
hyperplasia with secondary hyperaldosteronism, hypokalemic alkalosis, hypercalciuria,
elevated renin or angiotensin levels, normal or low blood pressure, and growth retardation;
edema is absent. Autosomal recessive inheritance, caused by mutation in either the Na-K-2Cl
cotransporter gene (SLC12A1) on chromosome 15q or the K(+) channel gene (KCNJ1) on 11q.
b. Furosemide ➔ Yes
c. Diabetes (If they are talking about DKA➔ Hypokalemia and acidosis, so this should be
the answer?
d. Nasogastric tube suction ➔ Yes (loss through upper GI of K and Hydrogen)
e. Thiazides ➔ Yes
16) In a patient with HIV, the worst prognosis is for a patient with:
a. ?
b. ?
c. ?
d. ?
Ans: C (this is the lowest CD4 and highest viral load )
17) HIV is composed of:A single strand of RNA
18) Which of the following is least likely to transmit HIV?
a. Semen
b. Saliva
c. Blood
d. CSF
e. Transplanted organs
Answer: B (saliva_) http://www.cdc.gov/hiv/resources/qa/transmission.htm
19. Which of the following influenza viruses subtypes is expected to cause the (???)
pandemic influenza?
Ans: ????
Answer :b
21. Which of the following is true regarding brucella:
a) monotherapy is the standard of care
b) Usually is treated for 3 weeks. ➔ For 6 weeks
c) Intracellular activity is important in the antibiotic (choice) for brucellosis
d) It is a Gram +ve coci ➔ G-ve bacilli
e) Penumonia is a frequent complication ➔ Pneumoniitis is a less common complication
Answer : c
22. Which of the following is the closest measurement of core body temperature:
a) Oral temperature
b) Axillary temperature
c) Rectal temperature
d) Ear temperature
e) Mixed venous temperature
Ans: E (mixed venous termpature)
23. Which of the following is false about acute HIV disease?
a. It occur within 2-6 weeks of infection
b. It is best diagnosed by ELISA ➔ Wrong (However, the standard third generation
enzyme linked immunosorbent assays (ELISAs) used in clinical practice and in blood banks in
the United States do not detect antibodies to HIV until three to seven weeks after infection.)
c. It manifess aa flue-like illness➔ Yes
d. It is associated with high infectoiu state → Patients with primary HIV infection are
highly contagious to others,
e. It occurs in about 70% of patients
In 1980 the American College of Rheumatology agreed upon diagnostic criteria for
scleroderma.[20]
32. All of the following statement about gout are true, except:
a. In adult men the solubility of monosodium urate is 7 mg/dL
b. Women of child-bearing age have lower serum uric acid
c. Initial treatment of acuteattack should include NSAIDs, colchicine, and allopurinol
d. Diuretics should elevate serum uric acid
e. Attacks can be precipitated by acute MI.
Answer: C (allopurinol and colchicine never in acute treatment)
33. One of the following is correct about aldosterone?
a. Increased Na and K reabsorption in the renal collecting duct
b. Causes increase synthesis of Na/K pumps in the principal cells of the renal collecting
duct
c. Is released from the adrenal cortex in response to decreased plasma K
d. Promotes H secretion from the principle cells of the normal collecting duct
e. ?
Answer : B
34. Acute interstitial nephritis might present with any of the following, except:
a. Fever
b. Rash
c. Renal impairment
d. Hypertension
e. Low grade proteinuria
Answer: D (hypertension )
35. All of the following may cause renal papillary necrosis, except:
a. DM b. Analgesic abuse c. Sickle cell anemia d. pyelonephirits e.
hypertension
Answer E: Any condition that involves ischemia can lead to renal papillary necrosis. The four
most significant causes are sickle cell disease or trait, analgesic use, diabetes mellitus, and
severe pyelonephritis.[2]
A mnemonic for the causes of renal papillary necrosis is POSTCARDS: pyelonephritis,
obstruction of the urogenital tract, sickle cell disease, tuberculosis, chronic liver disease,
analgesia/alcohol abuse, renal transplant rejection, diabetes mellitus, and systemic vasculitis.
Often, a patient with renal papillary necrosis will have numerous conditions acting
synergistically to bring about the disease. [3][4]
36. A 40-year-old female presented to clinic because his blood pressure was 160/100.
All the following are appropriate first line investigations, except:
a. K+ level
b. Urine analysis
c. Renal US
d. MRI suprarenals
e. Creatinien level
Answer: D
37. A 20-year-old male presented to you with generalized weakness. Labs showed:
a. DIarrhea
b. Spironolactone
c. Recovery from DKA
d. Thiazide diurectic
e. Amiloride
Ans: D (Thiazide diuretic cause metabolic alkalosis)
38. All of the following are true about pre-renal acute renal failure, except:
a. FeNa<1%
b. BUN/CR ratio is elevated
c. Mostly irreversilbe
d. Most common cause of ARF
e. Urine output imporoves with IV fluid coverage
Answer: C
39. All of the following is true about diabetic nephropathy in IDDM< except:
a. Microalbumnuria is seen within 5 years from onset
b. Usually preceded by retionopathy
c. Thickening of basement membrane is a ??? factor determining progressing of disease
d. It requires kidney transplant
e. More common in diabetic who have siblings with diabetic nephropathy
Answering: D.
40. All the following are true about effect of drugs in causing ARF, except:
a. Furosemise causes toxicity by forming crystals?
b. NSAIDs cause vasoconstriction
c. Aminogrlyicoside cause toxicity to proimal tubules
d. D-penicillamine causes membranous nephropahty
e. Ampicillin cuases acute tubulointerstitial nephritis
Ansewr: A
45. All of the following are initial management strategies in aptients with upper GI
bleeding, except:
a. Somatostatitn
b. Bleeding scan
c. Esophagogastroduodensoscoyp
d. Acid suppressing medicaiton
e. Gastric lavage
Answer: b
46. Wilson’s disease should be considered in all of the following medical scenarios,
except:
a. Abnormal liver enzymes and non-immune hemolytic anemia
b. Exaggerated high bilirubin level and depressed alkaline phosphtaea
c. Decreases serum ceruloplasmis
d. Elderly patient with neuropsychiatric problem
e. Fuliminat liver failure with low uric acid
Answer: D
47. All of the following medications are being used for non-alcohol steatohepatitis
(NASH), except:
a. Betaite
b. Ursodeoxycholic acid
c. ribaverin
d. Vitamin e
e. Beta carotene
Anwer: C (akeed)
48. All of the following medciations are being used in chronic hepatitis B, except:
a. Lamividine
b. Ribavirine
c. Pregyled interferon
d. Adefovel dig
e. Entovavir
Answer: B (akeed)
49. All of the following micro-organisms can cause infectious diarrhea with positive
fecal leucocytes, except:
a. Shigella
b. Yersinia
c. Giardia
d. Campylobacet
e. Salmonella
Answer: c
50. All of the following are protective from colo-rectal cancers, except:
a. Aspirn
b. Folic acd
c. Fier diet
d. calcium
e. moderate use of alcohol
Answer: E.
53. Regarding Crohn’s disease, all of the following are true, except:
a. The rectum is often spared
b. Fistual fissures and absesscess can occur in patietns with colo-rectal Crohns
c. The disease is limited to the mucosa
d. The mucosa can appear as cobble stone
Answer: C.
54. All of the following are true regarding ulcerative colitis, except:
a. Azathioripne can be sued in treatment
b. Maybe associated with Pyoderma gangreonusum
c. Patients may be P-ANCA positive
d. The rectum is never involved
Answer: D.
57. All of the following are true regarding inflammatory acute diarrhea, except:
a. Can be caused by shigella ➔ Correct
b. There is polymorphonuclear cells in the stool sample
c. There may be blood in the stool sample
d. Usually affects the small bowel in the infected type inflammatory acute diarrhea
Answer: D
58. 5 year-old girl came to ER because of fatigue and shortness of breath. She was
taking amoxacililn for acute otitis medica. Laboratory testing showed Hb 5.5 gm/dL with
normal WBC and platelet count. The smear showed numerous nucleated RBCs and
spherocytes. Both direct and indirect test Coomb’s test were positive. The patient has:
a. Warm autoimmune hemolytic anemia
60. All of the following help differentiate between inflammatory and non-
inflammatory arthritis, except:
a. Favorable response to NSAIDS
b. Mornign stiffness for 2 hours
c. Pain improves with continued ambulation
d. Presence of extra-articular features
e. Hottness and rednes of the affected joint
Answer: E.
Answer: A.
62. All of the following are consistent with the diagnosis of rheumoatoid arthritis,
ecept:
a. Symmetrical involvemtn of the small joints of the hands?
b. Elevated ESR
c. Wan neck deformities in the fingers
d. Nodes of the distal interphalangeal Heberden’s joint
e. Erosive changes othe MCP joints on x-ray
Answer: D (osteoarthritis)
63. A positive rheumatoid factor can be seen in all of the following conditions, ecept:
a. RA
b. TB
c. Hepatitc C
d. Malignancies
e. Combined immune deficiency syndrome → Correct
Answer: B (TB)
64. All of the following are extra-articular features of rheumatoid arthritis, except:
a. Posterior uveitis
b. Cutaneous vasculitis
c. Pulmonary fibrosis
d. Sicca syndrome (secondary sjogren’s)
e. Pericarditis
Answer:B
66. In conductive system of the heart muscle, all of the following are ture, except:
a. Conduction started in SA node. AV node, bundle of His, left and right bundle branch-
purkinjee fibers.
b. Left bundle branch is shorter than right bundle
c. Right bundle supplies right ventricle and left bundle supplied left ventricle and spetum
d. Action potential in the ventricle is rapid and generated by rapid transmembrane K
diffusion.
Answer:D (sodium not K)
68. The least common ause of AF is? Or: The commonest cause of AF?
a. WPW syndrome
b. Mitral valve disease
c. Hypertension
d. Pericarditis
e. Thyrotoxicosis
73. Clinical features in infective endocarditis include all of the following, except:
a. Appearnace of new murmur or change in the quantiy of eisting murmur
b. Fever
c. CHF
d. Skin and eye lesions
e. No splenomegaly
Answer: E (A wide variety of diseases are associated with splenomegaly, or enlargement of the
... Such as in subacute bacterial endocarditis or infectious mononucleosis ...
http://emedicine.medscape.com/article/206208-overview)
A. Chest pain increase by deep breating\ b. usually follow URI’\ C. on EKG, ST segment
elevation is conve upwards\ D. pericardial rub can confirm diagnosis\ E. treated with NSIAD or
aspirin
Answer: C (saddle shaped ST segment – convex downwards not upwards)
76. All of the following are causes of secondary hypertension, except:
a. ?
77. Inspiratory arm of the flow-volume loop wil be typically abnormal in which of the
following conditions?
A. Bronchial asthma
B. Vocal cord dysfunction
C. Emphysema
D. Interstitial lung diseae
E. Bronchoiolitis obliterans
Answer: B.
Vocal cord dysfunction involves inappropriate vocal cord motion that produces partial airway
obstruction. Patients may present with respiratory distress that is often mistakenly diagnosed as
asthma. Exercise, psychological conditions, airborne irritants, rhinosinusitis, gastroesophageal reflux
disease, or use of certain medications may trigger vocal cord dysfunction. The differential diagnosis
includes asthma, angioedema, vocal cord tumors, and vocal cord paralysis. Pulmonary function testing
with a flow-volume loop and flexible laryngoscopy are valuable diagnostic tests for confirming vocal
cord dysfunction. Treatment of acute episodes includes reassurance, breathing instruction, and use of a
helium and oxygen mixture (heliox). Long-term management strategies include treatment for symptom
triggers and speech therapy.
http://www.aafp.org/afp/2010/0115/p156.html
78. All of the following associations between conditions and mechanisms of hypoxia
are true, except:
a. COPD and V/Q mismatch (The principal contributor to hypoxemia in COPD patients is
ventilation/perfusion (V/Q) mismatch resulting from progressive airflow limitation)
b. ARDS and pulmonary shunt (edema in patients with ALI/ARDS is impaired gas
exchange with intrapulmonary shunt,)
c. Multiple rib fractures and hypoventilation
d. Hepatopulmonary syndrome and V/Q mismatch (The hepatopulmonary syndrome is
characterized by a defect in arterial oxygenation induced by pulmonary vascular dilatation in
the setting of liver disease1) (Dyspnea and hypoxemia are worse in the upright position (which
is called platypnea and orthodeoxia, respectively)
e. Motor neuron disease and hypoventilation
Answer: C
79. IN patients with sarcoidosis, all of the following are associated with good
prognosis, except:
a. Fever
b. Erythema nodosum
c. Age less than 40 years
d. Black race
e. Presence of polyarthritis
Answer: D.
80. Regarding the pathogenesis of bronchial asthma, one of the following is specific for
the disease:
a. Air flow limitation
b. Airway hyper-responsiveness
c. Inflammation of the mucosa
d. Peak flow variability
e. Brochioalevolar eosinophils
Answer: B.
81. The most common organism responsible for severe community pneumonia needing
ICU care is:
a. Strep. pnuemonia
b. Legionella
c. H. influenza
d. Gram negative bacilli
e. Mycoplasama pneumonia
Answer: e
83. Actions of PTH include all of the following, except:
a. Increase bone resoption
b. Increase net acid excretion
c. Increased calcium absorption from GI tract
d. Increased calcium reabsorption in the kidney
e. Increased phosphate excretion
Answer: B
effect of serum osmolarity of a patient with the following labs: Na+ 125 mmol/l, glucose
108 mg/dL and BUN of 140 mg/dL is:
Serum osmolarity= 2 x[Na+] + Glucose/18 + Urea/2.8 = 2x15+108/18+140/2.8= 250+ 6 + 50 =
306
84.
85. Actions of PTH include all of the following, except:
a. Increase bone resoption
b. Increase net acid excretion
c. Increased calcium absorption from GI tract
d. Increased calcium reabsorption in the kidney
e. Increased phosphate excretion
Answer: C (this is a function of vitamin D). So, PTH acts indirectly to increase calcium
absorption from GI tract by increasing the synthesis of vitamin D.
86. The effect of serum osmolarity of a patient with the following labs: Na+ 125 mmol/l,
glucose 108 mg/dL and BUN of 140 mg/dL is:
Serum osmolarity= 2 x[Na+] + Glucose/18 + Urea/2.8 = 2x15+108/18+140/2.8= 250+ 6 + 50 =
306
Answer: D.
86. All of the following factors increase distal tubular secretion of K+, except:
a. Increased serum K+.
b. Increased tubular flow rate
c. Increased serum H+ concentration
d. Increased tubular Cl- concentration
e. Increased aldosterone → Correct
Answer: D?
87. All of the following electrolyte and acid-base disturbances may be seen in a patient
with diabetic ketoacidosis upon presentation, except:
a. Hyponatremia
b. Normal anion gap metabolic acidosis
c. Hyperkalemia
d. Hyperphosphatemia
e. Increased urea
88. All of the following are causes of high turnover bone disease in chronic renal
failure, except:
a. Aluminum toxicity
b. Decreased vitamin D hydroxylation
c. Metabolic acidosis
d. Hyperpohsphatemia
e. Increased parathyroid hormones
Answer: A
89. All of the following are true about chronic myeloid leukemia (CML), except:
a. It is a disease of middle aged which could present with constitutional symptoms
b. The laboratory finding usually show leukocytosis, with left shift and high leucocyte
alkaline phosphatase.
c. It is characterized by specific transloation between chromsomes 9, 22 (Philadelphia
chromosome)
d. Possible treatment for CML include; imtinib ??? and allogenic BMT
e. CML could transfer to AML or ALL
90. All of the following are true about myeloproliferative disorders, except:
a. In polycythemia vera, the serum erythropoietin level is high.
b. In essential thrombocytosis, the bone marrow biospy usually show hypercellular
marrow with increased megakaryocytes
c. Massive splenomegaly in CML and myelofibrosis
d. ?
e. ?
Answer: A.
96. The adrenal glands, one is correct:
a. The zona reticularis is the most important area in the cortex during embryogenesis
b. There is no relation between the adrenal cortex and the adrenal medullar regarding
catecholamine synthesis ➔ Wrong! Cortisol increases catecholamine sysnthesis.
c. Phechromocytoma is associated with high blood pressure and hyperkalemia
d. Atrophy of the glnads is a late sign of autoimmune adrenalitisi
e. Zona fasiculata is the place for adrenal androgen synthesis
Ans:
Ans: C.
Ans: B???
Extra info: Schmitdt syndrome: Primary adrenal insufficiency + Hypothryoidism (and often
type 1 DM).
99. A 17 year old pregnant lady was referred for evaluation of anemia. As a child, she
was hospitalized with pneumonia and visited ER twice with abdominal pain. Two years
ago, she was found anemia and iron was recommended, but intermittently taken. The
examination was unremarkable except for a palpable spleen tip. The Hb was 10 with
ferritin 105 and saturation 18%. The peripheral smear revealed slight hypochrmoasia and
target cells, but no sickle forms. Hb electrophoresis results were HbA 26%, HbF 5%, and
HbS 69%. Which of the following is the most likely diagnosis:
a. ?
b. ?
c. ?
d. ?
e. ?
Answer: D
Final exam 2007 6th year
1. Pulsus paradoxus pulse is felt in ONE of the following.
a- aortic regurgitation
b- aortic stenosis
c- mitral stenosis
d- VSD
e- Cardiac tamponade
2 . A 30-year-old man admitted with right sided hemiplegia.Clinical examination
reveals loss of a wave in JVP with irregular irregular pulse. He has ONE of the
following cardiac rhythm abnormalities.
a- complete heart block
b- atrial fibrillation
c- atrial flutter
d- sinus tachycardia
e- sinus bradycardia
3. Major criteria for Rheumatic fever include all the following Except.
a- carditis
b- Sydenham's chorea
c- Polyarthralgia
d- Erythema marginatum
e- Subcutaneous nodules
4. ONE of the following drugs is LEAST used in treatment of acute sever asthma.
a- nebulized B2 agonist
b- i.v hydrocortisone
c- epinephrine (adrenaline)
d- oxygen
e- i.v . aminophylline
5. Hypoxia (decreased PaO2) and decreased Pa CO2 is found in all the following
Except.
a- left ventricular failure
b- massive pulmonary embolism
c- acute sever asthma
d- acute exacerbation of COPD
e- pneumonia
6. All the following are true following splenectomy Except.
a- thrombocytopenia
b- pneumococcal vaccine should be given
c- annual influenza vaccine should be given
d- long term oral penicillin V 500 mg 12 hourly should be given
e- Howell-Jolly bodies are characteristically seen on blood film.
7. ONE of the following drugs is most appropriate in treatment of pneumocystis
carinii pneumonia.
a- clarithromycin
b- ethambutol
c- azithromycin
d- Trimethoprim-Sulphamethoxazole
e- INH and rifampicine
11. Which one of the following is LEAST useful in assessing patient with a poor
prognosis in community-acquired pneumonia?
a- mental confusion
b- urea of 11.4 mmol/l
c- positive C-reactive protein
d- respiratory rate of 35/ min.
e- age 75 years old.
14. ONE of the following is the most frequent cause of death in acute renal failure.
a) Uremia
b) Pulmonary edema
c) Hyperkalemia
d) Infection
e) Hyponatremia
15. A 29-year-old medical student developed a positive PPD (purified protein
derivative) test. She was started on isoniazid (INH) and rifampin
prophylaxis. Three months into her therapy, she began to experience
pins and needles (parasthesia ) in her lower limbs. Administration of which of
the following vitamins might have prevented these symptoms?
A. Niacin
B. Pyridoxine
C. Riboflavin
D. Thiamine
E. Vitamin C
16. Increased bleeding time and PTT is found in ONE of the following.
a- hemophelia A
b- hemophelia B (Xmas disease)
c- Von Willebrand disease
d- treatment with warfarin
e- idiopathic thrombocytopenic purpura
17.All the following may be found in Iron deficiency anemia Except.
a- Red cell distribution width (RDW) is less than 13.
b- microcytic RBC
c- low serum ferritin
d- low serum iron
e- increased TIBC
18.Bilateral hilar lymph nodes enlargement occurs commonly in all the following
Except.
a- pulmonary Tuberculosis
b- chronic myeloid leukemia
c- non-Hodgkins lymphoma
d- Hodgkin lymphoma
e- sarcoidosis
19.All the following may be found in Intravascular hemolysis Except.
a- increased unconjucated bilirubin
b- increased haptoglobin
c- increased methemalbumin
d- reticulosytosis
e- Hemoglobinurea
20.All the following are causes of WORM autoimmune hemolytic anemia Except.
a- SLE
b- chronic lymphocytic leukemia
c- methyldopa
d- infectious mononucleosis
e- non-Hodgkins lymphoma
21. A 72-year-old woman comes to you to control her high blood pressure (180/100)
mmHg.
What is the ONE target blood pressure in the long term for this patient?
a- <160/90
b- <150/90
c- <145/90
d- <130/85
e- <120/70
22. All the following are true about side effects of anti-diabetic agents Except.
a- metformin carries a risk of lactic acidosis.
b- sulphonylurea is used safely pregnancy
c- glitazones may cause prominent fluid retention
d- insulin may cause lipohypertrophy
e- acarbose causes diarrhea
25.Precipitating factors for hepatic encephalopathy in patient with liver cirrhosis include
all the following Except.
a- occult infection
b- aggressive diuresis
c- gastrointestinal bleeding
d- treatment with oral neomycin
e- excess dietary proteins
26. All the following hepatitis viruses are RNA Except.
a- hepatitis A
b- hepatitis B
c- hepatitis C
d- hepatitis D
e- hepatitis E
28.All the following are found in left sided heart failure Except.
a- bilateral basal creptations
b- third heart sound
c- pulsus alternans
d- raised JVP
e- pulmonary oedema
31. All the following are criteria to define sever attack of ulcerative colitis Except.
a- stool frequency > 10 per day with out blood
b- fever > 37.5 C
c- tachycardia >90/min
d- anemia hemoglobin < 10 gram/dl
e- albumin < 30 g/L
34. A 65-year-old man with liver cirrhosis presented with ascitis,abdominal pain,
tenderness and peripheral edema. A diagnostic tap revealed a neutrophil count of 400
/mm 3(normal < 250).
Which ONE of the following would be of the most immediate benefit ?
a- Fluid restriction and no added salt diet.
b- Intravenous antibiotics.
c- Oral spironolactone.
d- Therapeutic paracentesis
e- Trans-jugular intrahepatic porto-systemic shunt.
35. All the following are recognized complications of Hepatitis C infection Except.
a- diffuse proliferative glomerilonephritis.
b- hepatocellular carcinoma
c- liver cirrhosis
d- chronic hepatitis C infection
e- cryoglobulinemia
36. ONE of the following tests is most suitable in screening patients for celiac disease.
a- Anti-casein antibodies
b- Anti-endomyseal antibodies
c- Anti-gliadin antibodies
d- ESR
e- Aplha feto protein.
41. 20-year old woman presents with a week history of fever, rigor and productive
rusty cough. The chest X-ray shows left lower lobe consolidation. Which ONE of
the following is most appropriate treatment?
a- clarithramycin
b- gentamycin
c- Cotrimoxazole
d- Benzypenicillin
e- Flucloxacillin
42. ONE of the following is most likely diagnosis for patient with thyroid function test
showing elevated serum T4 and low radioactive iodine uptake.
a- Grave's disease.
b- Hashimoto's thyroiditis.
c- subacute thyroiditis.
d- non-toxic goiter.
e- pregnancy.
43. A-25- year old man presents with urethritis, painful swollen left knee and
conjunctivitis.
ONE of the following is most likely diagnosis.
a- SLE
b- Gonococcal arthritis
c- Gout
d- Reiter's syndrome
e- Ankylosising spondylitis
46. All the following are true about calcium metabolism except.
a. calcitonin inhibit bone resorption
b. vit. D3. is hydroxylated in the liver to 25-hydroxycholecalciferol
c-. parathyroid hormone decrease phosphate execretion by the kidneys.
d. parathyroid hormone is increased renal tubular reabsorption of calcium.
e. vit. D deficiency is manifested as low parathyroid hormone level.
47. A 54- year- old male with Child's grade C hepatic encephalopathy presents with
haemetemesis. Which ONE of the following is most appropriate immediate
therapy?
a. i.v desmopressin
b. i.v isosorbide dinitrate
c. i.v. omperazole
d. i.v. propranolol
e. i.v. somatostatin.
48. All the following are risk factors for development of peptic ulcer disease Except.
a. daily use of NSAID
b. gastric infection with H.pylori
c. sever emotional stress.
d. cigarette smoking
e. gastrin-secreting tumors.
50. A29- year-old man presents with symptoms of gastroesophageal reflux. Which ONE
of the following is most useful in assessing the role of surgery.
a. cardiac sphincter manometry.
b. gastric emptying study.
c. intragastric PH monotring off therapy .
d. oesophgeal motility study.
e. oesophgeal PH monotring on therapy
51.All the following are true about hepatitis A ,except.
a. has an incubation period of 2-4 weeks.
b. it is transmitted during vaginal delivery.
c. does not cause chronic hepatitis.
d. may cause hepatosplenomegaly.
e. a vaccine is avalible.
53. All the following may be used in treatment of idiopathic thrombocytopenic purpura
Except.
a. oral predinsolone.
b. Fresh frozen plasma
c. splenectomy
d.I.V. immunioglobulin
e. immunosuppresent drug ( cyclophosphamide)
54.A-23- year old woman presents with lethargy, the following blood results are
obtained. Hb 10.4 g/dl, platelet 268x 10 9/L, WBC 6.3X 10 9/L, MCV 65 fl, Hb A2 9% (
NORMAL < 3.5% ),
Which ONE of the following is the most likely diagnosis?
a. B-Thallassemia minor
b. B-Thallassemia major
c. sickle cell anemia
d. hereditary spherocytosis
e. G6PD deficiency
58. All the following are true about renal osteodystrophy Except.
a. reduced conversion of 25 (OH)2 D3 to 1-25-(OH) 2 D3
b. increased parathyroid hormone
c.increased intestinal calcium absoprption
d. decreased osteoclastic activity
e. increased reabsorption of calcium from bone.
59. All the following may be found in polycythemia rubra vera Except.
a. elevated WBC
b. elevated platelets
c. splenomegaly
d. elevated serum uric acid
e. high erythropoietin level
61. All the following are true about thalassemia major Except
a. Hb electrophoresis shows mainly increase in Hb A2
b. failure to thrive with short stature
c. sever anemia
d. hepatosplenomegaly
e. treatment is by blood transfusion with iron chelating agent ( desferrioxamine)
63. All the following are true about uric acid metabolism except.
a- 2/3 of body uric acid pool is dietary in origin
b- 2/3 is from endogenous purine metabolism
c- 2/3 of uric acid is excreted by the kidney
d- serum uric acid is increased in polycythemia rubra vera
e- serum uric acid is increased in eclampsia of pregnancy.
64- All the following are poor prognostic signs in scleroderma except.
a- old age of onset.
b- limited skin involvement.
c- high ESR
d- renal involvement
e- pulmonary hypertension
66. All the following are most likely causes of pyrexia of unknown origin Except.
a- occult bacterial infection
b- lymphoma
c- factitious fever
d- viral infection
e- SLE
67. A 50-year old woman has pain in her fingers on exposure to cold, arthralgia, and
difficulty in swallowing solid food.
The most useful One test to make a definitive diagnosis is
a- rheumatoid factor
b- anti-nuclear antibody
c- ECG
d- Blood urea and serum creatinin
e- anti-mitochondrial antibody
68. A 20-yea-old male is complaining of arthritis and eye irritation. He has a history of
burring on urination. On examination, he has Right knee effusion and dermatitis of the
glans penis.
Which of the following is ONE most correct statement about this patient?
a- Nisseria gonorrhoeae is likely to be cultured from the glans penis
b- B- the patient is likely to have positive rheumatoid factor
c- An infectious process of the GI tract may precipitate this disease
d- The anti-nuclear antibody is very likely (highly) to be positive
e- There is strong association with HLA-B8 antigen.
69. A pleural effusion analysis results: ratio of concentration of total protein in pleural
fluid to serum of 0. 38 , latate dehydrogenase LDH level of 125 IU, and ratio of LDH in
pleural fluid to serum of 0. 45.
Which of the following ONE disease is the most likely the cause for this pleural effusion.
a- uremia
b- pulmonary embolism
c- sarcoidosis
d- SLE
e- Congestive heart failure
71. A 57-year-old man develops acute shortness of breath shortly after a 20-hour
automobile ride. He has normal physical examination except for tachycardia,ECG:
shows sinus tachycardia, but is otherwise normal.
Which ONE of the following is correct?
a- the patient should admitted to hospital and if there is no contraindication to
anticoagulant, Heparin should be started while waiting for tests.
b- Normal finding on examination of the lower limbs are extremely unusual
c- A definitive diagnosis can be made by history alone
d- Early treatment has little effect on overall mortality
e- The disease can be diagnosed definitely by Chest X-Ray
72. Which ONE of the following Arterial Blood Gases is most likely to be found in a 60-
year-old heavy smoker man, He has chronic bronchitis, peripheral odema and cyanosis?
a- PH 7.50, PO2 75, PCO2 28
b- PH 7.15, PO2 78, PCO2 92
c- PH 7.06, PO2 36, PCO2 95
d- PH 7.06, PO2 108, PCO2 13
e- PH 7.39, PO2 48, PCO2 54
73. A 60-year-old man has an inferior myocardial infarction; his heart rate is 45 /min.
The artery most likely to be involved in this process is:
a- right coronary artery
b- left main artery
c- left anterior descending artery
d- circumflex artery
e- left mammary artery
74. A patient with stable angina on asprine, nitrate and B-Blocker, developed 3 episodes
of sever and long –lasting chest pain each day over the past 3 days.
His ECG and cardiac enzymes are normal.
One of the following is the best treatment
a- admit the patient and start I.V digoxine
b- admit the patient and start I.V heparine
c- admit the patient and start I.V prophylactic streptokinase
d- admit the patient and for observation without changing his medications
e- Discharge the patient with increasing the dose of B-blocker and nitrate
75. ONE of the following drugs reduces myocardial remodeling after acute myocardial
infarction.
a- ACE inhibitors
b- digoxine
c- verapamil
d- furosemide (lasix)
e- hydralazine.
77. A70 hypertensive woman patient with mild left hemiparesis and finding of peristant
atrial fibrillation. Optimal treatment with anti-hypertensive drugs would be ONE of the
following
a- close observation
b- permenant pace maker
c- asprin
d- warfarin
e- I.V heparin
79. Which ONE of the following should be immediately given to a patient with
ventricular fibrillation.
a-I.V amiodrone
b-I.V epinephrinr (adrenaline)
c- defibrillation at 200 joules
d- I.Vadenosine
e-I.V verapamil
80. Which ONE of the following drugs would be most appropriately used in treatment of
patient with inferior myocardial infarction and has a heart rate of 40/minute .
a- atropine
b- digoxine
c- propranolol
d- calcium channel blockers
e- heparine
82. A 50-year-old female , she is 155 cm tall and weighs100 Kg, her fasting bloods sugar
is 150 mg/100 ml on 2 occasions, she is a symptomatic and no abnormal physical signs
on examination.
The treatment of choice include ONE of the following.
a- observation
b- medical nutrition therapy
c- insulin
d- sulphonylurea
e- biguanides ( metformin) !!!!!!!!!!!!!!
84. Hypocalcemia with increased serum phosphate is found in ONE of the following
a- hypoparathyrodism
b- osteomalacia
c- acute pancreatitis
d- chronic renal failure
e- malabsorption
85. All the following may be findings in primary hypoadrenalism (Addison's disease)
Except.
a- hypernitremia with hypokalemia
b- palmer creases skin pigmentatioin
c- impotance and amenorrhoea
d- postural hypotension
e- weight loss
87. Modifiable risk factors for ischemic heart disease include all the following Except.
a- smoking
b- hypertension
c- hyperlipidaemia
d- age
e- diabetes mellitus
88. All the following antibiotics may be used in treatment of H.pylori Except.
a- amoxicillin
b- tetracycline
c- metronodazo;e
d- clarithramycin
e- strepotomycin
89. All the following are found in chronic renal failure Except.
a- hyperkalemia
b- hyperurecemia
c- hypophosphatemia
d- hypocalcemia
e- Low serum erythropitein
91. After undergoing surgical resection for carcinoma of stomach, a 60-year-old male
develop numbness in the lower limb. Blood film shows macrocytosis and MCV = 120 fl.
The abnormality is most likely due to ONE of the following
a- folic acid
b- Vit. B12 …… (IF)
c- thiamin
d- Vit. K
e- Riboflavin
92. ONE of the following is not a disease –modifing anti-rheumatoid arthritis drug.
a- sulfasalazine
b- NSAIDs
c- methotrexate
d- leflunamide
e- sodium aurothiomalate (Gold)
93. All the following are early complications of acute myocardial infarction Except.
a- cardiogenic shock
b- heart block
c- ventricular fibrillation
d- aneurismal dilatation of infracted area
e- sudden cardiac death
94. ECG shows ST elevation in leads II, III, AVF, indicate infarction in ONE of the
following
a- anteroseptal MI
b- anterolateral MI
c- posterior MI
d- inferior MI
e- subendocardial MI
95. All the following ECG findings are found in hypokalemia Except.
a) Flattened T waves
b) U waves
c) Shortened QT interval
d) ST segment depression
e) Ectopic beats
96. ONE of the following is LEAST common cause of Microscopical hematuria
a-Minimal change disease (lipoid nephrosis)
b-Membranous glomerulonephritis
c-Proliferative glomerulonephritis
d-Membranoproliferative glomerulonephritis
e-Lupus nephritis
97. Causes of nephrotic syndrome include all the following Except.
a) SLE
b) DM
c) Amyloidosis
d) Membranous glomerulionephritis
e) Autosomal-dominant polycystic kidney disease
98. All the following are true regarding the pathogenesis of lupus erythematosis
except.
a- the exact cause is unknown.
b- It is a chronic inflammatory disease.
c- the basic pathological unit is vasculitis
d- it is due to type I hypersensitivity reaction.
e- genetic and environmental factors may play a role in the disease.
99. All the following are causing hypokalemia Except.
a- Conn's syndrome
b- Addison's disease
c- B-agonist (salbutamol) therapy
d- Alkalosis
e- Thiazide diuretics
100. Repeated multiple mouth ulcers are seen in the following conditions EXCEPT:
a- Behcet's disease.
b- Systemic lupus erythematosus.
c- Herpes simplex virus infection.
d- Ankylosing spondylitis.
e- Mental stress.
Final Exam past years 2004 - 2007
Q. of unknown origin
Part 1
1 ) A 22 year old female presents to clinic because of severe generalized muscle
weakness .Her Blood pressure 120/70 mm Hg , pulse 76/min . Physical exam is
normal except for the muscle weakness .
Labs : Na 142 meq/L , K 2.8 meq / L , Cl 94 meq / L , Ca 10 mg / dl , Po4 3.5 mg/dl,
CO2 36 meq/L
Urine ;
Ph : 7 , protein : trace elements , Cl 75 meq / L , glucose : negative , K : 74 meq/L , Na :
55 meq /L
The clinical findings are most consistent with :
A .) Hypokalemic periodic paralysis
B ) Villous adenoma
C ) Primary aldosteronism
D ) Bartter s syndrome xxx
E ) Surreptitious vomiting
Peripheral smear shows schistocytes & helmet cells , retic 5 % , fibrin split products 20
mg/dl ( normal < 10 ) .
What is the most likely diagnosis ?
A ) Thrombotic thrombocytopenic purpura
B ) Malignant hypertension
C ) Acute glomerulonephritis
D ) Acute pancreatitis
E ) Fulminant Preeclampsia ( HELLP)
3 ) During a routine physical examination for obtaining health care insurance , a 31 year
old woman is found t have asymptomatic hematuria with 25-30 RBC/HPF in her urine
. Her serum Cr level is 0.9 mg/dl , and her BUN is 12 mg/dl . On questioning , the
patient said that she had a sore hroat within the past month . Physical Exam is
unremarkable & shows no evidence of hypertension or edema . Dipstick analysis
shows only trace proteinuria .
The patient returns for follow up evaluation 2 weeks later , the microscopic hematuria
has resolved .
What is the most appropriate next step you would take to arrive at a diagnosis ?
A ) Order complement , ANA, and ANCA tests
B ) Order a urine test for cytology
C ) Order measurement of Serum IgA level
D ) Repeat urine analysis xxx
E ) Order an intravenous pyelogram
4 ) A 67 year old man with a 4 year history of NIDDM is admitted to the hospital
with DVT in his calf . He is placed at bed rest & given a diet for diabetic patients &
started on heparin therapy . He is treated with his chronic antihypertensive
regimen of Captopril , 25 mg, twice daily
Labs :
Na 138 meq/L, K 4.6 meq/L , HCO3 25 meq/L , Cr 2 mg/dl stable for 2 years ,
5 days later Blood pressure remained stable 135/85 mmHg , but labs became :
glucose 225mg/dl, Na 135 meq/L , k 7 meq/L , HCO3 21 meq/L , Cr 2.4 mg/dl , TTKG
4.
What is the most likely cause of hyperkalemia ?
A ) Acute adrenal hemorrhage
B ) Acute Renal failure
C ) Hyperglycemia
D ) Pulmonary embolus
E ) Hypoaldosteronism xxx
6 ) A 66 year old man with ESRD secondary to hypertension has been receiving
maintenance hemodialysis for 15 years . He is hospitalized for evaluation of neck
and bilateral shoulder pain associated with pain and parasthesias in both hands .
Nerve conduction studies show bilateral median nerve entrapment .
He undergoes a bilateral carpal tunnel release procedure . A skeletal survey shows
numerous periarticular lytic lesions in both humeral heads , the right acetabulum ,
& right scaphoid . Diffuse osteopenia , and subperiosteal erosion of the medial
aspects of the middle phalanges of the hands .
Which of the following procedures is most likely to document the cause of this
patients symptoms ?
A ) MRI of the cervical region and shoulders
B ) An intact parathyroid hormone level test
C ) Congo red staining of the carpal tunnel band or periarticular lytic lesions xxx
D ) A transiliac bone biopsy
E ) A deferoxamine challenge test
7 ) A 19 year old man presents to ER with nausea & vomiting . He has been
attending a wrestling camp for 2 days and had been in good health previously . He
has an oral temperature of 38 ºC , the remainder of his vital signs are normal .
Labs :
Cr 2 mg/dl , Urine analysis : protein + 1 , blood + 3 , microscopic RBC 0-3 / HPF , WBC
0 , casts positive
What is the most appropriate next step :
A ) Prescribe acetaminophen , oral hydration , and abstinence from exercise for the next
3 days
B ) Prescribe Aspirin , oral hydration, & abstinence from exercise for the next 3 days
C ) Admit to hospital & order Iv hydration xxxx
D ) Order a toxic screen for drugs
E ) Order a throat culture and antistreptolysin O titer
BUN 13 mg/dl ,Na 140 meq/L , K 3.8 meq/L , Cl 105 meq/L , Ca 11.9 mg/dl , PO4 3.5
mg/dl , Cr 1.9 mg/dl , alb 4 g/dl , CO2 23 meq/L .
9 ) A 55 year old male has progressive CRI dueto type II Diabetic Nephropathy &
hypertension . His Cr clearance is 23 ml/min , his serum Cr is 3.1 mg/dl . He has
just returned from an introductory educational session regarding dialysis &
transplant options . He asks your opinion about the best options
Which of the following offers the best prognosis for this patient :
A ) NIPD
B ) Hemodialysis
C )Renal transplant xxxx
D ) Combined renal & pancreas transplant
E ) CCPD
10 ) A 41 year old man has had recurrent Calcium oxalate stones for the past 3
years . A recent 24 hour collection showed the following :
Cr 1.56 g/24 hrs
Ca 380 mg/24 hrs , normal ( < 300 mg/24hrs ) .
Urate 740 mg/24 hrs , normal ( < 750 mg/24 hrs )
Oxalate 38 mg/24 hrs , normal ( < 40 mg/24 hrs )
Citrate 643 mg/24 hrs , normal ( 300-700 mg/24 hrs )
Na 104 meq / 24 hrs
Which of the following would be most effective in reducing his urinary Calcium
excretion ?
A ) Dietary Ca restriction
B ) Cranberry juice
C ) Hydrochlorothiazide xxxx
D ) Furosemide
E ) High fluid intake
13 ) After 4 years on dialysis , a 42 year old HCV positive black patient received a
living unrelated transplant from his wife . He is treated with Tacrolimus , Sirolimus
& prednisone .Four months post transplant he has high blood sugar ranging 200-
300 mg/dl .He has no family history of diabetes . His BMI is 35 .
His risk factors for past transplant diabetes include all Except :
A ) Tacrolimus therapy
B ) Increased BMI
C ) HCV infection
D ) Sirolimus therapy xxxx
E ) Ethinicity
14 ) All the following are true in regards to corticosteroid withdrawal after kidney
transplant except :
a ) Acute rejection is more likely in blacks
b ) Avoidance of steroids may be more effective than withdrawal
c ) If achieved without acute rejection , it does not affect graft survival x
d ) Results in acute rejection in approximately 30 % of patients
e ) Reduces the need for antihypertensive therapy .
15 ) The use of which one of the following immunosuppressant therapies is not
Associated with hyperlipidemia ?
A ) Tacrolimus
B ) Cyclosporine
C ) Rapamycine
D ) Mycophenolate mofetil xxxx
E ) Prednisone
Part 2
1- the drug that act on cell wall:
vancomysin
14- peritoneal lavage in trauma patient is not sensitve fer intra peret bleeding
onco
18- radiation not cause lymphocytosis
20- mean age for mammogram check up for normal female y3ne :40 yr
31- nipple retraction msh sign for advanced local malig!!!!!! mn dr.jamal
cardio
about varicise vein
32- most common cause for visiting dr
asthesis
Part3
1. A 50 year old man with no past medical history is found to be in atrial fibrillation
during routine medical examination. He reports no history of palpitation or
dyspnea.Normal physical examination. He refused DC cardioversion. If the patient
remains in chronic Atrial fibrillation.
Which ONE of the following is most suitable treatment to offer?
a- Asprine.**********************
b- warfarin,target INR 2-3.
c- no anticoagulation.
d- warfarin, target INR3-4.
e- warfarin, target INR2-3, for 6 months then Asprin.
6. Major criteria for Rheumatic fever include all the following Except.
f- carditis
g- Sydenham's chorea
h- Polyarthralgia******************
i- Erythema marginatum
j- Subcutaneous nodules
8. ONE of the following drugs is LEAST used in treatment of acute sever asthma.
f- nebulized B2 agonist
g- i.v hydrocortisone
h- epinephrine (adrenaline)********************
i- oxygen
j- i.v . aminophylline
9. Hypoxia (decreased PaO2) and decreased Pa CO2 is found in all the following Except.
f- left ventricular failure
g- massive pulmonary embolism
h- acute sever asthma
i- acute exacerbation of COPD*****************??????????????
j- pneumonia
##12. All the following are true about Giardia lamblia Except.
a- Is usually acquired by ingestion of food or water contaminated by
trphozoites.*******************************
b- Can cause malabsorption
c- Both cystic and trohozoites can be found in stool
d- Can be effectively treated by metonidazole (flagyel)
e- Cysts are destroyed by boiling.
##14. All the following are true about Amoebic liver abscess Except.
a- it can be treated by metronidazole (flagyel)
b- should be aspirated routinely************************************
c- there may be signs of pleural effusion on right side of chest.
d- Usually affect the right lobe of liver.
e- There is often no history of dysentery.
##15. All the following neoplasm have a known infective etiology Except.
a- Burkitt's lymphoma****************************
b- Hepatocellular carcinoma
c- Gastric lymphoma
d- Nasopharyngeal carcinoma
e- Skin basal cell carcinoma
##16. All the following are findings in Visceral leishmaniasis (Kala azar),Except.
a- pancytopenia
b- hypergammaglobulinaemia
c- lymphadenopathy
d- splenomegaly
e- the drug of choice for treatment is chloroquine.*********************
17. ONE of the following drugs is most appropriate in treatment of pneumocystis carinii.
f- clarithromycin
g- ethambutol
h- azithromycin
i- Trimethoprim-Sulphamethoxazole*************
j- INH and rifampicine
18. ONE of the following is the mode of action for B-Blockers in controlling
hypertension.
f- decrease cardiac out put.
g- Slow the heart rate****************************
h- Increase cardiac force of contraction
i- Increase cardiac output
j- Decrease plasma volume
21. Which one of the following is LEAST useful in assessing patient with a poor
prognosis in community-acquired pneumonia?
f- mental confusion
g- urea of 11.4 mmol/l
h- positive C-reactive protein**************************
i- respiratory rate of 35/ min.
j- age 75 years old.
23. ONE of the following ECG changes will NOT be found in patient with
cirrhotic liver and chronic diarrhea who has been taking diuretics
f) Flattened T waves
g) U waves
h) Shortened QT interval********************
i) ST segment depression
j) Ectopic beats
27. All the following are associated with large or normal size kidneys Except:
a) Diabetes
b) Amyloid*******************
c) Scleroderma
d) Acute tubular necrosis
e) Chronic pyelonephritis
31. ONE of the following is the most frequent cause of death in acute renal failure.
e) Uremia
f) Pulmonary edema
g) Hyperkalemia
h) Infection***********************
e) Hyponatremia
33. All the following are causes of low Total gas transfer (TLCO) in respiratory function
test Except.
a- pulmonary fibrosis
b- pulmonary oedema
c- emphysema
d- pulmonary emboli
e- asthma************************************
34. A 71-year-old woman with no significant past medical history is investigated for
generalized tiredness. She has recently lost 7 Kg in weight.
The following blood results are obtained.
Hb: 9.8 g/ dl, platelates: 104 x 10 9/ L, WBC: 70 X 10 9/L
Blood film: small mature lymphoctosis, smudge cell seen, no abnormal (blast) cells.
##35. All the following cardiac lesions are associated with high risk of infective
endocarditis Except.
a- VSD
b- combined mitral valve disease
c- aortis stenosis
d- Atrial septal defect********************************
e- Aortic regurgitation
36. Increased bleeding time and PTT is found in ONE of the following.
a- hemophelia A
b- hemophelia B (Xmas disease)
c- Von Willebrand disease*************************
d- treatment with warfarin
e- idiopathic thrombocytopenic purpura
38. Bilateral hilar lymph nodes enlargement occurs in all the following Except.
a- pulmonary Tuberculosis
b- chronic myeloid leukemia*********************
c- non-Hodgkins lymphoma
d- Hodgkin lymphoma
e- sarcoidosis
40.All the following are causes of WORM autoimmune hemolytic anemia Except.
a- SLE
b- chronic lymphocytic leukemia
c- methyldopa
d- infectious mononucleosis***************************+
e- non-Hodgkins lymphoma
##43. Life threatening complications of multiple myeloma include all the following
Except.
a- renal impairment
b-hypercalcemia
c- hyperurcemia*********************************
d- hyperviscosity due to high level of paraprotein
e- spinal cord compression.
45. A 72-year-old woman comes to you to control her high blood pressure (180/100)
mmHg.
What is the ONE target blood pressure in the long term for this patient?
a- <160/90
b- <150/90
c- <140/90
d- <130/85**********************
e- <120/70
47. All the following are true about anti-diabetic agents Except.
a- metformin carries a risk of lactic acidosis.
b- sulphonylurea is used safely pregnancy*******************
c- glitazones may cause prominent fluid retention
d- insulin may cause lipohypertrophy
e- acarbose causes diarrhea
49. All the following drugs are used in treatment of congestive heart failure Except.
a- bisoprolol
b- metaprolol
c- carvidolol
d- spironolactone
e- propranolol**********************
51. Precipitating factors for hepatic encephalopathy in patient with liver cirrhosis include
all the following Except.
a- occult infection
b- Aggressive diuresis
c- Gastrointestinal bleeding
d- Treatment with oral neomycin***************
e- Excess dietary proteins
53. ONE of the following statements is true about treatment of pulmonary tuberculosis.
a- pyrazinamide may precipitate hyperurecmic gout.******************
b- INH can cause optic neuritis
c- renal impairment with rifampicine
d- streptomycin is causing reversible damage to vestibular nerve
e- hepatitis is usually caused by ehambutol
##54. In patient with coma due to morphine over dose, which ONE of the following
Medications is used to reverse its act.
a- methionine
b- N-acetyl cystine
c- Naloxne********************************
d- Atropine
e- Praladoxine
55. All the following are found in left sided heart failure Except.
a- bilateral basal creptations
b- third heart sound
c- pulsus alternans
d- raised JVP*******************
e- pulmonary oedema
56. All the following are long term complications of sickle cell anemia Except.
a- pulmonary hypertension
b- leg ulcer
c- neurological complications
d- aplastic crisis
e- splenomegaly.**************
##57. All the following are true about secondary syphilis Except..
a- generalized lymphadenopathy
b- condylomata lata
c- mucosal ulceration (snail track ulcer )
d- generalized itchy skin rash*******************************
e- acute neurological signs.
58. A 35-year-old man with type 1 diabetes mellitus, is evaluated for recent onset
morning hypoglycemia. For the last 10 days his morning blood glucose has ranged from
220 mg/dl-300 mg/dl. He has experienced nightmares recently.
Which of the following is best explanation for his morning hyperglycemia.
a- Diabetic nephropathy
b- Under treatment with insulin
c- Overtreatment with insulin**************???????????????????
d- Diabetic neuropathy
e- Hypothyrodism.
59. All the following can cause high prolactin level Except.
a- prolactinoma
b- acromegaly
c- polycystic ovary syndrome
d- metacopramide
e- hyperthyroidism.**********************
63. All the following are true regarding the pathogenesis of lupus erythematosis except.
a- the exact cause is unknown.
b- It is a chronic inflammatory disease.
c- the basic pathological unit is vasculitis
d- it is due to type I hypersensitivity reaction.*************************
e- genetic and environmental factors may play a role in the disease.
64. All the following are criteria to define sever attack of ulcerative colitis Except.
a- stool frequency > 10 per day with out blood***************************
b- fever > 37.5 C
c- tachycardia >90/min
d- anemia hemoglobin < 10 gram/dl
e- albumin < 30 g/L
67. A 32-year-old alcoholic with shock due to bleeding oesphageal varices. After
resuscitation. Which ONE of the following is the treatment of choice.
a- intravenous octreotide.
b- intravenous glypressin
c- oesophagial variceal endoscopy ligation
d- Transjugulartranshepatic portocaval shunt (TIPS)
e- oesophagial variceal sclerotherapy********************************
68.A 65-year-old man with liver cirrhosis presented with ascitis,abdominal pain,
tenderness and peripheral edema. A diagnostic tap revealed a neutrophil count of 400
/mm 3(normal < 250).
Which ONE of the following would be of the most immediate benefit?
f- Fluid restriction and no added salt diet.
g- Intravenous antibiotics.
h- Oral spironolactone.
i- Therapeutic paracentesis**********************************
j- Trans-jugular porto-systemic shunt.
##69. The presence of fecal leukocytes in a sample of stool or rectal mucus is consistent
with all the following causes of diarrhea Ecxept ONE.
a- campylobacter jejuni.
b- Shigella sonnei
c- Giardia Lambilia**********************************
d- Ulcerative colitis
e- Entamoeba histolytica
70. All the following are recognized complications of Hepatitis C infection Except.
a- diffuse proliferative glomerilonephritis.**********************
b- hepatocellular carcinoma
c- liver cirrhosis
d- chronic hepatitis C infection
e- cryoglobulinemia
71. All the following statements are associated with Wilson's disease Except.
a- Kayser-Fleischer rings.
b- haemolysis
c- Elevated serum caerloplasmine******************************
d- Renal tubular acidosis
e- Chorea.
72. ONE of the following tests is most suitable in screening patients for celiac disease.
a- Anti-casein antibodies
b- Anti-endomyseal antibodies******************************
c- Anti-gliadin antibodies
d- Xylose absorption test
e- C-reactive protein.
75. Cardiac risk factors for CNS ischemic stroke are all the followings except:
a- Atrial fibrillation.
b- Supraventricular tachycardia.
c- Myocardial infarction.**************************
d- Left atrial myxoma.
e- Cardiomyopathy.
78. A60-year-old man recently treated for renal tuberculosis, presents with weight loss,
diarrhea, anorexia, and hypotension and is noted to have hyper pigmented buccal mucosa
and hand creases.
ONE of the following is discriminating investigations which is useful in diagnosis.
a- Stool for ova, cysts and parasites.
b- Full blood count.
c- thyroid function test
d- Plasma ACTC and Cortisol*********************************
e- Blood cultures.
79. A20-year-old woman presents with fever, abdominal pain, purpura and focal
neurological signs.
ONE of the following is most likely diagnosis.
a- idiopathic thrombocytopenic purpura
b- thrombotic thrombocytopenic purpura****************************
c- DIC
d- Henoch-Schonlein purpura
e- Von Willebrand's disease.
80. A -30-year old man has, on heart auscultation, loud first heart sound, rumbling mid
diastolic murmur with opening snap.
ONE of the following is most likely diagnosis.
a- Pliable (mobile) mitral valve stenosis.*****************************
b- Calcified (immobile) mitral valve stenosis.
c- Mitral valve prolepses.
d- Aortic regurgitation
e- Mitral regurgitation.
81. A-40-year old man post thyroidectomy for medullary thyroid carcinoma presents
with hypertension and complains of attacks of sever headache and palpitations. He is
noted to have glycosuria.
Which ONE of the following is most likely cause of his hypertension.?
a- Cushining syndrome.
b- Primary hyperaldosternosim.
c- Essential hypertension
d- Pheochromocytoma*******************************************
e- Polyarteritis nodosa.
82. A-20-year old woman presents with a week history of fever, rigor and productive
rusty cough. The X-ray shows left lower lobe consolidation.
Which ONE of the following is most appropriate treatment?
f- Clarithramycin*************************************
g- ciprofloxacin
h- Cotrimoxazole
i- Benzypenicillin
j- Flucloxacillin
83. A-60- year-old female with rheumatoid arthritis presents with splenomegaly.Her
CBC shows: WBC 1500/ mm3, platelates 60000/mm3, Hb 8 g/dl. No blast in peripheral
blood film.
Which ONE of the following is most likely diagnosis?
a- Malaria
b- Lymphoma
c- Polycythemia rubra vera
d- Felty's syndrome**********************************************
e- Gaucher's disease.
84. A patient with mild congestive heart failure is treated with high-dose
furosemide and diureses 25 pounds of fluid. A complete blood count (CBC)
taken before the diuresis shows an RBC count of 4 million/mm3; a CBC
taken after diuresis shows a RBC count of 7 million/mm3. Which of the
ONE of the following is the most likely explanation?
a- Cyanotic heart disease
b- Increased erythropoietin
c- Polycythemia vera
d- Relative polycythemia********************???????????????????/
e- Renal cell carcinoma
85. ONE of the following is most likely diagnosis for patient with thyroid function test
showing elevated serum T4 and low radioactive iodine uptake.
a- Grave's disease.
b- Hashimoto's thyroiditis.
c- subacute thyroiditis.**********************************
d- non-toxic goiter.
e- pregnancy.
86. Blood stored in a blood bank tends, with time, to become relatively depleted of
2,3-diphosphoglycerate. What effect does this have on the hemoglobin-oxygen
dissociation curve?
a- Shifts the curve to the left, so that the hemoglobin has a decreased oxygen affinity
**************************************
b- Shifts the curve to the left, so that the hemoglobin has an increased oxygen affinity
c- Shifts the curve to the right, so that the hemoglobin has a decreased oxygen affinity
d- Shifts the curve to the right, so that the hemoglobin has an increased oxygen affinity
e- Does not change the dissociation curve
87. A-25-year woman who takes oral contraceptive pills, recently developed intermittent
left sided headache with photophobia, vomiting. They occur about once a month and last
24-36 hours.
ONE of the following is most likely diagnosis.
a- tension headache
b- temporal arteritis
c- trigeminal neuralgia
d- migraine********************************
e- Cluster headache.
88.In anaphylactic shock, ONE of the following drugs should be given FIREST.
a- intramuscular epinephrine********************************
b- oral predinsolone
c- oral antihistamine
d- intramuscular antihistamine
e- intravenous atropine.
89. All the following are true regarding the ANA(Antinuclear antibody ) test Except.
a- it is the mainstay test in SLE
b- there are several techniques for making this test.
c- it is highly specific test for SLE.****************************************
d- it may be positive in normal individuals.
e- it is highly sensitive in SLE.
91. An elderly man on treatment for irregular heart rate develop ankle oedema for which
he is given a new drug. 2 weeks later he develops complete heart block, nausea and
complains of seeing " yellow".
ONE of the following drug combination is most likely the cause for above complaints.
a- theophylline and erythromycin.
b- Propranolol with verapamil
c- Amiodarone with captopril
d- Digoxin with frusemide (lasixs)******************************
e- ACE inhibitor and atenolol.
92. A 52-year-old man presents to his physician after a community health screening test
reveals a fasting glucose of 170 mg/dL. Physical examination is remarkable for bronze
skin pigmentation, hepatomegaly, splenomegaly, and limitation of motion in the second
and third metacarpophalangeal joints of both hands. The man has no known history of
hemolytic anemia, and takes daily multivitamins without minerals.
Which ONE of the following pigments is most likely present in the
man's liver?
a- Bilirubin
b- Carotene
c- Ferritin ***************************
d- Lipofuscin
e- Melanin
93. A 45-year-old homeless man has a chronic cough, a cavitary lesion of the
lung, and is sputum positive for acid-fast bacilli. Which ONE of the following
is the principle form of defense by which the patient's body fights this
infection?
a- Antibody-mediated phagocytosis
b- Cell-mediated immunity******************
c- IgA-mediated hypersensitivity
d- IgE-mediated hypersensitivity
e- Neutrophil ingestion of bacteria
94. A 54-year-old male with acute lymphocytic leukemia develops a blast crisis. He is
treated with intensive systemic chemotherapy. Following treatment, the patient will be at
increased risk for the development of ONE of the following.
a- bile pigment gallstones
b- cholesterol gallstones
c- cystine kidney stones
d- struvite kidney stones
e- uric acid kidney stones *****************************
95. A 30-year-old pregnant woman complains to her physician of feeling very tired
during her pregnancy. A complete blood count with differential reveals a Hg 10 g/dl,
with hypersegmented neutrophils and large red cells. Deficiency of which ONE of the
following would be most likely to produce these findings?
a- Ascorbic acid
b- Calcium
c- Copper
d- Folate ****************************
e- Iron
96. A 26-year-old man presents to his physician with a chronic cough. The man is a
smoker, and states that he also gets frequent headaches and aches in his legs when he
exercises. Chest x-ray demonstrates notching of his ribs.
Which ONE of the following undiagnosed congenital defects may be
responsible for these findings?
a- Coarctation of the aorta ******************
b- Eisenmenger's syndrome
c- Tetralogy of Fallot
d- Transposition of great vessels
e- Ventricular septal defect
97. All the following statements regarding the uric acid are true Except.
a- two third of the body uric acid pool is dietary in origin. *****************
b- two third is from endogenous purine metabolism.
d-normal serum uric acid level dose not exclude acute gouty arthritis.
e- there are variation in normal values between male and female.
98. A 42-year-old female presents with a recent onset of fatigue, malaise, constipation,
and a 12-pound weight gain. On examination, her thyroid is firm and enlarged. What
ONE laboratory test is most likely to confirm the expected diagnosis?
a- Antithyroid antibodies
b- Serum thyroid-stimulating hormone (TSH) measurement *****************
c- Serum thyroxine (T4) measurement
d- Serum triiodothyronine (T3) measurement
e- T3 resin uptake
99. 5 mL of synovial fluid is aspirated from an inflamed knee joint. The fluid contains
Needle-shaped, strongly negatively birefringent crystals. These crystals most likely to
have ONE of the following compositions?
a- Basic calcium phosphate
b- Calcium oxalate
c- Calcium pyrophosphate dihydrate
d- Cholesterol
e- Monosodium urate ***************************
All the following have a role in the progression of his renal disease except :
2. All the following are true about Focal Segmental Sclerosis ( FSGS) except :
a) Familial type has better prognosis
b) Progresses fast in Renal failure
c) Collapsing type is associated with HIV
d) Recurrence after transplant is high
e) Main presentation is Nephrotic syndrome
3. 25 year old female presented to OPD with Bp 160/100 , she stated that her Bp was the same over
the past 2 weeks .
All the following are first line investigations for this patient except :
a) Urine analysis
b) Serum K, Urea ,Cr
c) Lipid profile
d) Fasting blood sugar
e) MRA for renal arteries
4. 35 year old female previously healthy presented to ER c/o generalised weakness , Bp 110/80 .
Irregular irregular pulse
Labs : K 2.5 meq / L , Cl 100 meq /L , Na 135 meq /L
All the following can be in the differential diagnosis of this case except :
a) Bulimia
b) Barter syndrome
c) Hypercalcemia
d) Primary Hyperaldosteronism
e) Diuretic abuse
5. 20 year old male came to OPD with c/o of passing red urine , which was preceded by URTI the
previous morning .
Upon exam Bp 170/95 otherwise Negative :
6. 25 year old female was admitted to hospital with referred to OPD due to incidental finding of the
following labs & ABG :
PH 7.32 , HCO3 15
a) Acetazolamide treatment
b) Fanconi syndrome
c) Treatment with Thiazide
d) Primary hyper parathyroid
e) Diarrhea
7. 70 year old male presented to OPD with Bp 180/80 , he had similar readings over the last month
otherwise asymptomatic . The best management for this patient is :
a) Observation
b) Start on Enalapril 5 mg Q day
c) Start on Nifidipine 20 mg + Thiazide 25 mg
d) Start on Furosemide 40 mg Q day
e) Start on α Methyl Dopa 250 mg 1 x 3
8. 50 year old patient previously healthy presented with hemoptysis & hematurea
Labs : Cr 3.0 mg/dl , Urea 70 mg/dl , PO4 5 mg/dl , Ca 9.2 mg/dl . Hb 13 .Kidney biopsy showed 55 %
crescents with linear deposits on basement membrane by IF . The best management is :
a) Prednisone 20 mg Q day
b) Azathioprine 50 mg Q day + Cyclosporin 1mg/kg/day
c) Plasmapheresis + Methylprednisone IV + Cyclophosmide PO
d) Prednisone 30 mg Q day alternating with Chlorambucil
e) Tacrolimus 1 mg 1x2
9. 40 year old male know to have Nephrotic syndrome for 15 years , now presenting with Cr 8.0 mg/dl ,
Urea 100 mg/dl , All the following are indications to start this patient on dialysis except :
a) Peripheral Neuropathy
b) Anemia
c) Pericarditis
d) Low albumin
e) Bleeding tendency
10. All the following are true about Diabetic Nephropathy except :
a) More likely to occur if patient ha siblings with Nephropathy
b) More severe in black
c) Occurs within 5 years in I DDM
d) It needs 15 years to progress into ESRD after start of overt proteinurea
e) Mostly preceeded by Diabetic Retinopathy
11. 60 year old male known to have Diabetes for 10 years and is on Enalapril 10 mg 1x2 , presented to
ER because all his peripheral extremities became paralised , Labs K 8.0 meq /L , Cr 1.0 mg/dl. Which of the
following should be used first in the management of this patient :
a) NAHCO3 Iv
b) Ventolin nebuliser
c) Glucose + insulin Iv
d) K exalate ( Na polysterene Sulfonate )
e) Ca gluconate Iv
12. 25 year old male presented c/o of polyurea and weakness , Bp 110/70 , Labs : K 3.0 meq/l , PH 7.46
, HCO3 32 . All the following could be part of the dfferential of the above case except :
a) Barter Syndrome
b) Furosemide abuse
c) Hypercalcemia
d) Excessive Licorice ingestion
e) Gittleman syndrome
13. All the following are true for a Diabetic with ESRD except
a) Oral hypoglycaemic agents should be stopped
b) First year post transplant survival is the same as in the general population
c) More prone to hypotension during HD than other patients
d) They have higher Insulin requirements
e) PD is associated with increase in Triglycerides level
14. A 40 year old female known to have Membranous GN came to OPD with 24 hour protein 4 gm/24
hours , Cr 1.0 mg/dl , Urea 40 mg/dl . Her Bp 160/100 , she was started on Enalapril 20 mg 1x1 . The desirable
Bp reading in such a lady should be :
a) 140/90
b) 130/85
c) 120/75
d) 130/80
e) 135/85
15. 20 year old male has a LRD kidney transplant 2 years ago , he is not known to be Diabetic nor
Hypertensive . His medications are Tacrolimus 3 mg 1x2 , prednisone 5mg 1x2 , MMF 1 gm 1x2, Labs : FBS
400 mg/dl , Cr 1.0 mg/dl , Urea 35 mg/dl , Tacrolimus level 12 . The next step in managing his Diabetes other
than start him on treatment and re checking his sugar level is :
a) Stop Prednisone
b) Decrease MMF to 500 mg 1x2
c) Stop Tacrolimus
d) Stop MMF , and increase Tacrolimus
e) Decrease Tacrolimus to 2 mg 1x2
16. Each of the glomerular lesions listed below can cause Nephrotic syndrome . Which of them may be
found in all the following conditions : non – Hodgkins lymphoma , hepatitis B, hepatitis C , and infective
endocarditis ?
a) Focal and segmental glomerulosclerosis
b) Minimal change disease
c) Membranous nephropathy
d) Type I membranoproliferative glomerulonephritis ( with subendothelial deposits )
e) Type II membranoproliferative glomerulonephritis ( dense deposit disease )
17. 83 year old male who has DM , CHF,CRI is admitted to hospital with volume overload & Cr 4.0 mg/dl
( baseline 2.3 mg/dl ) . He was treated by Iv diuretics , post voiding residual was 250 ml after foleys catheter
was inserted . He was discharged 2 days later with Cr 3.0 mg/dl . One week lter he came to OPD , Cr is 3.5
mg/dl , ultrasound shows mild bilateral hydronephrosis . Which of the following would best predict the effect
of the patient bladder outlet problem on kidney function :
a) Serum PSA
b) Serum Cr after several days with foleys catheter
c) Kidney size on U/S
d) Retrograde urography
e) Renal Scan
18. 49 year old female is evaluated in ER after being found lying in the street in a semiconscious state ,
she is known to have hypertension and a history of seizures. Lab : BUN 79 mg/dl , Cr 8.7 mg/dl , Na 138
meq/l , K 4.2 meq/l , Cl 60 meq/l , HO3 54 meq/l . ABG PH 7.43 , PCO2 85 mmHg. Which of the following Acid
Base disorder is most compatible with these lab findings
a) Metabolic Acidosis and Metabolic Alkalosis
b) Metabolic Acidosis and Respiratory Acidosis
c) Metabolic Acidosis and Metabolic Alkalosis and Respiratory Acidosis
d) Metabolic Alkalosis and Respiratory Acidosis
e) Metabolic Acidosis
Part 5
1. Which one of the following arterial blood gas sets on room air is compatable with
completely compensated metabolic acidosis?
A B C D E
PH 7.44 7.38 7.60 7.36 7.56
PaC02 mmHg 26 25 25 95 40
Bicarb. mEq 18 15 24 49 34
The pH must be normal. Therefore, exclude “E” and “C”. The correction will be respiratory in the form of
“washed-out” CO2 need to be low. Therefore, exclude D. Bicarbonate will be low. The remaining options are A &
B.
2. Lung’s failure type respiratory failure is characterized by which one o the followings :
a. Normal chest X ray .
b. Hypocapnia or normocapnia .
c. Diffusion is the main mechanism of hypoxia .
d. Easy to correct hypoxia .
e. PEEP is contraindicated .
Respiratory failure is a syndrome in which the respiratory system fails in one or both of its gas exchange
functions: oxygenation and carbon dioxide elimination. In practice, it may be classified as either hypoxemic or
hypercapnic.
Hypoxemic respiratory failure (type I) is characterized by an arterial oxygen tension (Pa O2) lower than 60 mm
Hg with a normal or low arterial carbon dioxide tension (Pa CO2). This is the most common form of respiratory
failure, and it can be associated with virtually all acute diseases of the lung, which generally involve fluid filling
or collapse of alveolar units. Some examples of type I respiratory failure are cardiogenic or noncardiogenic
pulmonary edema, pneumonia, and pulmonary hemorrhage.
Hypercapnic respiratory failure (type II) is characterized by a PaCO2 higher than 50 mm Hg. Hypoxemia is
common in patients with hypercapnic respiratory failure who are breathing room air. The pH depends on the
level of bicarbonate, which, in turn, is dependent on the duration of hypercapnia. Common etiologies include
drug overdose, neuromuscular disease, chest wall abnormalities, and severe airway disorders (eg, asthma
and chronic obstructive pulmonary disease [COPD]).
Answer: A.
4. In patients with idiopathic pulmonary fibrosis (usual interstitial pneumonia) all of the
followings are expected patho physiological changes EXCEPT :
a. Low DLCO .
b. Decreased FEV1/FVC .
c. Severe O2 desaturation on exercise.
d. Reduced vital capacity and total lung capacity .
e. Increased pulmonary artery pressure
Answer: B (increased FEV1/FVC ratio).
6. All of the following statement regarding lung cancer are true EXCEPT :
a. Small cell lung carcinoma metastasis late in the course of the disease
b. Adenocarcinoma usually is a peripheral lung tumor .
c. Adenocarcinoma in some cases is difficult to be differentiated from mesothelioma .
d. Thromboembolic disease can be the first manifestation of the disease.
e. Surgery can be curative for early diagnosed cases .
Answer: “A. Compared to non-small cell lung cancer, small cell lung cancer is just bad disease. The tumor grows
fast and metastasizes early. Small cell is more often associated with paraneoplastic syndromes (e.g., Eaton-
Lambert) and ectopic hormonal syndromes (e.g., SIADH).
7. All of the followings are useful for the assessment of the severity of an attack of
bronchial asthma, EXCEPT :
a. Spirometry .
b. Methacholine test
c. ABG (arterial blood gases)
d. Peak expiratory flow rate
e. Physical examination.
Answer: B. Methacholine tes: methacholine challenge test: a test that involves the inhalation of increasing
concentrations of methacholine, a potent bronchoconstrictor, in patients with possible bronchial hyperreactivity;
usually performed when a diagnosis of asthma or bronchospastic lung disease is not clinically obvious. Source:
Stedman’s.
8. Which one of the following pulmonary function values indicates airflow limitation
a. FEV1 of 60% of predicted .
b. FVC of 60% of predicted .
c. FEV1/FVC of 60% of predicted .
d. DLCO of 60% of predicted.
e. Residual volume of 60% of predicted.
Answer: C. FEV1/FVC of 60%. Total lung capacity (TLC) is used to assess interstitial lung disease. Expiratory flow
rate (FEV1/FVC is used to assess obstructinve lung disease. Airway obstruction is diagnosed when the FEV1/FVC
is <0.7 (70%0). (Source: MedStudy Pulmonology 2013, p. 6)
9. Wide alveolar-arterial Po2 (PA-a O2) gradient can be increased in all of the following
conditions EXCEPT:
a. Morphine overdose .
b. Severe pneumonia .
c. Acute Bronchial Asthma .
d. Acute Pulmonary edema .
e. ARDS (acute respiratory distress syndrome)
Answer: In morphine overdose ➔ Hypoventilation ➔ No washout of alveolar CO2 and replacement with new O2
➔ Both arterial and alveolar O2 are decreased. Therefore, the PAa O2 gradient is decreased.
Answer: A? Hypercapnia ➔ metabolic acidosis?. For hypoxia and deoxygenated hemoglobin, it is true that they
cause dyspnea. But in metabolic acidosis, for example, there is no hypoxia. Nonetheless, there is “dyspnea”.
12. All of the followings are true combination between a risk factor and pathogens causing
pneumonia EXCEPT :
a. Alcoholism and klebsella pneumonia
b. Old age and mycoplasma pneumonia
c. Cigarette smoking and H .infleunza
d. Mechanical ventilation and pseudomonal pneumonia.
e. Abnormal level of consciousness and anaerobic bacteria
Answer: I don’t know and I don’t want to know! Most probably, this is not required from us! Medscape article
about polysmnography: http://emedicine.medscape.com/article/1188764-overview#showall
Part 6
1. All of the following statements regarding acute lower limb ischemia are true
except:
A) Acute lower limb ischemia of thrombotic origin generally has a long history of
intermittent claudication
B) In cases of thrombosis, the embolic source of acute lower limb ischemia is
generally present
C) Treatement of acute lower limb ischemia of embolic origin is embolectomy and
anticoagulation
D) Arteries in acute lower limb ischemia of embolic origin are soft to tender
E) In acute lower limb ischemia of embolic origin , contralateral pulses are
generally present
11) Which of the following regarding congenital anterior chest wall deformity is
true?
A) Pectus carinatum is the most common type
B) Its etiology is well established
C) A familial tendency is rare
D) Cardiopulmonary derangement constitutes its main operative indication
E) Its repair is recommended around the preschool age.
12) All of the following are accepted treatment options for empyema except:
A) Thoracocentesis
B) Closed tube thoracostomy
C) Thoracotomy + decortication
D) Thoracotomy +lung resction.
E) Open drainage + rib resection
13) Which of the following is the most common clinical manifeatation of major
pulmonary embolism?
A) Tachypnea.
B) Tachycardia
C) Pleural pain
D) Cough
E) Rales
14) A 65-year diabetic and hypertensive patient is complaining of symptomatic
aorto-iliac occlusive disease. Which of the following therapeutic modality is not
applicable on him?
A) Aortofemoral bypass
B) Aortoiliac endarterectomy
C) Extra anatomic by pass
D) Angioplasty/stenting
E) Lumbar sympathectomy.
15) All of the following regarding varicose veins(VV) are true except:
A) Varicose veins are the most common vascular disorders affecting the human
being
B) Leg paraesthesia is the most common late post operative complication.
C) The majority of patients with varicose veins are treated conservartively
D) Large thigh VV should be treated surgically
E) Varicose veins with l sapheno-femoral incompetence should be treated with
sclerotherapy.
16) All of the following are accepted surgical indication for patients with lung
abscess except;
A) Failed medical treatment
B) Serious hemorrhage
C) Suspicion of cancer
D) Unsuccessful drainage
E) An abscess of 3 cm diameter.
19) All of the following regarding intercostals tube drainage for pneumothorax are
true except:
A) It is done in cases of unsuccessful simple aspiration or catheter aspiration drainage
B) It is especially recommended in secondary spontaneous pneumthorax
C) A non-bubbling chest tube should not usually be clamped
D) Bubbling chest tube should never be clamped
e) A patient with a non-bubbling and clamped chest tube for pneumothorax can leave the
ward environment.
20) All of the following regarding chest drain suction are true except:
A) Suction to an intercostal tube should not be applied directly after tube insertion
B) Suction to an intercostal tube is recommended in cases of lung failure to re-
expand
C) When applying suction,high volume, low pressure (-10 to –20 cm H2O) suction
systems are recommended
D) Suction to an intercostal tube should can be applied in cases of persistent air leak
E) Patients requiring suction can be managed on outpatient basis.
23) All of the following are causes of exudative pleural effusion except:
A) Malignancy
B) Trauma
C) Collagen vascular disease
D) Infection
E) Congestive heart failure.
Part7
1 ) 35 year old man presented to ER after an episode of Grand mal seizure and by exam he was afebrile , Bp
130/95 and confused .
Labs showed : Cr 1.0 mg/dl , BUN 12mg/dl , Na 140 meq/L , K 4.8 meq /L , Cl 100 meq/L , HCO3 12 meq/L .
Which of the following is the most appropriate initial treatment for the Metabolic Acidosis :
1 L of 5 % dextrose in H2O & HCO3 3 ampoules ( 150 meq ) infused over 3 hours
Hemodialysis
Fomepizole
2 ) All the following affect short term survival in kidney transplant except :
HLA antibodies
Acute rejection
Donor illness
3 ) 25 year old man was found to have microscopic hematuria by chance . Urine analysis showed many RBC
/HPF, no RBC casts , no Dysmorphic RBC
The most appropriate counseling for this patient includes which of the following :
Advice him that this disorder is likely to progress to CRF over 10-20 years
Advice him that this is a benign finding , there may be a risk for nephrolithiasis but it never progresses to
Renal failure
Advice him to have his children undergo genetic testing and get treatment early
5 ) 35 year old female is evaluated because of an elevated Bp 160/105 for the past 2-3 months . Her mother
has hypertension and kidney disease , and a maternal aunt is now on hemodialysis
Labs : Cr 0.8 mg/dl , Na 140 meq/ L , K 5.0 meq /L , Cl 102 meq / L , HCO3 25 MEQ / l , Urine Analysis is
negative .Which of the following is most likely to provide information regarding cause of her hypertension .
Renal U/S
Plasma PTH
6 ) All the following are Renal changes expected in normal pregnancy except :
7 ) 50 year old man has history of recurrent kidney stones which were Ca containing stones .
All of the following are risk factors for formation of these stones except :
Hypercalciuria
Hyperoxaluria
Hypercitrauria
Hyperuricosuria
8 ) All the following can be clinical & lab manifestations of hpokalemia except :
Nephrogenic DI
Tubular vacuolization
Rhabdomyolysis
Decreased Amoniagenesis
Tetany
9 ) According to National kidney foundation guidelines . All the following are acceptable target PTH levels in
CRF as per stage except :
10 ) All the following diuretic site of action combinations are true except :
Acetazolamide inhibits CA in PT
A. Mood changes
B. Iron deficiency
C. B12 deficiency
D. Unexplained elevation of liver enzymes
E. Recurrent abdominal pain
4. All of the following factors are associated with rapid progression of chronic
hepatitis C to cirrhosis except:
7. All of the following are risk factors for Squamous cell carcinoma of the
esophagus except:
A. Zinc Deficiency
B. Low serum Selenium
C. Infection with Human Papilloma virus
D. Chronic Gastroesophageal reflux disease
E. Alcoholism
8. All of the following are accepted initial management strategies in patients with
upper GI bleeding except:
A. Somatostatin
B. Bleeding scan
C. Esophagogastrodeudeoscopy EGD
D. Acid suppressive medication
E. Gastric Lavage
10. All of the following medications are being used for Non Alcohol steatohepatitis
NASH except:
A. Betaine
B. Ursodeoxycholic acid
C. Ribavirin
D. Vitamin E
E. Beta- Carotene
11. In treatment of patients with Spontanous Bacterial peritonitis, all of the following
are true except:
A. Initiate therapy when ascitic fluid Neutophils > 250/mm2
B. Majority sterile at presentation (culture negative)
C. Gentamycin is the drug of choice
D. Treat for at least 5 days
E. 30% of patients are Asymptomatic at presentation and during follow up
12. In regard to hepatitis C and pregnancy, all of the following is true except:
A. The rate of transmission from mother to baby during delivery is around 6%.
B. Transmission is higher in vaginal delivery comparing to cesserian.
C. Higher rate of transmission is seen if the mother is co-infected with HIV
D. Severe hepatitis is rare in infected infants
Breast-feeding is safe
13. All of the following medications are being used in chronic hepatitis B except:
A. Lamivudine
B. Ribavirin
C. Pegylated interferon
D. Adefovir Dipivoxil
E. Entecavir
14. All of the following micro organisms can cause infectious diarrhea with positive
fecal leucocytes except:
A. Shigella
B. Yersinia
C. Giardia
D. Campylobacter
E. Salmonella
15. All of the following are protective from colo-rectal cancer except:
A. Aspirin
B. Folic Acid
C. Fiber Diet
D. Calcium
E. Moderate use of Alcohol
Part 8
Q3. Which of the following is not a criteria for diagnosing Sphincter of Oddi
Dysfunction
a) CBD diameter more than 12 mm on USG
b) Decrease in CBD pressure after infusion of Cholecystokinin
c) Ampullary pressure more than 40 mm Hg
d) Delayed emptying of contrast from CBD after ERCP
Q7) All segments of liver drain into Right Hepatic Duct except?
A) I
B) III
C) V
D) Viii
Q21 After liver resection earliest proliferation is of which kinds of liver cells occurs?
a) Parenchymal
b) Ductal
c) Canalicular
d) Non parenchymal
Q26 Which of the following is not true about TME (Total Mesorectal Excision)
a) It improves survival
b) It decreases local recurrence
c) It is associated with with increased blood losss
d) Associated with higher anastomotic leakage as compared to Low Anterior Resection
e) Associated with pelvic sepsis
Q31 What is not true regarding Siguira's procedure for Portal Hypertension
a) Transesophageal variceal ligation
b) Splenectomy
c) Vagotomy
d) Pyloroplasty
Q35. Write True or false for the following statements for Extra Hepatic Biliary atresia
a)More in premature babies
b)Jaundice occurs after 1wk
c) surgery should be done after 1year
d) Most common operation is Kasai operation
e) Most common indication for liver transplant in children
f) only intrahepatic tree is involved
Q40. Contraindication for resection of locally recurent rectal cancer are all except
a) Extrapelvic disease
b)Sciatic pain
c) Bilateral ureteric obstruction
d) S1 or S2 nerve inolvement
e) Circumferential or extensive pelvic side wall involvement
f) None
Q43. In liver laceration if there is uncontrolled bleeding after pringles's maneuver what
is the likely cause
a) arterial bleeding
b) capillary bleed
c) hepatic vein bleed
d) portal vein bleed
Q44. A 3cm non bleeding liver laceration, what is the optimal intraoperative
management
a) suture the laceration
b) Mesh closure of the laceration
c) Peritoneal Toileting and close abdomen
d) Use a binding glue
a) Turcot syndrome
b) cowden syndrome
c) Juvenile polyposis coli
d) none
Q56. which of the following statements regarding arterial blood supply of the colon is
incorrect
a) Marginal artery of Drummond is a collateral vessel that connects SMA with IMA
b)Arc of Riolon or meandering mesenteric artery connects proximal SMA with proximal
IMA
c) Right colic artery is the most constant branch of Ileo colic artery
d) Splenic flexure is the area of watershed
Q57. which of the following about blood supply to the liver is incorrect
a)Portal vein provides 75% of total blood flow
b)Hepatic artery provides 75% of total blood flow
c)Portal flow provides 50-70% of total oxygen deman
d)Increase in Hepatic artery flow is autoregulated and increase
in hepatic artery flow decreases portal blood flow
Q59) Which of the following statements regarding the quantitative tests is false
a)Aminopyrine breath test based on clearance by hepatic p450 depends on functional
hepatic mass
b)Aminopyrine breath test is mainly used for prognosis in chronic liver disease but its
not useful for subclinical hepatic dysfunction.
c)Lidocaine and MEGX have some value in transplant population
d)Indocyanine green has a role in predicting prognosis in cirrhosis patients undergoing
resection
e)None of the above
2. The initial treatment of choice for a symptomatic patient with isolated pulmonic
stenosis is
A. closed surgical blade valvotomy
B. open surgical valvotomy
C. balloon catheter valvuloplasty
D. Blalock-Taussig shunt
E. valve replacement
For the following question one or more of the answers is correct , select :
A. if only 1,2,3 are corret.
B. if if only 1,3 are corret
C. only 2,4 are corret
D. if only 4 is corret
E. all are correct
3. Factors that are thought contribute to maintaining the patency of the ductus arteriousis
are
1.increased pulmonary vesicular resistance secondary to hypoxia.
2. high arterial oxygen tension.
3. prostaglandin.
4. acetylcholine
4. All of the followinq are acyanotic child may have increased pulmonary marking on x-
ray ,except:
A. ASD
B. VSD
C. PDA
D. Endocardial cushing defect
E. Pulmonary stenosis.
5. all are mechanisms of increase risk of infections in minimal lesion nephrotic syndrome
, except:
a.Loss of immunoglobulin
b.Loss of Properdin factor B
c.Decreased perfusion of the spleen
d.Loss of opsonizination factors
e.leukopenia
10. 11 month-old girl presents to your office with fever (39.c) for the last 2 days. 3 hrs
ago she started to to have vomiting and decrease oral intake, she looked tired and ill. Her
exam reveals no focus and moderate to severe dehydration. you suspect UTI
19. Her urine culture is positive at 24 hrs, the most likely organism , is:
a. klebsiella
b. E. coli
c. staph. Aureus
d. proteus
e. enterococcus
11. After treating her infection, what investigation(s) needed?
a. no test are needed
b. renal U /S
c. VCUG
d. DTPA
e. renal U /S & VCUG
12. an 4-year old boy presented with his mother with abnormal posturing of the hands,
the mother stated that his hands turned into a claw shape, first time noticed by his teacher
in the day care center for mental retarded kids. This is his photo you were surprised by
his funny looking and laughing.
In your cardiac exam, you heard a systolic murmur, you conclude that he has at most:
a. VSD
b. subvalvular aortic stenosis
c. supravalvular aortic stenosis
d. ASD
e. critical pulmonary stenosis with right sided aortic arch
Cardiology
Part 1 (top secret)
1.One of the following is the principle symptoms of heart desease :
a- chest pain with deap inspiration
b- nerveousness
c- chest pain on movement ?xxx
d- edema of the lower limbs
e- pain in the right arms
18.The leading cause of early death in patients with acute myocardial infarction is
a-Rupture of the myocardial wall
b-Rupture of the septum
c-Rupture of the cordae tendinea leading to acute mitral regurgitation
d-Ventricular arrhythmias ####
e-Ventricular aneurysm
19.The best description of Angina is:
a-pain in both hands
b-pain in the back of the chest
c-interscapular pain
d-retrosternal heaviness xxx
d-Sharp pain comes on movement or breathing
21.All the following are found in left sided heart failure Except.
a- bilateral basal creptations
b- third heart sound
c- pulsus alternans
d- raised JVP xxx
e- pulmonary oedema
29.In conductive system of the heart muscle, all of the following are ture, except:
a-Conduction started in SA node. AV node, bundle of His, left and right bundle branch-
purkinjee fibers.
b-Left bundle branch is shorter than right bundle
c-Right bundle supplies right ventricle and left bundle supplied left ventricle and spetum
d-Action potential in the ventricle is rapid and generated by rapid transmembrane K
diffusion xxx
31.You are examining a 63-year old man. You hear a blowing diastolic murmur at
the right upper sternal border. What is the probable diagnosis?
a-Mitral stenosis
b-Mitral regurgitation
c-Aortic stenosis
d-Aortic regurgitation ####
e-Tricuspid regurgitation
33.All the following drugs reduce mortality in patient with congestive heart failure
except:
a Angiotensin receptor blockers
b ACE inhibitors
c B blocker
d Loop diuretic ####
e Spinono lactone
36.You are examining a 63-year old man. You hear a blowing diastolic murmur at
the right upper sternal border. What is the probable diagnosis?
a-Mitral stenosis
b-Mitral regurgitation
c-Aortic stenosis
d-Aortic regurgitation ####
e-Tricuspid regurgitation
39.All of the following are risk factors for coronary artery disease EXCEPT:
a Morbid obesity
b Diabetes mellitus
c Elevated HDL ####
d Elevated LDL
e Elevated homocysteine
40.Which one of the following ECG changes is most typical of hyper kalemia:
a-Peaked P wave
b-Presence of U wave
c-Packed T wave ####
d-ST depression
e-Narrowed QRS complex
43. a 50 year old was found to have a heart murmur. On examination his BP in the
right arm is 160/100 and in the right leg 120/80. CXR showed rib notching in the
upper ribs. What’s the likely Diagnosis?
a-Coarctation of the aorta xxx
b-Supravalvular aortic stenosis
part 2
1-In ASD the second heart sound is best described by the following:
7-Edema, ascites , enlarged liver and venous pressure of 180mm. of saline suggest:
A. Laennec’s cirrhosis
B. Congestive failure
C. Interior vena caval obstruction
D. Acute glomerulonephritis
E. Cirrhosis of the liver
14) Pathophysilogical abnormalities in heart failure include all of the following except :
a) Reduced myocyte shortening and wall motion
b) Sodium retention and circulatory congestion
c) Systemic vasodilation that increase impedence of the LV ejection
d) Structural remodelling and dilation of the LV
e) Renin-Angiotensin-Aldosterone activation
16) Systems responsible for BP regulation include all of the following except :
a) Heart
b) Blood vessels
c) Kidney
d) Baroreceptors in aortic arch and carotid sinuses
e) Direct CNS control
19) Major manifestations of acute rheumatic fever include all of the following except :
25)A disease modifying anti rheumatic drugs (DMARD) include all f the following
except for :
A)Salazo pyrine.
B)Hydroxychloro quine.
C)Colchicine.
D)Methotrexate
E)leflenamide
26)All of the following are criteria for Behcet disease except for :
A)Mouth ulcer's.
B)Arterial Anuyresm .
C)Hypopyron..
D)Pethergy test.
E)Acne-like lesion
29)All of the following are indications for the treatment of Gouty arthritis except for:
A)Chronic Gouty arthritis.
B)Renal stones.
C)Renal failure.
D)Serum uric acid more than 8mg in men.
E)All of the above.
8- On examining the radial pulses, all the following are essential except :
a- rate of the pulses
b- rhythm of the pulses weather is regular or irregular
c- the volume of the pulse
d- the character of the pulse
e- thrill of the pulse
12- fixed splitting of the second heart sound occurs in one of the following :
a-left bundle branch block
b- Atrial septal defect
c-hypertension
d-aortic stenosis.
e- left ventricular outflow obsruction
18- The best description of the physiological cause of a heart murmur are except:
a-turbulant blood flow
b-increase blood flow through a normal valve.
c-increase blood flow through an abnormal valve.
d-occurs in preganancy and athletes
a-Aortic regurgitation
b-Thyrotoxosis
c-Increase the number of sweat glands
d-Anxiety neurosis
e-Acromegaly
a- LVEnlargement
b- RT Ventricular Enlargement
c- LA enlargement
d_RA enlargement
e- Aortic enlargement
a- The length of the bladder cuff should be double than the width
b- The bladder of the cuff should cover the Brachial artery
c- Systolic blood pressure should be messured only by palpatory method
d-Diastolic blood pressure is the point which the sounds becomes muffled
e-The patient should have an exercise before measuring the blood pressure
a-Chest pain
b-SOB
c-Palpitation
d-Syncopy
e-Cough and expectoration
8-In ASD the second heart sound is best described by the following:
ONE of the following is the mode of action for B-Blockers in controlling hypertension.
a- decrease cardiac out put.
b- Slow the heart rate
c- Increase cardiac force of contraction
d- Increase cardiac output
e- Decrease plasma volume
31. A 72-year-old woman comes to you to control her high blood pressure (180/100)
mmHg.What is the ONE target blood pressure in the long term for this patient?
a- <160/90
b- <150/90
c- <145/90
d- <130/85
e- <120/70
38. All the following are found in left sided heart failure Except.
a- bilateral basal creptations
b- third heart sound
c- pulsus alternans
d- raised JVP
e- pulmonary oedema
77. Modifiable risk factors for ischemic heart disease include all the following Except.
a- smoking
b- hypertension
c- hyperlipidaemia
d- age
e- diabetes mellitus
81. A 57-year-old man develops acute shortness of breath shortly after a 20-hour
automobile ride. He has normal physical examination except for tachycardia,ECG:
shows sinus tachycardia, but is otherwise normal.
Which ONE of the following is correct?
a- the patient should admitted to hospital and if there is no contraindication to
anticoagulant, Heparin should be started while waiting for tests.
b- Normal finding on examination of the lower limbs are extremely unusual
c- A definitive diagnosis can be made by history alone
d- Early treatment has little effect on overall mortality
e- The disease can be diagnosed definitely by Chest X-Ray
83. A 60-year-old man has an inferior myocardial infarction; his heart rate is 45 /min.
The artery most likely to be involved in this process is:
a- right coronary artery
b- left main artery
c- left anterior descending artery
d- circumflex artery
e- left mammary artery
84. A patient with stable angina on asprine, nitrate and B-Blocker, developed 3 episodes
of sever and long –lasting chest pain each day over the past 3 days.
His ECG and cardiac enzymes are normal.
One of the following is the best treatment
a- admit the patient and start I.V digoxine
b- admit the patient and start I.V heparine
c- admit the patient and start I.V prophylactic streptokinase
d- admit the patient and for observation without changing his medications
e- Discharge the patient with increasing the dose of B-blocker and nitrate
85. ONE of the following drugs reduces myocardial remodeling after acute myocardial
infarction.
a- ACE inhibitors
b- digoxine
c- verapamil
d- furosemide (lasix)
e- hydralazine.
87. A70 hypertensive woman patient with mild left hemiparesis and finding of peristant
atrial fibrillation. Optimal treatment with anti-hypertensive drugs would be ONE of the
following
a- close observation
b- permenant pace maker
c- asprin
d- warfarin
e- I.V heparin
89. Which ONE of the following should be immediately given to a patient with
ventricular fibrillation.
a-I.V amiodrone
b-I.V epinephrinr (adrenaline)
c- defibrillation at 200 joules
d- I.Vadenosine
e-I.V verapamil
90. Which ONE of the following drugs would be most appropriately used in treatment of
patient with inferior myocardial infarction and has a heart rate of 40/minute .
a- atropine
b- digoxine
c- propranolol
d- calcium channel blockers
e- heparine
93. All the following are early complications of acute myocardial infarction Except.
a- cardiogenic shock
b- heart block
c- ventricular fibrillation
d- aneurismal dilatation of infracted area
e- sudden cardiac death
94. ECG shows ST elevation in leads II, III, AVF, indicate infarction in ONE of the
following
a- anteroseptal MI
b- anterolateral MI
c- posterior MI
d- inferior MI
e- subendocardial MI
95. All the following ECG findings are found in hypokalemia Except.
a) Flattened T waves
b) U waves
c) Shortened QT interval
d) ST segment depression
e) Ectopic beats
Answer:B
65. In conductive system of the heart muscle, all of the following are ture, except:
a. Conduction started in SA node. AV node, bundle of His, left and right bundle
branch-purkinjee fibers.
b. Left bundle branch is shorter than right bundle
c. Right bundle supplies right ventricle and left bundle supplied left ventricle and
spetum
d. Action potential in the ventricle is rapid and generated by rapid transmembrane K
diffusion.
67. The least common ause of AF is? Or: The commonest cause of AF?
a. WPW syndrome
b. Mitral valve disease
c. Hypertension يمكن
d. Pericarditis
e. Thyrotoxicosis
Answer: E Conditions that may cause myocardial ischemia include Coronary artery
disease (atherosclerosis). Atherosclerosis occurs when plaques made of cholesterol and
other cellular waste products build up on your artery walls and restrict blood flow.
Atherosclerosis of the heart arteries is called coronary artery disease and is the most
common cause of myocardial ischemia.
Blood clot. The plaques that develop in atherosclerosis can rupture, causing a blood clot,
which may lead to sudden, severe myocardial ischemia, resulting in a heart attack.
Coronary artery spasm. A coronary artery spasm is a brief, temporary tightening
(contraction) of the muscles in the artery wall. This can narrow and briefly decrease or
even prevent blood flow to part of the heart muscle. Coronary artery spasms are more
common in people with risk factors for heart disease, such as high cholesterol and high
blood pressure, but the spasms can happen in people who have no risk factors, too.
Coronary artery spasms can also occur in people who have conditions that affect their
immune systems, such as lupus.
Severe illnesses. Myocardial ischemia can occur when the metabolic demands of your
heart increase or when blood pressure is very low due to infection, bleeding or other
severe illness. Source: http://www.mayoclinic.com/health/myocardial-
ischemia/DS01179/DSECTION=causes
72. In acute MI< all of the following are true, except:
a. Inf MI, St elevation in 1, 2, AVF
b. Anteroseptal MI – ST segment elevation in V1-V2-V3
c. In acute MI, thrombolytic therapy achieve 100% reperfusion arate.
d. Treatment of MI include morphine, coronary vasodilation, aspirin.
e. Cardiac markers in acute MI, serial cardiac enzymes, like CPK, troponin.
73. Clinical features in infective endocarditis include all of the following, except:
a. Appearnace of new murmur or change in the quantiy of eisting murmur
b. Fever
c. CHF
d. Skin and eye lesions
e. No splenomegaly
Answer: E.
12- signs of Cor pulmonale (RT ventricular failure) all are true except:
a- RT ventricular heave
b- Congested neck vein
c- Tricuspid regurgitation
d- Wheezing chest on examination
e- Pan systolic murmur at the apex transmitted to axilla
20-IN idiopathic hypertrophic sub aortic stenosis (IHSS)all are false One is true:
a) it is a type of dilated cardiomyophy
b) AN important cause of sudden death in athelets
c)Left ventricle is dilated
d) pulse examination is normal in character
e)need nitrate for treatment
a) Mortality from CVD is 2-8 folds higher in people with diabetes than in those without.
b) Thiazolidine are Synthetic legends of Newer transcription factor PPARY
c) Recent meta analysis questioned the cardiac safety of rosiglitazone by decreasing LV
contractility.
d) In recent studies ENHANCED and ACCORD study –mortality has increased in
diabetes in those
with intersively lowering blood glucose.
e)Thiazolidinediones enhances insulin sensitivity in patients with high risk of CV events.
a) The most Common congenital Anomaly is bicuspid Arotic valve which is about 1-2%.
b)US guidelines do not recommend balloon valvoplasty in adults because of high risk of
complications >10%.
c) patients with low cardiac output usually have a small aortic area and small gradient ,
this can be distinguished by doing Dobutamine stimulation.
d)Degenerative Arotic scleosis is distinguished from Arotic Stenosis by valve thickening
and calcification without obstruction of significant gradient.
e) In US guidelines severe stenosis valve area is 1-1.5/cm2 and the mean gradient is 25-
40 mmHg
a) Methoxamine
b) Valsalva maneuver
c) Hemorrhage
d) Rapid heart rate
e) digitalis
a-Aortic regurgitation
b-Thyrotoxosis
c-Increase the number of sweat glands
d-Anxiety neurosis
e-Acromegaly
3-RT ventricular heave occur in one of the following:
a- LVEnlargement
b- RT Ventricular Enlargement
c- LA enlargement
d_RA enlargement
e- Aortic enlargement
a- The length of the bladder cuff should be double than the width
b- The bladder of the cuff should cover the Brachial artery
c- Systolic blood pressure should be messured only by palpatory method
d-Diastolic blood pressure is the point which the sounds becomes muffled
e-The patient should have an exercise before measuring the blood pressure
a-Chest pain
b-SOB
c-Palpitation
d-Syncopy
e-Cough and expectoration
8-In ASD the second heart sound is best described by the following:
17- On examining the radial pulses, all the following are essential except :
f- rate of the pulses
g- rhythm of the pulses weather is regular or irregular
h- the volume of the pulse
i- the character of the pulse
j- thrill of the pulse
12- fixed splitting of the second heart sound occurs in one of the following :
a-left bundle branch block
b- Atrial septal defect
c-hypertension
d-aortic stenosis.
e- left ventricular outflow obsruction
18- The best description of the physiological cause of a heart murmur is:
a-turbulant blood flow
b-increase blood flow through a normal valve.
c-increase blood flow through an abnormal valve.
d-occurs in preganancy and athletes
e-all of the above.
6- ORGANISM RESPOSIBLE OF SUBACUTE ENDOCARDITIS one is true
a)streptococcus pyogenes
b)B haemolytic streptoccous
c)streptococcus viridians
d)stafyllococcus aures
e)H pylori
8-In ASD the second heart sound is best described by the following:
3. Major criteria for Rheumatic fever include all the following Except.
k- carditis
l- Sydenham's chorea
m- Polyarthralgia
n- Erythema marginatum
o- Subcutaneous nodules
4. ONE of the following drugs is LEAST used in treatment of acute sever asthma.
k- nebulized B2 agonist
l- i.v hydrocortisone
m- epinephrine (adrenaline)
n- oxygen
o- i.v . aminophylline
5. Hypoxia (decreased PaO2) and decreased Pa CO2 is found in all the following Except.
k- left ventricular failure
l- massive pulmonary embolism
m- acute sever asthma
n- acute exacerbation of COPD
o- pneumonia
9. A healty patient who is HLA-B27 is most likely to develop ONE of the following.
a- psoratic arthritis
b- enteropathic spondylitis
c- gonococcal arthritis
d- Reiters disease
e- ankylosing spondylitis
11. Which one of the following is LEAST useful in assessing patient with a poor
prognosis in community-acquired pneumonia?
k- mental confusion
l- urea of 11.4 mmol/l
m- positive C-reactive protein
n- respiratory rate of 35/ min.
o- age 75 years old.
14. ONE of the following is the most frequent cause of death in acute renal failure.
i) Uremia
j) Pulmonary edema
k) Hyperkalemia
l) Infection
e) Hyponatremia
16. Increased bleeding time and PTT is found in ONE of the following.
a- hemophelia A
b- hemophelia B (Xmas disease)
c- Von Willebrand disease
d- treatment with warfarin
e- idiopathic thrombocytopenic purpura
18.Bilateral hilar lymph nodes enlargement occurs commonly in all the following
Except.
a- pulmonary Tuberculosis
b- chronic myeloid leukemia
c- non-Hodgkins lymphoma
d- Hodgkin lymphoma
e- sarcoidosis
20.All the following are causes of WORM autoimmune hemolytic anemia Except.
a- SLE
b- chronic lymphocytic leukemia
c- methyldopa
d- infectious mononucleosis
e- non-Hodgkins lymphoma
21. A 72-year-old woman comes to you to control her high blood pressure (180/100)
mmHg. What is the ONE target blood pressure in the long term for this patient?
a- <160/90
f- <150/90
g- <145/90
h- <130/85
i- <120/70
22. All the following are true about side effects of anti-diabetic agents Except.
a- metformin carries a risk of lactic acidosis.
b- sulphonylurea is used safely pregnancy
c- glitazones may cause prominent fluid retention
d- insulin may cause lipohypertrophy
e- acarbose causes diarrhea
25.Precipitating factors for hepatic encephalopathy in patient with liver cirrhosis include
all the following Except.
a- occult infection
b- aggressive diuresis
c- gastrointestinal bleeding
d- treatment with oral neomycin
e- excess dietary proteins
28.All the following are found in left sided heart failure Except.
a- bilateral basal creptations
b- third heart sound
c- pulsus alternans
d- raised JVP
e- pulmonary oedema
31. All the following are criteria to define sever attack of ulcerative colitis Except.
a- stool frequency > 10 per day with out blood
b- fever > 37.5 C
c- tachycardia >90/min
d- anemia hemoglobin < 10 gram/dl
e- albumin < 30 g/L
34. A 65-year-old man with liver cirrhosis presented with ascitis,abdominal pain,
tenderness and peripheral edema. A diagnostic tap revealed a neutrophil count of 400
/mm 3(normal < 250).
Which ONE of the following would be of the most immediate benefit ?
k- Fluid restriction and no added salt diet.
l- Intravenous antibiotics.
m- Oral spironolactone.
n- Therapeutic paracentesis
o- Trans-jugular intrahepatic porto-systemic shunt.
35. All the following are recognized complications of Hepatitis C infection Except.
a- diffuse proliferative glomerilonephritis.
b- hepatocellular carcinoma
c- liver cirrhosis
d- chronic hepatitis C infection
e- cryoglobulinemia
36. ONE of the following tests is most suitable in screening patients for celiac disease.
a- Anti-casein antibodies
b- Anti-endomyseal antibodies
c- Anti-gliadin antibodies
d- ESR
e- Aplha feto protein.
41. 20-year old woman presents with a week history of fever, rigor and productive rusty
cough. The chest X-ray shows left lower lobe consolidation.
Which ONE of the following is most appropriate treatment?
k- clarithramycin
l- gentamycin
m- Cotrimoxazole
n- Benzypenicillin
o- Flucloxacillin
42. ONE of the following is most likely diagnosis for patient with thyroid function test
showing elevated serum T4 and low radioactive iodine uptake.
a- Grave's disease.
b- Hashimoto's thyroiditis.
c- subacute thyroiditis.
d- non-toxic goiter.
e- pregnancy.
43. A-25- year old man presents with urethritis, painful swollen left knee and
conjunctivitis.
ONE of the following is most likely diagnosis.
f- SLE
g- Gonococcal arthritis
h- Gout
i- Reiter's syndrome
j- Ankylosising spondylitis
46. All the following are true about calcium metabolism except.
a. calcitonin inhibit bone resorption
b. vit. D3. is hydroxylated in the liver to 25-hydroxycholecalciferol
c-. parathyroid hormone decrease phosphate execretion by the kidneys.
d. parathyroid hormone is increased renal tubular reabsorption of calcium.
e. vit. D deficiency is manifested as low parathyroid hormone level.
47. A 54- year- old male with Child's grade C hepatic encephalopathy presents with
haemetemesis. Which ONE of the following is most appropriate immediate therapy?
a. i.v desmopressin
b. i.v isosorbide dinitrate
c. i.v. omperazole
d. i.v. propranolol
e. i.v. somatostatin.
48. All the following are risk factors for development of peptic ulcer disease Except.
a. daily use of NSAID
b. gastric infection with H.pylori
c. sever emotional stress.
d. cigarette smoking
e. gastrin-secreting tumors.
50. A29- year-old man presents with symptoms of gastroesophageal reflux. Which ONE
of the following is most useful in assessing the role of surgery.
a. cardiac sphincter manometry.
b. gastric emptying study.
c. intragastric PH monotring off therapy .
d. oesophgeal motility study.
e. oesophgeal PH monotring on therapy
53. All the following may be used in treatment of idiopathic thrombocytopenic purpura
Except.
a. oral predinsolone.
b. Fresh frozen plasma
c. splenectomy
d.I.V. immunioglobulin
e. immunosuppresent drug ( cyclophosphamide)
54.A-23- year old woman presents with lethargy, the following blood results are
obtained. Hb 10.4 g/dl, platelet 268x 10 9/L, WBC 6.3X 10 9/L, MCV 65 fl, Hb A2 9% (
NORMAL < 3.5% ),
Which ONE of the following is the most likely diagnosis?
a. B-Thallassemia minor
b. B-Thallassemia major
c. sickle cell anemia
d. hereditary spherocytosis
e. G6PD deficiency
55.A 70-year-old woman is referred to hospital due to evidences of congestive heart
failure. Blood test reveal the following: Hb 7.4 g/dl, MCV 124 fl, platelets 98 x10 9/l,
WBC 3X10 9/L,
All the following investigations are required to reach a diagnosis Except.
a. Schilling test
b. Intrensic factor antibodies
c. antiparitel cell antibodies
d. bone marrow aspiration, looking for megaloblasts
e. C-reactive protein.
58. All the following are true about renal osteodystrophy Except.
a. reduced conversion of 25 (OH)2 D3 to 1-25-(OH) 2 D3
b. increased parathyroid hormone
c.increased intestinal calcium absoprption
d. decreased osteoclastic activity
e. increased reabsorption of calcium from bone.
59. All the following may be found in polycythemia rubra vera Except.
a. elevated WBC
b. elevated platelets
c. splenomegaly
d. elevated serum uric acid
e. high erythropoietin level
11
61. All the following are true about thalassemia major Except
a. Hb electrophoresis shows mainly increase in Hb A2
b. failure to thrive with short stature
c. sever anemia
d. hepatosplenomegaly
e. treatment is by blood transfusion with iron chelating agent ( desferrioxamine)
63. All the following are true about uric acid metabolism except.
a- 2/3 of body uric acid pool is dietary in origin
b- 2/3 is from endogenous purine metabolism
c- 2/3 of uric acid is excreted by the kidney
d- serum uric acid is increased in polycythemia rubra vera
e- serum uric acid is increased in eclampsia of pregnancy.
64- All the following are poor prognostic signs in scleroderma except.
a- old age of onset.
b- limited skin involvement.
c- high ESR
d- renal involvement
e- pulmonary hypertension
66. All the following are most likely causes of pyrexia of unknown origin Except.
a- occult bacterial infection
b- lymphoma
c- factitious fever
d- viral infection
e- SLE
67. A 50-year old woman has pain in her fingers on exposure to cold, arthralgia, and
difficulty in swallowing solid food.
The most useful One test to make a definitive diagnosis is
a- rheumatoid factor
b- anti-nuclear antibody
c- ECG
d- Blood urea and serum creatinin
e- anti-mitochondrial antibody
68. A 20-yea-old male is complaining of arthritis and eye irritation. He has a history of
burring on urination. On examination, he has Right knee effusion and dermatitis of the
glans penis.
Which of the following is ONE most correct statement about this patient?
a- Nisseria gonorrhoeae is likely to be cultured from the glans penis
b- B- the patient is likely to have positive rheumatoid factor
c- An infectious process of the GI tract may precipitate this disease
d- The anti-nuclear antibody is very likely (highly) to be positive
e- There is strong association with HLA-B8 antigen.
69. A pleural effusion analysis results: ratio of concentration of total protein in pleural
fluid to serum of 0. 38 , latate dehydrogenase LDH level of 125 IU, and ratio of LDH in
pleural fluid to serum of 0. 45.
Which of the following ONE disease is the most likely the cause for this pleural effusion.
f- uremia
g- pulmonary embolism
h- sarcoidosis
i- SLE
j- Congestive heart failure
71. A 57-year-old man develops acute shortness of breath shortly after a 20-hour
automobile ride. He has normal physical examination except for tachycardia,ECG:
shows sinus tachycardia, but is otherwise normal.
Which ONE of the following is correct?
f- the patient should admitted to hospital and if there is no contraindication to
anticoagulant, Heparin should be started while waiting for tests.
g- Normal finding on examination of the lower limbs are extremely unusual
h- A definitive diagnosis can be made by history alone
i- Early treatment has little effect on overall mortality
j- The disease can be diagnosed definitely by Chest X-Ray
72. Which ONE of the following Arterial Blood Gases is most likely to be found in a 60-
year-old heavy smoker man, He has chronic bronchitis, peripheral odema and cyanosis?
a- PH 7.50, PO2 75, PCO2 28
b- PH 7.15, PO2 78, PCO2 92
c- PH 7.06, PO2 36, PCO2 95
d- PH 7.06, PO2 108, PCO2 13
e- PH 7.39, PO2 48, PCO2 54
73. A 60-year-old man has an inferior myocardial infarction; his heart rate is 45 /min.
The artery most likely to be involved in this process is:
f- right coronary artery
g- left main artery
h- left anterior descending artery
i- circumflex artery
j- left mammary artery
74. A patient with stable angina on asprine, nitrate and B-Blocker, developed 3 episodes
of sever and long –lasting chest pain each day over the past 3 days.
His ECG and cardiac enzymes are normal.
One of the following is the best treatment
f- admit the patient and start I.V digoxine
g- admit the patient and start I.V heparine
h- admit the patient and start I.V prophylactic streptokinase
i- admit the patient and for observation without changing his medications
j- Discharge the patient with increasing the dose of B-blocker and nitrate
75. ONE of the following drugs reduces myocardial remodeling after acute myocardial
infarction.
a- ACE inhibitors
b- digoxine
c- verapamil
d- furosemide (lasix)
e- hydralazine.
77. A70 hypertensive woman patient with mild left hemiparesis and finding of peristant
atrial fibrillation. Optimal treatment with anti-hypertensive drugs would be ONE of the
following
a- close observation
b- permenant pace maker
c- asprin
d- warfarin
e- I.V heparin
79. Which ONE of the following should be immediately given to a patient with
ventricular fibrillation.
a-I.V amiodrone
b-I.V epinephrinr (adrenaline)
c- defibrillation at 200 joules
d- I.Vadenosine
e-I.V verapamil
80. Which ONE of the following drugs would be most appropriately used in treatment of
patient with inferior myocardial infarction and has a heart rate of 40/minute .
a- atropine
b- digoxine
c- propranolol
d- calcium channel blockers
e- heparine
82. A 50-year-old female , she is 155 cm tall and weighs100 Kg, her fasting bloods sugar
is 150 mg/100 ml on 2 occasions, she is a symptomatic and no abnormal physical signs
on examination.
The treatment of choice include ONE of the following.
f- observation
g- medical nutrition therapy
h- insulin
i- sulphonylurea
j- biguanides ( metformin) !!!!!!!!!!!!!!
84. Hypocalcemia with increased serum phosphate is found in ONE of the following
a- hypoparathyrodism
b- osteomalacia
c- acute pancreatitis
d- chronic renal failure
e- malabsorption
85. All the following may be findings in primary hypoadrenalism (Addison's disease)
Except.
a- hypernitremia with hypokalemia
b- palmer creases skin pigmentatioin
c- impotance and amenorrhoea
d- postural hypotension
e- weight loss
86.All the following are true about nephrotic syndrome Except.
a- dietary sodium restriction is initial treatment.
b- high protein diet (120-150 gram) daily is recommended
c- prolong bed rest should be avoided as thromboembolism is common.
d- Sepsis is the major cause of death
e- hyperlipdemia is responsible for increase risk of ischemic heart disease.
87. Modifiable risk factors for ischemic heart disease include all the following Except.
a- smoking
b- hypertension
c- hyperlipidaemia
d- age
e- diabetes mellitus
88. All the following antibiotics may be used in treatment of H.pylori Except.
a- amoxicillin
b- tetracycline
c- metronodazo;e
d- clarithramycin
e- strepotomycin
89. All the following are found in chronic renal failure Except.
a- hyperkalemia
b- hyperurecemia
c- hypophosphatemia
d- hypocalcemia
e- Low serum erythropitein
91. After undergoing surgical resection for carcinoma of stomach, a 60-year-old male
develop numbness in the lower limb. Blood film shows macrocytosis and MCV = 120 fl.
The abnormality is most likely due to ONE of the following
f- folic acid
g- Vit. B12 …… (IF)
h- thiamin
i- Vit. K
j- Riboflavin
92. ONE of the following is not a disease –modifing anti-rheumatoid arthritis drug.
a- sulfasalazine
b- NSAIDs
c- methotrexate
d- leflunamide
e- sodium aurothiomalate (Gold)
93. All the following are early complications of acute myocardial infarction Except.
a- cardiogenic shock
b- heart block
c- ventricular fibrillation
d- aneurismal dilatation of infracted area
e- sudden cardiac death
94. ECG shows ST elevation in leads II, III, AVF, indicate infarction in ONE of the
following
a- anteroseptal MI
b- anterolateral MI
c- posterior MI
d- inferior MI
e- subendocardial MI
95. All the following ECG findings are found in hypokalemia Except.
f) Flattened T waves
g) U waves
h) Shortened QT interval
i) ST segment depression
j) Ectopic beats
98. All the following are true regarding the pathogenesis of lupus erythematosis except.
a- the exact cause is unknown.
b- It is a chronic inflammatory disease.
c- the basic pathological unit is vasculitis
d- it is due to type I hypersensitivity reaction.
e- genetic and environmental factors may play a role in the disease.
100. Repeated multiple mouth ulcers are seen in the following conditions EXCEPT:
a- Behcet's disease.
b- Systemic lupus erythematosus.
c- Herpes simplex virus infection.
d- Ankylosing spondylitis.
e- Mental stress.
1. A 50 year old man with no past medical history is found to be in atrial fibrillation
during routine medical examination. He reports no history of palpitation or
dyspnea.Normal physical examination. He refused DC cardioversion. If the patient
remains in chronic Atrial fibrillation.
Which ONE of the following is most suitable treatment to offer?
a- Asprine.**********************
b- warfarin,target INR 2-3.
c- no anticoagulation.
d- warfarin, target INR3-4.
e- warfarin, target INR2-3, for 6 months then Asprin.
##4. All the following occurs usually in 3rd week of Enteric (Typhoid ) fever Except.
f- meningitis
g- lobar pneumonia
h- maculopapular rash (rose spots)*************************
i- oestomyelitis
j- intestinal perforation
6. Major criteria for Rheumatic fever include all the following Except.
p- carditis
q- Sydenham's chorea
r- Polyarthralgia******************
s- Erythema marginatum
t- Subcutaneous nodules
8. ONE of the following drugs is LEAST used in treatment of acute sever asthma.
p- nebulized B2 agonist
q- i.v hydrocortisone
r- epinephrine (adrenaline)********************
s- oxygen
t- i.v . aminophylline
9. Hypoxia (decreased PaO2) and decreased Pa CO2 is found in all the following Except.
p- left ventricular failure
q- massive pulmonary embolism
r- acute sever asthma
s- acute exacerbation of COPD*****************??????????????
t- pneumonia
##12. All the following are true about Giardia lamblia Except.
f- Is usually acquired by ingestion of food or water contaminated by
trphozoites.*******************************
g- Can cause malabsorption
h- Both cystic and trohozoites can be found in stool
i- Can be effectively treated by metonidazole (flagyel)
j- Cysts are destroyed by boiling.
26- On examining the radial pulses, all the following are essential except :
k- rate of the pulses
l- rhythm of the pulses weather is regular or irregular
m- the volume of the pulse xxx
n- the character of the pulse
o- thrill of the pulse
18- The best description of the physiological cause of a heart murmur is:
a-turbulant blood flow
b-increase blood flow through a normal valve.
c-increase blood flow through an abnormal valve. xxx
d-occurs in preganancy and athletes
e-all of the above.
Gastroenterology
16.One of the following is the least likelyOne of the following is the least likely
future of Hemochromatosis:
A. Fulminant liver failure
B. Pseudogout
C. Diabetes
D. Bronze skin
E. Hepatocellular carcinoma
A. Mood changes
B. Iron deficiency
C. B12 deficiency
D. Unexplained elevation of liver enzymes
E. Recurrent abdominal pain
20.All of the following factors are associated with rapid progression of chronic
hepatitis C to cirrhosis except:
23.All of the following are risk factors for Squamous cell carcinoma of the
esophagus except:
A. Zinc Deficiency
B. Low serum Selenium
C. Infection with Human Papilloma virus
D. Chronic Gastroesophageal reflux disease
E. Alcoholism
24.All of the following are accepted initial management strategies in patients with
upper GI bleeding except:
A. Somatostatin
B. Bleeding scan
C. Esophagogastrodeudeoscopy EGD
D. Acid suppressive medication
E. Gastric Lavage
26.All of the following medications are being used for Non Alcohol steatohepatitis
NASH except:
A. Betaine
B. Ursodeoxycholic acid
C. Ribavirin
D. Vitamin E
E. Beta- Carotene
27.In treatment of patients with Spontanous Bacterial peritonitis, all of the following
are true except:
A. Initiate therapy when ascitic fluid Neutophils > 250/mm2
B. Majority sterile at presentation (culture negative)
C. Gentamycin is the drug of choice
D. Treat for at least 5 days
E. 30% of patients are Asymptomatic at presentation and during follow up
28.In regard to hepatitis C and pregnancy, all of the following is true except:
A. The rate of transmission from mother to baby during delivery is around 6%.
B. Transmission is higher in vaginal delivery comparing to cesserian.
C. Higher rate of transmission is seen if the mother is co-infected with HIV
D. Severe hepatitis is rare in infected infants
Breast-feeding is safe
29.All of the following medications are being used in chronic hepatitis B except:
A. Lamivudine
B. Ribavirin
C. Pegylated interferon
D. Adefovir Dipivoxil
E. Entecavir
30.All of the following micro organisms can cause infectious diarrhea with positive
fecal leucocytes except:
A. Shigella
B. Yersinia
C. Giardia
D. Campylobacter
E. Salmonella
23. A 32-year-old alcoholic with shock due to bleeding oesphageal varices. After
resuscitation.Which ONE of the following is the treatment of choice.
a- intravenous octreotide.
b- intravenous glypressin
c- oesophagial variceal endoscopy ligation
d- Transjugulartranshepatic portocaval shunt (TIPS)
e- oesophagial variceal sclerotherapy
25. All the following are recognized complications of Hepatitis C infection Except.
a- diffuse proliferative glomerilone0phritis.
b- hepatocellular carcinoma
c- liver cirrhosis
d- chronic hepatitis C infection
e- cryoglobulinemia
26 ONE of the following tests is most suitable in screening patients for celiac disease.
a- Anti-casein antibodies
b- Anti-endomyseal antibodies
c- Anti-gliadin antibodies
d- ESR
e- Aplha feto protein.
57. A 54- year- old male with Child's grade C hepatic encephalopathy presents with
haemetemesis. Which ONE of the following is most appropriate immediate therapy?
a. i.v desmopressin
b. i.v isosorbide dinitrate
c. i.v. omperazole
d. i.v. propranolol
e. i.v. somatostatin.
58. All the following are risk factors for development of peptic ulcer disease Except.
a. daily use of NSAID
b. gastric infection with H.pylori
c. sever emotional stress.
d. cigarette smoking
e. gastrin-secreting tumors.
78. All the following antibiotics may be used in treatment of H.pylori Except.
a- amoxicillin
b- tetracycline
c- metronodazo;e
d- clarithramycin
Strepotomycin -f
38.One of the following is least likely feature of hemochromatosis:
a. Fulminant liver failure
b. Psuedogout
c. Diabetes
d. Bronze skin
e. Hepatocellular carcinoma
Answer: A (akeed)
39.All the following are subclinical presentations of celiac disease, except:
a. Mood changes
b. Iron def
c. B12 dfe
d. Unexplained elevation of liver enzymes
e. Recurrent abdominal pain
Ans: C (Mild to moderate anemia is present in 50% of cases. Folate deficiency is
common, often causing macrocytosis. B12 deficiency is rare. Iron deficiency due to
malabsorption of iron and increased loss of desquamated cells is common).
40.All of the following are associated with rapid progression of chronic hepatitis C
to cirrhosis, except:
a. Acquiring the infection at older age
b. Female sex
c. Alcohol use
d. HIV co-infeciton
e. HBV co-infection
Ans: Female sex (akeed)
45.All of the following are initial management strategies in aptients with upper GI
bleeding, except:
a. Somatostatitn
b. Bleeding scan
c. Esophagogastroduodensoscoyp
d. Acid suppressing medicaiton
e. Gastric lavage
Answer: b
46.Wilson’s disease should be considered in all of the following medical scenarios,
except:
a. Abnormal liver enzymes and non-immune hemolytic anemia
b. Exaggerated high bilirubin level and depressed alkaline phosphtaea
c. Decreases serum ceruloplasmis
d. Elderly patient with neuropsychiatric problem
e. Fuliminat liver failure with low uric acid
Answer: D
47.All of the following medications are being used for non-alcohol steatohepatitis
(NASH), except:
a. Betaite
b. Ursodeoxycholic acid
c. ribaverin
d. Vitamin e
e. Beta carotene
Anwer: C (akeed)
48.All of the following medciations are being used in chronic hepatitis B, except:
a. Lamividine
b. Ribavirine
c. Pregyled interferon
d. Adefovel dig
e. Entovavir
Answer: B (akeed)
49.All of the following micro-organisms can cause infectious diarrhea with positive
fecal leucocytes, except:
a. Shigella
b. Yersinia
c. Giardia
d. Campylobacet
e. Salmonella
Answer: c
Answer: C.
Answer: D.
8.Bilateral hilar lymph nodes enlargement occurs commonly in all the following Except.
a- pulmonary Tuberculosis
b- chronic myeloid leukemia
c- non-Hodgkins lymphoma
d- Hodgkin lymphoma
e- sarcoidosis
10.All the following are causes of WORM autoimmune hemolytic anemia Except.
a- SLE
b- chronic lymphocytic leukemia
c- methyldopa
d- infectious mononucleosis
e- non-Hodgkins lymphoma
43. All the following may be used in treatment of idiopathic thrombocytopenic purpura
Except.
a. oral predinsolone.
b. Fresh frozen plasma
c. splenectomy
d.I.V. immunioglobulin
e. immunosuppresent drug ( cyclophosphamide)
44.A-23- year old woman presents with lethargy, the following blood results are
obtained. Hb 10.4 g/dl, platelet 268x 10 9/L, WBC 6.3X 10 9/L, MCV 65 fl, Hb A2 9%
( NORMAL < 3.5% ),
Which ONE of the following is the most likely diagnosis?
a. B-Thallassemia minor
b. B-Thallassemia major
c. sickle cell anemia
d. hereditary spherocytosis
e. G6PD deficiency
49. All the following may be found in polycythemia rubra vera Except.
a. elevated WBC
b. elevated platelets
c. splenomegaly
d. elevated serum uric acid
e. high erythropoietin level
61. All the following are true about thalassemia major Except
a. Hb electrophoresis shows mainly increase in Hb A2
b. failure to thrive with short stature
c. sever anemia
d. hepatosplenomegaly
e. treatment is by blood transfusion with iron chelating agent ( desferrioxamine)
Rheumatology and bone disease
21)What is swan neck deformity in RA :
A)Hyper flextion of proximal interphalangal (PIP) and hyper extension of distal
interphalangal (DIP).
B)Hyper extension of PIP and hyper flextion of DIP.
C)Hyper extension of PIP and hyper extension of DIP.
D)Sublaxation of Metacarpophalangal.
E)Non of the above.
25)A disease modifying anti rheumatic drugs (DMARD) include all f the following
except for :
A)Salazo pyrine.
B)Hydroxychloro quine.
C)Colchicine.
D)Methotrexate
E)leflenamide
26)All of the following are criteria for Behcet disease except for :
A)Mouth ulcer's.
B)Arterial Anuyresm .ِ
C)Hypopyron..
D)Pethergy test.
E)Acne-like lesion
29)All of the following are indications for the treatment of Gouty arthritis except for:
A)Chronic Gouty arthritis.
B)Renal stones.
C)Renal failure.
D)Serum uric acid more than 8mg in men.
E)All of the above.
19. A healty patient who is HLA-B27 is most likely to develop ONE of the following.
a- psoratic arthritis
b- enteropathic spondylitis
c- gonococcal arthritis
d- Reiters disease
e- ankylosing spondylitis
53. A-25- year old man presents with urethritis, painful swollen left knee and
conjunctivitis.
ONE of the following is most likely diagnosis.
a- SLE
b- Gonococcal arthritis
c- Gout
d- Reiter's syndrome
e- Ankylosising spondylitis
63. All the following are true about uric acid metabolism except.
a- 2/3 of body uric acid pool is dietary in origin
b- 2/3 is from endogenous purine metabolism
c- 2/3 of uric acid is excreted by the kidney
d- serum uric acid is increased in polycythemia rubra vera
f- serum uric acid is increased in eclampsia of pregnancy.
64- All the following are poor prognostic signs in scleroderma except.
a- old age of onset.
b- limited skin involvement.
c- high ESR
d- renal involvement
e- pulmonary hypertension
67. A 50-year old woman has pain in her fingers on exposure to cold, arthralgia, and
difficulty in swallowing solid food.
The most useful One test to make a definitive diagnosis is
a- rheumatoid factor
b- anti-nuclear antibody
c- ECG
d- Blood urea and serum creatinin
e- anti-mitochondrial antibody
92. ONE of the following is not a disease –modifing anti-rheumatoid arthritis drug.
a- sulfasalazine
b- NSAIDs
c- methotrexate
d- leflunamide
e- sodium aurothiomalate (Gold)
98. All the following are true regarding the pathogenesis of lupus erythematosis except.
a- the exact cause is unknown.
b- It is a chronic inflammatory disease.
c- the basic pathological unit is vasculitis
d- it is due to type I hypersensitivity reaction.
e- genetic and environmental factors may play a role in the disease.
100. Repeated multiple mouth ulcers are seen in the following conditions EXCEPT:
a- Behcet's disease.
b- Systemic lupus erythematosus.
c- Herpes simplex virus infection.
d- Ankylosing spondylitis.
e- Mental stress.
26.A 70-year-old man with 3 months headache, stiffness, low grade fever, jaw
claudication, and amuresiss fugax, his ESR is 100, all of the following
statements are true regarding this condition, except:
a. Increased risk of permenant blindness
b. Tongue claudicaiton
c. Weight loss
d. Drug therapy can only be started after tissue biospy
e. Dramatic response to steroids
Which of the following is the most appropriate initial treatment for the Metabolic
Acidosis :
2 ) All the following affect short term survival in kidney transplant except :
3 ) 25 year old man was found to have microscopic hematuria by chance . Urine
analysis showed many RBC /HPF, no RBC casts , no Dysmorphic RBC
IVP showed Medullary Spomge Kidney , but no stones
The most appropriate counseling for this patient includes which of the following :
Advice him that this disorder is likely to progress to CRF over 10-20 years
Advice him that this is a benign finding , there may be a risk for nephrolithiasis
but it never progresses to Renal failure
Advice him to have his children undergo genetic testing and get treatment early
Advice him that ACE inhibitor can modify course of disease
Advice him that he should be on daily Trimethoprim –Sulfa forever
5 ) 35 year old female is evaluated because of an elevated Bp 160/105 for the past 2-3
months . Her mother has hypertension and kidney disease , and a maternal aunt is now
on hemodialysis
Labs : Cr 0.8 mg/dl , Na 140 meq/ L , K 5.0 meq /L , Cl 102 meq / L , HCO3 25 MEQ /
l , Urine Analysis is negative .
Which of the following is most likely to provide information regarding cause of her
hypertension .
6 ) All the following are Renal changes expected in normal pregnancy except :
7 ) 50 year old man has history of recurrent kidney stones which were Ca containing
stones .
All of the following are risk factors for formation of these stones except :
Hypercalciuria
Hyperoxaluria
Low urine volume
Hypercitrauria
Hyperuricosuria
8 ) All the following can be clinical & lab manifestations of hpokalemia except :
Nephrogenic DI
Tubular vacuolization
Rhabdomyolysis
Decreased Amoniagenesis
Tetany
10 ) All the following diuretic site of action combinations are true except :
30 year old male has IDDM for the past 15 years , now presenting with lower limb
edema .
Cr 2.0 mg/dl , urea 70 mg/dl . 24 hour urine collection 4.0 gm/24 hrs
All the following have a role in the progression of his renal disease except :
All the following are true about Focal Segmental Sclerosis ( FSGS) except :
25 year old female presented to OPD with Bp 160/100 , she stated that her Bp was the
same over the past 2 weeks .
All the following are first line investigations for this patient except :
Urine analysis
Serum K, Urea ,Cr
Lipid profile
Fasting blood sugar
MRA for renal arteries
All the following can be in the differential diagnosis of this case except :
Bulimia
Barter syndrome
Hypercalcemia
Primary Hyperaldosteronism
Diuretic abuse
20 year old male came to OPD with c/o of passing red urine , which was preceded by
URTI the previous morning .
Upon exam Bp 170/95 otherwise Negative :
Acetazolamide treatment
Fanconi syndrome
Treatment with Thiazide
Primary hyper parathyroid
Diarrhea
6 th year
70 year old male presented to OPD with Bp 180/80 , he had similar readings over the
last month otherwise asymptomatic .
Observation
Start on Enalapril 5 mg Q day
Start on Nifidipine 20 mg + Thiazide 25 mg
Start on Furosemide 40 mg Q day
Start on α Methyl Dopa 250 mg 1 x 3
50 year old patient previously healthy presented with hemoptysis & hematurea
Labs : Cr 3.0 mg/dl , Urea 70 mg/dl , PO4 5 mg/dl , Ca 9.2 mg/dl . Hb 13 .
Kidney biopsy showed 55 % crescents with linear deposits on basement membrane by
IF
Prednisone 20 mg Q day
Azathioprine 50 mg Q day + Cyclosporin 1mg/kg/day
Plasmapheresis + Methylprednisone IV + Cyclophosmide PO
Prednisone 30 mg Q day alternating with Chlorambucil
Tacrolimus 1 mg 1x2
40 year old male know to have Nephrotic syndrome for 15 years , now presenting with
Cr 8.0 mg/dl , Urea 100 mg/dl
All the following are indications to start this patient on dialysis except :
Peripheral Neuropathy
Anemia
Pericarditis
Low albumin
Bleeding tendency
60 year old male known to have Diabetes for 10 years and is on Enalapril 10 mg 1x2 ,
presented to ER because all his peripheral extremities became paralised
Labs K 8.0 meq /L , Cr 1.0 mg/dl
Which of the following should be used first in the management of this patient :
NAHCO3 Iv
Ventolin nebuliser
Glucose + insulin Iv
K exalate ( Na polysterene Sulfonate )
Ca gluconate Iv
25 year old male presented c/o of polyurea and weakness , Bp 110/70 , Labs
All the following could be part of the dfferential of the above case except :
Barter Syndrome
Furosemide abuse
Hypercalcemia
Excessive Licorice ingestion
Gittleman syndrome
Resident
All the following are true for a Diabetic with ESRD except
A 40 year old female known to have Membranous GN came to OPD with 24 hour
protein 4 gm/24 hours , Cr 1.0 mg/dl , Urea 40 mg/dl . Her Bp 160/100 , she was started
on Enalapril 20 mg 1x1 .
140/90
130/85
120/75
130/80
135/85
20 year old male has a LRD kidney transplant 2 years ago , he is not known to be
Diabetic nor Hypertensive . His medications are Tacrolimus 3 mg 1x2 , prednisone 5mg
1x2 , MMF 1 gm 1x2
Labs : FBS 400 mg/dl , Cr 1.0 mg/dl , Urea 35 mg/dl , Tacrolimus level 12 .
The next step in managing his Diabetes other than start him on treatment and re
checking his sugar level is :
Stop Prednisone
Decrease MMF to 500 mg 1x2
Stop Tacrolimus
Stop MMF , and increase Tacrolimus
Decrease Tacrolimus to 2 mg 1x2
Each of the glomerular lesions listed below can cause Nephrotic syndrome . Which of
them may be found in all the following conditions : non – Hodgkins lymphoma ,
hepatitis B, hepatitis C , and infective endocarditis ?
83 year old male who has DM , CHF,CRI is admitted to hospital with volume overload
& Cr 4.0 mg/dl ( baseline 2.3 mg/dl ) . He was treated by Iv diuretics , post voiding
residual was 250 ml after foleys catheter was inserted . He was discharged 2 days later
with Cr 3.0 mg/dl .
One week lter he came to OPD , Cr is 3.5 mg/dl , ultrasound shows mild bilateral
hydronephrosis .
Which of the following would best predict the effect of the patient bladder outlet
problem on kidney function :
Serum PSA
Serum Cr after several days with foleys catheter
Kidney size on U/S
Retrograde urography
Renal Scan
49 year old female is evaluated in ER after being found lying in the street in a
semiconscious state , she is known to have hypertension and a history of seizures.
Lab : BUN 79 mg/dl , Cr 8.7 mg/dl , Na 138 meq/l , K 4.2 meq/l , Cl 60 meq/l , HO3 54
meq/l . ABG PH 7.43 , PCO2 85 mmHg
Which of the following Acid Base disorder is most compatible with these lab findings
Which of the following is the most appropriate initial treatment for the Metabolic
Acidosis :
2 ) All the following affect short term survival in kidney transplant except :
3 ) 25 year old man was found to have microscopic hematuria by chance . Urine
analysis showed many RBC /HPF, no RBC casts , no Dysmorphic RBC
The most appropriate counseling for this patient includes which of the following :
Advice him that this disorder is likely to progress to CRF over 10-20 years
Advice him that this is a benign finding , there may be a risk for nephrolithiasis
but it never progresses to Renal failure
Advice him to have his children undergo genetic testing and get treatment early
Advice him that ACE inhibitor can modify course of disease
Advice him that he should be on daily Trimethoprim –Sulfa forever
5 ) 35 year old female is evaluated because of an elevated Bp 160/105 for the past 2-3
months . Her mother has hypertension and kidney disease , and a maternal aunt is now
on hemodialysis
Labs : Cr 0.8 mg/dl , Na 140 meq/ L , K 5.0 meq /L , Cl 102 meq / L , HCO3 25 MEQ /
l , Urine Analysis is negative .
Which of the following is most likely to provide information regarding cause of her
hypertension .
6 ) All the following are Renal changes expected in normal pregnancy except :
7 ) 50 year old man has history of recurrent kidney stones which were Ca containing
stones .
All of the following are risk factors for formation of these stones except :
Hypercalciuria
Hyperoxaluria
Low urine volume
Hypercitrauria
Hyperuricosuria
8 ) All the following can be clinical & lab manifestations of hpokalemia except :
Nephrogenic DI
Tubular vacuolization
Rhabdomyolysis
Decreased Amoniagenesis
Tetany
10 ) All the following diuretic site of action combinations are true except :
a1-A 64-year-old female is brought to A&E by her family, who are concerned about her
increasing confusion over the past 2 days. On examination she is found to be pyrexial at
38ºC. Blood tests reveal:dssdfsselleds
Hb 9.6 g/dl
Platelets 65 * 109/l
WCC 11.1 * 109/l
Next question
• abnormally large and sticky multimers of von Willebrand's factor cause platelets
to clump within vesselshe
• in TTP there is a deficiency of caspase which breakdowns large multimers of von
Willebrand's factorhe
• overlaps with haemolytic uraemic syndrome (HUS)he
_____________________________________________________________________
2-A 25-year-old man has a renal biopsy due to worsening renal function. This reveals
linear IgG deposits along the basement membrane. What is the most likely diagnosis?ia
Goodpasture's syndrome
sqweqwesf erwrewfsdfs adasd dhe
Goodpasture's syndrome is rare condition associated with both pulmonary haemorrhage
and rapidly progressive glomerulonephritis. It is caused by anti-glomerular basement
membrane (anti-GBM) antibodies against type IV collagen. Goodpasture's syndrome is
more common in men (sex ratio 2:1) and has a bimodal age distribution (peaks in 20-30
and 60-70 age bracket). It is associated with HLA DR2
he earaer aeraer asdsadas eerw dssdfsselleds
Features
• pulmonary haemorrhage he
• followed by rapidly progressive glomerulonephritishe
• young maleshe
• smokinghe
• lower respiratory tract infectionhe
• pulmonary oedemahe
• inhalation of hydrocarbonshe
• plasma exchangehe
• steroidshe
• cyclophosphamidehe
_____________________________________________________________________
3-Which one of the following is the most common cause of nephrotic syndrome in
children?ia
Glomerulonephritides
sqweqwesf erwrewfsdfs adasd dhe
Knowing a few key facts is the best way to approach the difficult subject of
glomerulonephritis:
he earaer aeraer asdsadas eerw dssdfsselleds
Membranous glomerulonephritis
_____________________________________________________________________
4-A 5-year-old boy is seen in A&E due to lethargy and pallor. There is no recent history
of diarrhoea. The following results are obtained:
he earaer aeraer asdsadas eerw dssdfsselleds
Hb 8.4 g/dl
Platelets 30 * 109/l
Urea 24 mmol/l
A.A IV cyclophosphamideia
B.A Ciprofloxacinia
_____________________________________________________________________5-
Which of the following types of renal tubular acidosis is associated with
hyperkalaemia?ia
• causes hyperkalaemiahe
• causes include hypoaldosteronism, diabeteshe
_____________________________________________________________________6-
A 54-year-old woman with a history membranous glomerulonephritis secondary to
systemic lupus erythematous is admitted to hospital. Her previous stable renal function
has deteriorated rapidly. The following blood tests were obtained:
he earaer aeraer asdsadas eerw dssdfsselleds
K+ 5.8 mmol/l
Urea 44 mmol/l
Albumin 17 g/l
_____________________________________________________________________7-
Which one of the following types of glomerulonephritis is most characteristically
associated with partial lipodystrophy?ia
Glomerulonephritides
sqweqwesf erwrewfsdfs adasd dhe
Knowing a few key facts is the best way to approach the difficult subject of
glomerulonephritis:
he earaer aeraer asdsadas eerw dssdfsselleds
Membranous glomerulonephritis
_____________________________________________________________________8-
What is the most common site for extra-renal cysts in a patient with autosomal
dominant polycystic kidney disease (ADPKD)?ia
A.A Pancreasia
B.A Brainia
C.A Liveria
D.A Spleenia
E.A Thyroidia
Liver cysts are present in 70% of patients with ADPKD. Around 8% of patients have
berry aneurysms
ADPKD: features
sqweqwesf erwrewfsdfs adasd dhe
Features
• hypertensionhe
• recurrent UTIshe
• abdominal painhe
• renal stoneshe
• haematuriahe
• CRFhe
_____________________________________________________________________9-
A 65-year-old female with a 20 year history of rheumatoid arthritis is referred to the
acute medical unit with bilateral leg oedema. The following results are obtained:
aeraer asdsadas eerw dssdfsselleds
Albumin 26 g/l
Bilirubin 12 mmol/l
This rather odd question fooled most candidates when it appeared. The chronic
inflammatory process (rheumatoid) predisposes to amyloidosis which in turn can cause
nephrotic syndrome. Rectal biopsy is an (infrequent) test done to look for amyloidosis.
he earaer aeraer asdsadas eerw dssdfsselleds
Rheumatoid drugs such as gold may cause nephrotic syndrome but none of the other
options point to this as an answer
Amyloidosis: types
sqweqwesf erwrewfsdfs adasd dhe
AL amyloid
_____________________________________________________________________1
0-Which one of the following types of glomerulonephritis is most characteristically
associated with Goodpasture's syndrome?ia
Glomerulonephritides
sqweqwesf erwrewfsdfs adasd dhe
Knowing a few key facts is the best way to approach the difficult subject of
glomerulonephritis:
he earaer aeraer asdsadas eerw dssdfsselleds
Membranous glomerulonephritis
A.A Malariaia
B.A Lymphomaia
D.A Cryoglobulinaemiaia
E.A Goldia
Membranous glomerulonephritis
sqweqwesf erwrewfsdfs adasd dhe
Membranous glomerulonephritis is the commonest type of glomerulonephritis in adults
and is the third most common cause of end-stage renal failure (ESRF). It usually
presents as nephrotic syndrome or proteinuria
he earaer aeraer asdsadas eerw dssdfsselleds
Renal biopsy demonstrates:
• idiopathiche
• infections: hepatitis B, malariahe
• malignancy: lung cancer, lymphoma, leukaemiahe
• drugs: gold, penicillamine, NSAIDshe
• systemic lupus erythematous (class V disease)he
A.A Urticariaia
B.A Hypertensionia
Erythropoietin
sqweqwesf erwrewfsdfs adasd dhe
Erythropoietin is a haematopoietic growth factor that stimulates the production of
erythrocytes. The main uses of erythropoietin are to treat the anaemia associated with
chronic renal failure and that associated with cytotoxic therapy
he earaer aeraer asdsadas eerw dssdfsselleds
Side-effects of erythropoietin
• iron deficiencyhe
• inadequate dosehe
• concurrent infection/inflammationhe
• hyperparathyroid bone diseasehe
• aluminium toxicityhe
_____________________________________________________________________1
3-A 10-year-old boy is taken to see the GP by his mother. For the past two days he has
had a sore throat associated with blood in his urine. There is no significant past medical
history. The GP suspects glomerulonephritis and refers the patient to hospital. What
would a renal biopsy most likely show?ia
B.A No changeia
This boy is likely to have IgA nephropathy. Histological features include mesangial
hypercellularity and positive immunofluorescence for IgA & C3
IgA nephropathy
sqweqwesf erwrewfsdfs adasd dhe
Basics
• alcoholic cirrhosishe
• coeliac disease/dermatitis herpetiformishe
14-A 73-year-old with a history of alcohol excess is admitted following a fall at home.
On admission the following blood results are obtained:
he earaer aeraer asdsadas eerw dssdfsselleds
Creatinine 248
µmol/l
4 Year:
19.30 year old male has IDDM for the past 15 years , now presenting
with lower limb edema .
Cr 2.0 mg/dl , urea 70 mg/dl . 24 hour urine collection 4.0 gm/24 hrs
All the following have a role in the progression of his renal disease except
:
20.All the following are true about Focal Segmental Sclerosis ( FSGS)
except :
21.25 year old female presented to OPD with Bp 160/100 , she stated
that her Bp was the same over the past 2 weeks .
All the following are first line investigations for this patient except :
f) Urine analysis
g) Serum K, Urea ,Cr
h) Lipid profile
i) Fasting blood sugar
j) MRA for renal arteries
f) Bulimia
g) Barter syndrome
h) Hypercalcemia
i) Primary Hyperaldosteronism
j) Diuretic abuse
23.20 year old male came to OPD with c/o of passing red urine , which
was preceded by URTI the previous morning .
Upon exam Bp 170/95 otherwise Negative :
24.25 year old female was admitted to hospital with referred to OPD
due to incidental finding of the following labs & ABG :
PH 7.32 , HCO3 15
Cr 1.0 mg/dl , urea 35 meq/l , Na 135 meq /L , Cl 110 meq/l
All the following may cause the above except :
f) Acetazolamide treatment
g) Fanconi syndrome
h) Treatment with Thiazide
i) Primary hyper parathyroid
j) Diarrhea
6 th year:
25.70 year old male presented to OPD with Bp 180/80 , he had similar
readings over the last month otherwise asymptomatic . The best
management for this patient is :
f) Observation
g) Start on Enalapril 5 mg Q day
h) Start on Nifidipine 20 mg + Thiazide 25 mg
i) Start on Furosemide 40 mg Q day
j) Start on α Methyl Dopa 250 mg 1 x 3
26.50 year old patient previously healthy presented with hemoptysis &
hematurea
Labs : Cr 3.0 mg/dl , Urea 70 mg/dl , PO4 5 mg/dl , Ca 9.2 mg/dl . Hb 13 .
Kidney biopsy showed 55 % crescents with linear deposits on basement
membrane by IF . The best management is :
a) Prednisone 20 mg Q day
b) Azathioprine 50 mg Q day + Cyclosporin 1mg/kg/day
c) Plasmapheresis + Methylprednisone IV + Cyclophosmide PO
d) Prednisone 30 mg Q day alternating with Chlorambucil
e) Tacrolimus 1 mg 1x2
27.40 year old male know to have Nephrotic syndrome for 15 years ,
now presenting with Cr 8.0 mg/dl , Urea 100 mg/dl , All the
following are indications to start this patient on dialysis except :
a) Peripheral Neuropathy
b) Anemia
c) Pericarditis
d) Low albumin
e) Bleeding tendency
29.60 year old male known to have Diabetes for 10 years and is on
Enalapril 10 mg 1x2 , presented to ER because all his peripheral
extremities became paralised , Labs K 8.0 meq /L , Cr 1.0 mg/dl.
Which of the following should be used first in the management of
this patient :
a) NAHCO3 Iv
b) Ventolin nebuliser
c) Glucose + insulin Iv
d) K exalate ( Na polysterene Sulfonate )
e) Ca gluconate Iv
a) Barter Syndrome
b) Furosemide abuse
c) Hypercalcemia
d) Excessive Licorice ingestion
e) Gittleman syndrome
Resident:
31.All the following are true for a Diabetic with ESRD except
a) 140/90
b) 130/85
c) 120/75
d) 130/80
e) 135/85
33.20 year old male has a LRD kidney transplant 2 years ago , he is not
known to be Diabetic nor Hypertensive . His medications are
Tacrolimus 3 mg 1x2 , prednisone 5mg 1x2 , MMF 1 gm 1x2, Labs
: FBS 400 mg/dl , Cr 1.0 mg/dl , Urea 35 mg/dl , Tacrolimus level 12
. The next step in managing his Diabetes other than start him on
treatment and re checking his sugar level is :
a) Stop Prednisone
b) Decrease MMF to 500 mg 1x2
c) Stop Tacrolimus
d) Stop MMF , and increase Tacrolimus
e) Decrease Tacrolimus to 2 mg 1x2
a) Serum PSA
b) Serum Cr after several days with foleys catheter
c) Kidney size on U/S
d) Retrograde urography
e) Renal Scan
36.49 year old female is evaluated in ER after being found lying in the
street in a semiconscious state , she is known to have hypertension
and a history of seizures. Lab : BUN 79 mg/dl , Cr 8.7 mg/dl , Na
138 meq/l , K 4.2 meq/l , Cl 60 meq/l , HO3 54 meq/l . ABG PH 7.43
, PCO2 85 mmHg. Which of the following Acid Base disorder is
most compatible with these lab findings
1. G. Woodrow and
2. J. H. Turney
+Author Affiliations
1. Renal Unit, The General Infirmary at Leeds Great
George Street, Leeds, LSI 3EX, UK
1. Correspondence and offprint requests to: J. H. Turney, Renal Unit, The
General Infirmary at Leeds, Great George Street, Leeds, LSI 3EX, UK.
• Received May 1, 1991.
• Accepted August 12, 1991.
Abstract
The cause of 636 deaths during acute renal failure (ARF) occurring
between 1956 and 1989 were analysed. Deaths due to haemorrhage and to
nonrecovery of renal function have declined but cardiovascular deaths
and withdrawal of active treatment have increased. The causes of death
varied with the clinical situation in which ARF arose. The most important
factor contributing to death was the underlying cause of ARF. 67% deaths
due to sepsis resulted from infection present at the time of development
of ARF. Deaths due to secondary complications have declined, indicating
that the precipitating causes of ARF are the main determinant of overall
mortality
Key words
• cause of death
13. Major criteria for Rheumatic fever include all the following Except.
u- carditis
v- Sydenham's chorea
w- Polyarthralgia
x- Erythema marginatum
y- Subcutaneous nodules
14. ONE of the following drugs is LEAST used in treatment of acute sever
asthma.
u- nebulized B2 agonist
v- i.v hydrocortisone
w- epinephrine (adrenaline)
x- oxygen
y- i.v . aminophylline
15. Hypoxia (decreased PaO2) and decreased Pa CO2 is found in all the
following Except.
u- left ventricular failure
v- massive pulmonary embolism
w- acute sever asthma
x- acute exacerbation of COPD
y- pneumonia
18. ONE of the following is the mode of action for B-Blockers in controlling
hypertension.
k- decrease cardiac out put.
l- Slow the heart rate
m- Increase cardiac force of contraction
n- Increase cardiac output
o- Decrease plasma volume
19. A healty patient who is HLA-B27 is most likely to develop ONE of the
following.
a- psoratic arthritis
b- enteropathic spondylitis
c- gonococcal arthritis
d- Reiters disease
e- ankylosing spondylitis
21. All the following are criteria to define sever attack of ulcerative colitis
Except.
a- stool frequency > 10 per day with out blood
b- fever > 37.5 C
c- tachycardia >90/min
d- anemia hemoglobin < 10 gram/dl
e- albumin < 30 g/L
26 ONE of the following tests is most suitable in screening patients for celiac
disease.
a- Anti-casein antibodies
b- Anti-endomyseal antibodies
c- Anti-gliadin antibodies
d- ESR
e- Aplha feto protein.
6
31. A 72-year-old woman comes to you to control her high blood pressure
(180/100) mmHg.
What is the ONE target blood pressure in the long term for this patient?
a- <160/90
j- <150/90
k- <145/90
l- <130/85
m- <120/70
32. All the following are true about side effects of anti-diabetic agents Except.
a- metformin carries a risk of lactic acidosis.
b- sulphonylurea is used safely pregnancy
c- glitazones may cause prominent fluid retention
d- insulin may cause lipohypertrophy
e- acarbose causes diarrhea
Dubin-Johnson syndrome
dacbin joncs[n
7
37.ONE of the following statements is true about treatment of pulmonary
tuberculosis.
a- pyrazinamide may precipitate hyperurecmic gout.
b- INH can cause optic neuritis
c- renal impairment with rifampicine
d- streptomycin is causing reversible damage to vestibular nerve
e- hepatitis is usually caused by ehambutol
First-line
Antimycobacterial ROA Adverse Effects
drugs
I. Isoniazid INH O 1. Peripheral Neuritis –treated by➔ Daily
Pyridoxine (Vitamin B6)
2. Liver Damage➔ Hepatitis
&Hepatotoxicity
3. Optic Neuritis
4. Psychosis
5. Hypersensitivity reactions
# Effect on P-450 ➔ Inhibitor
II. Rifampicin O& Orange-red color of secretions
(Rifampin) PE
1. Nausea & Vomiting
[Should be limited 2. Rash
only when it is very 3. Fever
essential to? 4. Liver Damage➔ Transient disturbance
Avoid Emergence of liver function & Hepatotoxicity
of bacterial 5. Enzyme induction in liver (P-450
Inducer) ➔ decrease effects of drugs
resistance]
metabolized in liver e.g. Warfarin &
OCP
III. Pyrazinamide O 1. Liver Damage➔ May produce liver
toxicity\ hepatitis
2. Hyperuricemia
3. Allergy
4. Sideroblastic anemia
IV. Ethambutol O 1. Optic Neuritis
2. PTs with Renal impairment➔
A. Loss of visual acuity
B. Red-green color blindness
C. Visual field defects
V. Streptomycin IM 1. Ototoxicity
2. Nephrotoxicity
38. All the following are found in left sided heart failure Except.
a- bilateral basal creptations
b- third heart sound
c- pulsus alternans
d- raised JVP
e- pulmonary oedema
44.A-23- year old woman presents with lethargy, the following blood results
are obtained. Hb 10.4 g/dl, platelet 268x 10 9/L, WBC 6.3X 10 9/L, MCV 65
fl, Hb A2 9% ( NORMAL < 3.5% ),
Which ONE of the following is the most likely diagnosis?
a. B-Thallassemia minor
b. B-Thallassemia major
c. sickle cell anemia
d. hereditary spherocytosis
e. G6PD deficiency
45.A 70-year-old woman is referred to hospital due to evidences of congestive
heart failure. Blood test reveal the following: Hb 7.4 g/dl, MCV 124 fl,
platelets 98 x10 9/l, WBC 3X10 9/L,
All the following investigations are required to reach a diagnosis Except.
a. Schilling test
b. Intrensic factor antibodies
c. antiparitel cell antibodies
d. bone marrow aspiration, looking for megaloblasts
e. C-reactive protein.
48. All the following are true about renal osteodystrophy Except.
a. reduced conversion of 25 (OH)2 D3 to 1-25-(OH) 2 D3
b. increased parathyroid hormone
c.increased intestinal calcium absoprption
d. decreased osteoclastic activity
e. increased reabsorption of calcium from bone.
49. All the following may be found in polycythemia rubra vera Except.
a. elevated WBC
b. elevated platelets
c. splenomegaly
d. elevated serum uric acid
e. high erythropoietin level
51. 20-year old woman presents with a week history of fever, rigor and
productive rusty cough. The chest X-ray shows left lower lobe consolidation.
Which ONE of the following is most appropriate treatment?
p- clarithramycin
q- gentamycin
r- Cotrimoxazole
s- Benzypenicillin
t- Flucloxacillin
52. ONE of the following is most likely diagnosis for patient with thyroid
function test showing elevated serum T4 and low radioactive iodine uptake.
a- Grave's disease.
b- Hashimoto's thyroiditis.
c- subacute thyroiditis.
d- non-toxic goiter.
e- pregnancy
53. A-25- year old man presents with urethritis, painful swollen left knee and
conjunctivitis.
ONE of the following is most likely diagnosis.
g- SLE
h- Gonococcal arthritis
i- Gout
j- Reiter's syndrome
k- Ankylosising spondylitis
10
57. A 54- year- old male with Child's grade C hepatic encephalopathy presents
with haemetemesis. Which ONE of the following is most appropriate
immediate therapy?
a. i.v desmopressin
b. i.v isosorbide dinitrate
c. i.v. omperazole
d. i.v. propranolol
e. i.v. somatostatin.
58. All the following are risk factors for development of peptic ulcer disease
Except.
a. daily use of NSAID
b. gastric infection with H.pylori
c. sever emotional stress.
d. cigarette smoking
e. gastrin-secreting tumors.
11
60. A29- year-old man presents with symptoms of gastroesophageal reflux.
Which ONE of the following is most useful in assessing the role of surgery.
a. cardiac sphincter manometry.
b. gastric emptying study.
c. intragastric PH monotring off therapy .
d. oesophgeal motility study.
e. oesophgeal PH monotring on therapy
61. All the following are true about thalassemia major Except
a. Hb electrophoresis shows mainly increase in Hb A2
b. failure to thrive with short stature
c. sever anemia
d. hepatosplenomegaly
e. treatment is by blood transfusion with iron chelating agent (
desferrioxamine)
63. All the following are true about uric acid metabolism except.
a- 2/3 of body uric acid pool is dietary in origin
b- 2/3 is from endogenous purine metabolism
c- 2/3 of uric acid is excreted by the kidney
d- serum uric acid is increased in polycythemia rubra vera
e- serum uric acid is increased in eclampsia of pregnancy.
64- All the following are poor prognostic signs in scleroderma except.
a- old age of onset.
b- limited skin involvement.
c- high ESR
d- renal involvement
e- pulmonary hypertension
67. A 50-year old woman has pain in her fingers on exposure to cold,
arthralgia, and difficulty in swallowing solid food.
The most useful One test to make a definitive diagnosis is
a- rheumatoid factor
b- anti-nuclear antibody
c- ECG
d- Blood urea and serum creatinin
e- anti-mitochondrial antibody
13
14
77. Modifiable risk factors for ischemic heart disease include all the following
Except.
a- smoking
b- hypertension
c- hyperlipidaemia
d- age
e- diabetes mellitus
78. All the following antibiotics may be used in treatment of H.pylori Except.
a- amoxicillin
b- tetracycline
c- metronodazo;e
d- clarithramycin
e- strepotomycin
79. All the following are found in chronic renal failure Except.
a- hyperkalemia
b- hyperurecemia
c- hypophosphatemia
d- hypocalcemia
e- Low serum erythropitein
80. Treatment of hyperkalemia include all the following Except.
a- i.v calcium gluconate
b- i.v salbutamol
c- i.v soluble insulin and glucouse
d- i.v hydrocortisone
e- hemodialysis
81. A 57-year-old man develops acute shortness of breath shortly after a 20-
hour automobile ride. He has normal physical examination except for
tachycardia,ECG: shows sinus tachycardia, but is otherwise normal.
Which ONE of the following is correct?
k- the patient should admitted to hospital and if there is no
contraindication to anticoagulant, Heparin should be started while
waiting for tests.
l- Normal finding on examination of the lower limbs are extremely
unusual
m- A definitive diagnosis can be made by history alone
n- Early treatment has little effect on overall mortality
o- The disease can be diagnosed definitely by Chest X-Ray
82. Which ONE of the following Arterial Blood Gases is most likely to be
found in a 60-year-old heavy smoker man, He has chronic bronchitis,
peripheral odema and cyanosis?
a- PH 7.50, PO2 75, PCO2 28
b- PH 7.15, PO2 78, PCO2 92
c- PH 7.06, PO2 36, PCO2 95
d- PH 7.06, PO2 108, PCO2 13
e- PH 7.39, PO2 48, PCO2 54
15
83. A 60-year-old man has an inferior myocardial infarction; his heart rate is
45 /min.
The artery most likely to be involved in this process is:
k- right coronary artery
l- left main artery
m- left anterior descending artery
n- circumflex artery
o- left mammary artery
84. A patient with stable angina on asprine, nitrate and B-Blocker, developed
3 episodes of sever and long –lasting chest pain each day over the past 3 days.
His ECG and cardiac enzymes are normal.
One of the following is the best treatment
k- admit the patient and start I.V digoxine
l- admit the patient and start I.V heparine
m- admit the patient and start I.V prophylactic streptokinase
n- admit the patient and for observation without changing his medications
o- Discharge the patient with increasing the dose of B-blocker and nitrate
85. ONE of the following drugs reduces myocardial remodeling after acute
myocardial infarction.
a- ACE inhibitors
b- digoxine
c- verapamil
d- furosemide (lasix)
e- hydralazine.
16
88. ONE of the following is used in treatment of hypertensive Emergency
a- I.V atenalol (tenormin)
b- oral captopril
c- sublingual nifedipine
d- continous infusion of sodium nitroprusside
e- oral alpha methyl dopa
93. All the following are early complications of acute myocardial infarction
Except.
a- cardiogenic shock
b- heart block
c- ventricular fibrillation
d- aneurismal dilatation of infracted area
e- sudden cardiac death
17
94. ECG shows ST elevation in leads II, III, AVF, indicate infarction in ONE
of the following
a- anteroseptal MI
b- anterolateral MI
c- posterior MI
d- inferior MI
e- subendocardial MI
95. All the following ECG findings are found in hypokalemia Except.
k) Flattened T waves
l) U waves
m) Shortened QT interval
n) ST segment depression
o) Ectopic beats
96. ONE of the following is LEAST common cause of Microscopical
hematuria
a-Minimal change disease (lipoid nephrosis)
b-Membranous glomerulonephritis
c-Proliferative glomerulonephritis
d-Membranoproliferative glomerulonephritis
e-Lupus nephritis
98. All the following are true regarding the pathogenesis of lupus
erythematosis except.
a- the exact cause is unknown.
b- It is a chronic inflammatory disease.
c- the basic pathological unit is vasculitis
d- it is due to type I hypersensitivity reaction.
e- genetic and environmental factors may play a role in the disease.
100. Repeated multiple mouth ulcers are seen in the following conditions
EXCEPT:
a- Behcet's disease.
b- Systemic lupus erythematosus.
c- Herpes simplex virus infection.
d- Ankylosing spondylitis.
e- Mental stress.
he earaer aeraer asdsadas eerw dssdfsselleds
Which one of the following would have been most likely to prevent the deterioration in
renal function?ia
D.A Frusemideia
E.A Mannitolia
Intravenous fluids are the most important management step in the prevent of
rhabdomyolysis in such patients
• seizurehe
• collapse/coma (e.g. elderly patients collapses at home, found 8 hours later)he
• ecstasyhe
• crush injuryhe
• McArdle's syndromehe
• drugs: statinshe
_____________________________________________________________________1
5-Which one of the following drugs may be safely continued at the same dose in renal
failure?ia
A.A Tetracyclineia
B.A Diclofenacia
C.A Warfarinia
D.A Nitrofurantoinia
E.A Lithiumia
_____________________________________________________________________1
6-Each of the following is a risk factor for renal stone formation, except:ia
B.A Cadmiumia
C.A Hyperparathyroidismia
D.A Dehydrationia
E.A Cystinosisia
• dehydrationhe
• hypercalciuria, hyperparathyroidism, hypercalcaemiahe
• cystinuriahe
• high dietary oxalatehe
• renal tubular acidosishe
• medullary sponge kidney, polycystic kidney diseasehe
• beryllium or cadmium exposurehe
• gouthe
• ileostomy: loss of bicarbonate and fluid results in acidic urine, causing the
precipitation of uric acidhe
• post-streptococcal glomerulonephritishe
• subacute bacterial endocarditishe
• systemic lupus erythematoushe
• mesangiocapillary glomerulonephritishe
_____________________________________________________________________1
8-Autosomal dominant polycystic kidney disease type 1 is associated with a gene defect
in:ia
ADPKD
sqweqwesf erwrewfsdfs adasd dhe
Autosomal dominant polycystic kidney disease (ADPKD) is the most common inherited
cause of kidney disease, affecting 1 in 1,000 Caucasians. Two disease loci have been
identified, PKD1 and PKD2, which code for polycystin-1 and polycystin-2 respectively
he earaer ae
Chromosome 16 Chromosome 4
_____________________________________________________________________1
9-A 27-year-old man is diagnosed with Goodpasture's syndrome. Which one of the
following does not increase the likelihood of a pulmonary haemorrhage?ia
A.A Smokingia
D.A Dehydrationia
Goodpasture's syndrome
• pulmonary haemorrhage he
• followed by rapidly progressive glomerulonephritishe
• young maleshe
• smokinghe
• lower respiratory tract infectionhe
• pulmonary oedemahe
• inhalation of hydrocarbonshe
• plasma exchangehe
• steroidshe
• cyclophosphamidehe
_____________________________________________________________________2
0-A patient with type 1 diabetes mellitus is reviewed in the nephrology outpatient clinic.
He is known to have stage 1 diabetic nephropathy. Which of the following best
describes his degree of renal involvement?ia
B.A Hyperfiltrationia
E.A Microalbuminuriaia
For the purposes of the MRCP, increase in the glomerular filtration rate (GFR) is most
characteristic of stage 1 diabetic nephropathy. It is however known that elevation of the
GFR usually persists into stage 2
Stage 1
_____________________________________________________________________
Alport's syndrome
sqweqwesf erwrewfsdfs adasd dhe
Alport's syndrome is a hereditary condition, usually X-linked dominant but may be
autosomal recessive or dominant. It is due to a defect in the gene which codes for type
IV collagen resulting in an abnormal glomerular-basement membrane (GBM). The
disease is more severe in males with females rarely developing renal failure
he earaer aeraer asdsadas eerw dssdfsselleds
A favourite question in the MRCP is an Alport's patient with a failing renal transplant.
This may be caused by the presence of anti-GBM antibodies leading to a Goodpasture's
syndrome like picture
he earaer aeraer asdsadas eerw dssdfsselleds
Alport's syndrome usually presents in childhood. The following features may be seen:
• microscopic haematuriahe
• progressive renal failurehe
• bilateral sensorineural deafnesshe
• retinitis pigmentosahe
• lenticonus: protrusion of the lens surface into the anterior chamberhe
_____________________________________________________________________2
2-Which one of the following may be useful in the prevention of calcium renal stones?ia
A.A Pyridoxineia
B.A Allopurinolia
C.A Lithiumia
• allopurinolhe
• urinary alkalinization e.g. oral bicarbonatehe
_____________________________________________________________________2
3-Which one of the following types of glomerulonephritis is associated with fusion of
podocytes on electron microscopy?ia
• nephrotic syndromehe
• hypertensionhe
• highly selective proteinuriahe
• renal biopsy: electron microscopy shows fusion of podocytes he
A.A Hydronephrosisia
B.A Osteomalaciaia
C.A Aminoaciduriaia
D.A Glycosuriaia
Fanconi syndrome
sqweqwesf erwrewfsdfs adasd dhe
A disorder of renal tubular function
he earaer aeraer asdsadas eerw dssdfsselleds
Features
Causes
_____________________________________________________________________
Amyloidosis
sqweqwesf erwrewfsdfs adasd dhe
Overview
• systemic or localizedhe
• further characterised by precursor protein (e.g. AL in myeloma - A for Amyloid,
L for immunoglobulin Light chain fragments)he
_____________________________________________________________________2
6-Each one of the following is associated with Bartter's syndrome, except:ia
B.A Hypertensionia
C.A Weaknessia
E.A Hypokalaemiaia
Bartter's syndrome is associated with normotension
Bartter's syndrome
sqweqwesf erwrewfsdfs adasd dhe
Bartter's syndrome is an inherited cause (usually autosomal recessive) of severe
hypokalaemia due to defective chloride absorption at the Na+ K+ 2Cl- cotransporter in
the ascending loop of Henle. It should be noted that is associated with normotension
(unlike other endocrine causes of hypokalaemia such as Conn's, Cushing's and Liddle's
syndrome which are associated with hypertension)
he earaer aeraer asdsadas eerw dssdfsselleds
Features
_____________________________________________________________________2
7-Which one of the following is least associated with focal segmental
glomerulosclerosis?ia
B.A Heroinia
D.A Sarcoidosisia
• idiopathiche
• secondary to other renal pathology e.g. IgA nephropathy, reflux nephropathyhe
• HIVhe
• heroinhe
• Alport's syndromehe
• sickle-cellhe
_____________________________________________________________________
28-A 45-year-old female with nephrotic syndrome develops renal vein thrombosis.
What changes in patients with nephrotic syndrome predispose to the development of
venous thromboembolism?ia
Nephrotic syndrome
sqweqwesf erwrewfsdfs adasd dhe
Triad of
1. Proteinuria (> 3g/24hr) causing
2. Hypoalbuminaemia (< 30g/L) and
3. Oedema
he earaer aeraer asdsadas eerw dssdfsselleds
Loss of antithrombin-III, proteins C and S and a associated rise in fibrinogen levels
predispose to thrombosis. Loss of TBG lowers total, but not free thyroxine levels
_____________________________________________________________________
29-Each of the following is a risk factor for renal stone formation, except:ia
A.A Cystinuriaia
B.A Berylliumia
C.A Hypoparathyroidismia
• dehydrationhe
• hypercalciuria, hyperparathyroidism, hypercalcaemiahe
• cystinuriahe
• high dietary oxalatehe
• renal tubular acidosishe
• medullary sponge kidney, polycystic kidney diseasehe
• beryllium or cadmium exposurehe
• gouthe
• ileostomy: loss of bicarbonate and fluid results in acidic urine, causing the
precipitation of uric acidhe
_____________________________________________________________________3
0-A 45-year-old woman with nephrotic syndrome is noted to have marked loss of
subcutaneous tissue from the face. What is the most likely underlying cause of her renal
disease?ia
_____________________________________________________________________3
1-Which one of the following is not a recognised risk factor for the development of
diabetic nephropathy?ia
B.A Smokingia
E.A Hypertensionia
Diabetic nephropathy
sqweqwesf erwrewfsdfs adasd dhe
Basics
• male sexhe
• poor glycaemic controlhe
• hypertension, hyperlipidaemia, smokerhe
• raised dietary proteinhe
• genetic predisposition (e.g. ACE gene polymorphisms)he
_____________________________________________________________________3
3-Which one of the following is least associated with retroperitoneal fibrosis?ia
D.A Methysergideia
E.A Sulphonamidesia
Retroperitoneal fibrosis
Lower back pain is the most common presenting feature
Associations
• Riedel's thyroiditishe
• previous radiotherapyhe
• sarcoidosishe
• inflammatory abdominal aortic aneurysmhe
• drugs: methysergidehe
34-Microalbuminuria may be defined as an albumin excretion of:ia
C.A 1 - 10 mg/dayia
E.A 3 - 30 mg/dayia
Proteinuria
sqweqwesf erwrewfsdfs adasd dhe
Microalbuminuria
_____________________________________________________________________3
5-What percentage of cardiac output does renal blood flow accounts for:ia
A.A 5%ia
B.A 10%ia
C.A 15%ia
D.A 20-25%ia
E.A 30-35%ia
Renal physiology
sqweqwesf erwrewfsdfs adasd dhe
Renal blood flow is 20-25% of cardiac output
he earaer aeraer asdsadas eerw dssdfsselleds
Renal cortical blood flow > medullary blood flow (i.e. tubular cells more prone to
ischaemia
36-Which one of the following types of glomerulonephritis is most characteristically
associated with streptococcal infection in children?ia
Glomerulonephritides
Knowing a few key facts is the best way to approach the difficult subject of
glomerulonephritis:
he earaer aeraer asdsadas eerw dssdfsselleds
Membranous glomerulonephritis
_____________________________________________________________________3
7-Which one of the following is least recognised as an indication for plasma
exchange?ia
Plasma exchange
Indications for plasma exchange
• Guillain-Barre syndromehe
• myasthenia gravishe
• Goodpasture's syndromehe
• ANCA positive vasculitis e.g. Wegener's, Churg-Strausshe
• TTP/HUShe
• cryoglobulinaemiahe
• hyperviscosity syndrome e.g. secondary to myelomahe
_____________________________________________________________________3
8-What is the most significant factor leading to the development of anaemia in patients
with chronic kidney disease?ia
A.A Reduced absorption of ironia
E.A Blood loss due to capillary fragility and poor platelet functionia
• the 2006 NICE guidelines suggest a target haemoglobin of 10.5 - 12.5 g/dlhe
• determination and optimisation of iron status should be carried out prior to the
administration of erythropoiesis-stimulating agents (ESA). Many patients,
especially those on haemodialysis, will require IV ironhe
- ESAs such as erythropoietin and darbepoietin should be used in those 'who are
likely to benefit in terms of quality of life and physical function'
_____________________________________________________________________3
9-A patient with type 1 diabetes mellitus is reviewed in the nephrology outpatient clinic.
He is known to have stage 4 diabetic nephropathy. Which of the following best
describes his degree of renal involvement?ia
A.A Microalbuminuriaia
D.A Hyperfiltrationia
Stage 1
_____________________________________________________________________4
0-Each one of the following is associated with papillary necrosis, except:ia
B.A Tuberculosisia
D.A Syphilisia
Papillary necrosis
sqweqwesf erwrewfsdfs adasd dhe
Causes
he___________________________________________________________________41
-Renal cell carcinoma is least associated with which one of the following hormones?ia
A.A Erythropoietinia
E.A Reninia
Features
_____________________________________________________________________4
2-A 6-year-old boy presents is diagnosed as having nephrotic syndrome. A presumptive
diagnosis of minimal change glomerulonephritis is made. What is the most appropriate
treatment?ia
A.A Cyclophosphamideia
B.A Supportive treatment as an inpatientia
E.A Prednisoloneia
• nephrotic syndromehe
• hypertensionhe
• highly selective proteinuriahe
• renal biopsy: electron microscopy shows fusion of podocytes he
_____________________________________________________________________4
3-What is the best way to differentiate between acute and chronic renal failure?ia
A.A 24 hr creatinineia
_____________________________________________________________________4
4-Which one of the following features is least likely to be seen in Henoch-Schonlein
purpura?ia
C.A Polyarthritisia
D.A Thrombocytopeniaia
Henoch-Schonlein purpura
Henoch-Schonlein purpura (HSP) is an IgA mediated small vessel vasculitis. There is a
degree of overlap with IgA nephropathy (Berger's disease). HSP is usually seen in
children following an infection
Features
• palpable purpuric rash (with localized oedema) over buttocks and extensor
surfaces of arms and legshe
• abdominal painhe
• polyarthritishe
• features of IgA nephropathy may occur e.g. haematuria, renal failurehe
_____________________________________________________________________4
5-Which of the following factors would suggest that a patient has established acute
tubular necrosis rather than pre-renal uraemia?ia
_____________________________________________________________________4
6-Each one of the following is seen in renal osteodystrophy, except:ia
47-A 26-year-old man with loin pain and haematuria is found to have autosomal
dominant polycystic kidney disease. A defect in which one of the following genes is
likely to be responsible?ia
Most cases of autosomal dominant polycystic kidney disease (ADPKD) are due to a
mutation in the PKD1 gene. The PKD1 gene encodes for a polycystin-1, a large cell-
surface glycoprotein of unknown function
ADPKD
sqweqwesf erwrewfsdfs adasd dhe
Autosomal dominant polycystic kidney disease (ADPKD) is the most common inherited
cause of kidney disease, affecting 1 in 1,000 Caucasians. Two disease loci have been
identified, PKD1 and PKD2, which code for polycystin-1 and polycystin-2 respectively
he earaer aeraer asdsadas eerw dssdfsselleds
Chromosome 16 Chromosome 4
_____________________________________________________________________
48-What is the most likely outcome following the diagnosis of minimal change
nephropathy in a 20-year-old male?
ia
• nephrotic syndromehe
• hypertensionhe
• highly selective proteinuriahe
• renal biopsy: electron microscopy shows fusion of podocytes he
he earaer aeraer asdsadas eerw dssdfsselleds
Management
_____________________________________________________________________4
9-Each one of the following is a cause of sterile pyuria, except:ia
E.A Appendicitisia
Sterile pyuria
sqweqwesf erwrewfsdfs adasd dhe
Causes
_____________________________________________________________________5
0-Which of the following types of renal stones are said to have a semi-opaque
appearance on x-ray?ia
51-A 61-year-old man with a history of hypertension presents with central chest pain.
Acute coronary syndrome is diagnosed and conventional management is given. A few
days later a diagnostic coronary angiogram is performed. The following week a
deteriorating of renal function is noted associated with a purpuric rash on his legs. What
is the most likely diagnosis?ia
Cholesterol embolisation
sqweqwesf erwrewfsdfs adasd dhe
Overview
• eosinophiliahe
• purpurahe
• renal failurehe
• livedo reticularishe
_____________________________________________________________________5
2-Which one of the following statements is true regarding autosomal recessive
polycystic kidney disease?ia
ARPKD
sqweqwesf erwrewfsdfs adasd dhe
Autosomal recessive polycystic kidney disease (ARPKD) is much less common than
autosomal dominant disease (ADPKD). It is due to a defect in a gene located on
chromosome 6
he earaer aeraer asdsadas eerw dssdfsselleds
D.A Lenticonusia
E.A Anosmiaia
Alport's syndrome
sqweqwesf erwrewfsdfs adasd dhe
Alport's syndrome is a hereditary condition, usually X-linked dominant but may be
autosomal recessive or dominant. It is due to a defect in the gene which codes for type
IV collagen resulting in an abnormal glomerular-basement membrane (GBM). The
disease is more severe in males with females rarely developing renal failure
he earaer aeraer asdsadas eerw dssdfsselleds
A favourite question in the MRCP is an Alport's patient with a failing renal transplant.
This may be caused by the presence of anti-GBM antibodies leading to a Goodpasture's
syndrome like picture
he earaer aeraer asdsadas eerw dssdfsselleds
Alport's syndrome usually presents in childhood. The following features may be seen:
• microscopic haematuriahe
• progressive renal failurehe
• bilateral sensorineural deafnesshe
• retinitis pigmentosahe
• lenticonus: protrusion of the lens surface into the anterior chamberhe
_____________________________________________________________________5
4-Each one of the following is a recognised complication of nephrotic syndrome,
except:ia
A.A Hyperlipidaemiaia
D.A Hypercalcaemiaia
_____________________________________________________________________5
5-You are asked to review a 75-year-old female on the surgical wards due to
hyperkalaemia. Results are as follows:
he earaer aeraer asdsadas eerw dssdfsselleds
Plasma Urine
K+ (mmol/l) 6.8
C.A Hydronephrosisia
E.A Pyelonephritisia
ATN or prerenal uraemia? In prerenal uraemia think of the kidneys
holding on to sodium to preserve volume
The low urine sodium points towards prerenal uraemia, as does the urine:plasma
osmolality and urea ratio
_____________________________________________________________________5
6-A patient with type 1 diabetes mellitus is reviewed in the nephrology outpatient clinic.
He is known to have stage 3 diabetic nephropathy. Which of the following best
describes his degree of renal involvement?ia
B.A Microalbuminuriaia
E.A Hyperfiltrationia
_____________________________________________________________________5
7-A two-year old boy presents with an abdominal mass. Which of the following is
associated with Wilm's tumour (nephroblastoma)?ia
Wilm's tumour
Wilm's nephroblastoma
• occurs mostly < 3 years (80% < 5 years); 20% of all childhood malignancieshe
• Beckwith-Wiedemann syndromehe
• AGR triad of Aniridia, Genitourinary, Retardationhe
• deletion on short arm of chromosome 11he
_____________________________________________________________________5
8-A 45-year-old presents to A&E with chest pain. An ECG shows anterior ST elevation
and he is thrombolysed with alteplase. His chest pain settles and he is started on aspirin,
atorvastatin, bisoprolol and ramipril. Three days later his blood results are as
follows:eraer asdsadas eerw dssdfsselleds
Urea 16 mmol/l
There is likely underlying renal artery stenosis revealed by the addition of an ACE
inhibitor
Renal vascular disease
sqweqwesf erwrewfsdfs adasd dhe
Renal vascular disease is most commonly due to atherosclerosis (> 95% of patients). It
is associated with risk factors such as smoking and hypertension that cause atheroma
elsewhere in the body. It may present as hypertension, chronic renal failure or 'flash'
pulmonary oedema. In younger patients however fibromuscular dysplasia (FMD) needs
to be considered. FMD is more common in young women and characteristically has a
'string of beads' appearance on angiography. Patients respond well to balloon
angioplasty
he earaer aeraer asdsadas eerw dssdfsselleds
Investigation
_____________________________________________________________________5
9-Which one of the following may be useful in the prevention of oxalate renal stones?ia
C.A Lithiumia
D.A Pyridoxineia
E.A Allopurinolia
Oxalate stones
• cholestyramine reduces urinary oxalate secretionhe
• pyridoxine reduces urinary oxalate secretionhe
• allopurinolhe
• urinary alkalinization e.g. oral bicarbonatehe
_____________________________________________________________________6
0-In Goodpasture's syndrome anti-glomerular basement membrane (anti-GBM)
antibodies are directed against which type of collagen?ia
Goodpasture's syndrome
sqweqwesf erwrewfsdfs adasd dhe
Goodpasture's syndrome is rare condition associated with both pulmonary haemorrhage
and rapidly progressive glomerulonephritis. It is caused by anti-glomerular basement
membrane (anti-GBM) antibodies against type IV collagen. Goodpasture's syndrome is
more common in men (sex ratio 2:1) and has a bimodal age distribution (peaks in 20-30
and 60-70 age bracket). It is associated with HLA DR2
he earaer aeraer asdsadas eerw dssdfsselleds
Features
• pulmonary haemorrhage he
• followed by rapidly progressive glomerulonephritishe
• young maleshe
• smokinghe
• lower respiratory tract infectionhe
• pulmonary oedemahe
• inhalation of hydrocarbonshe
he earaer aeraer asdsadas eerw dssdfsselleds
Investigations
• plasma exchangehe
• steroidshe
• cyclophosphamidehe
_____________________________________________________________________6
1-A 24-year-old man who has a sister with polycystic kidney diseases asks his GP if he
could be screened for the disease. What is the most appropriate screening test?ia
B.A CT abdomenia
ADPKD
sqweqwesf erwrewfsdfs adasd dhe
Autosomal dominant polycystic kidney disease (ADPKD) is the most common inherited
cause of kidney disease, affecting 1 in 1,000 Caucasians. Two disease loci have been
identified, PKD1 and PKD2, which code for polycystin-1 and polycystin-2 respectively
he earaer aeraer asdsadas eerw dssdfsselleds
Chromosome 16 Chromosome 4
Presents with ESRF earlier
62-Which one of the following is the most common type of SLE associated renal
disease?ia
• treat hypertensionhe
• corticosteroids if clinical evidence of diseasehe
• immunosuppressants e.g. azathiopine/cyclophosphamidehe
_____________________________________________________________________6
3-Each one of the following is a recognised side-effect of erythropoietin, except:ia
A.A Hypertensionia
C.A Encephalopathyia
E.A Thrombocytopeniaia
Erythropoietin
sqweqwesf erwrewfsdfs adasd dhe
Erythropoietin is a haematopoietic growth factor that stimulates the production of
erythrocytes. The main uses of erythropoietin are to treat the anaemia associated with
chronic renal failure and that associated with cytotoxic therapy
he earaer aeraer asdsadas eerw dssdfsselleds
Side-effects of erythropoietin
• iron deficiencyhe
• inadequate dosehe
• concurrent infection/inflammationhe
• hyperparathyroid bone diseasehe
• aluminium toxicityhe
_____________________________________________________________________6
4-Each one of the following is a feature of renal cell cancer, except:ia
D.A Haematuriaia
E.A Polycythaemiaia
_____________________________________________________________________6
5-Which one of the following types of glomerulonephritis is most characteristically
associated with Wegener's granulomatosis?ia
Glomerulonephritides
Knowing a few key facts is the best way to approach the difficult subject of
glomerulonephritis:
he earaer aeraer asdsadas eerw dssdfsselleds
Membranous glomerulonephritis
_____________________________________________________________________6
6-Each one of the following is typically seen in patients with rhabdomyolysis, except:ia
B.A Hypercalcaemiaia
E.A Myoglobinuriaia
Rhabdomyolysis
sqweqwesf erwrewfsdfs adasd dhe
Rhabdomyolysis will typically feature in the exam as a patient who has had a fall or
prolonged epileptic seizure and is found to have acute renal failure on admission
he earaer aeraer asdsadas eerw dssdfsselleds
Features
• seizurehe
• collapse/coma (e.g. elderly patients collapses at home, found 8 hours later)he
• ecstasyhe
• crush injuryhe
• McArdle's syndromehe
• drugs: statinshe
This man has nephrotic syndrome which is associated with coeliac disease
IgA nephropathy
sqweqwesf erwrewfsdfs adasd dhe
Basics
Presentations
• alcoholic cirrhosishe
• coeliac disease/dermatitis herpetiformishe
he earaer aeraer asdsadas eerw dssdfsselleds
Management
_____________________________________________________________________6
8-Which one of the following is not a feature of HIV-associated nephropathy?ia
B.A Normotensionia
D.A Proteinuriaia
• massive proteinuriahe
• normal or large kidneyshe
• focal segmental glomerulosclerosis with focal or global capillary collapse on
renal biopsyhe
• elevated urea and creatininehe
• normotensionhe
B.A Penicillamineia
C.A Hepatitis Bia
D.A SLEia
E.A Lymphomaia
Membranous glomerulonephritis
sqweqwesf erwrewfsdfs adasd dhe
Membranous glomerulonephritis is the commonest type of glomerulonephritis in adults
and is the third most common cause of end-stage renal failure (ESRF). It usually
presents as nephrotic syndrome or proteinuria
he earaer aeraer asdsadas eerw dssdfsselleds
Renal biopsy demonstrates:
• idiopathiche
• infections: hepatitis B, malariahe
• malignancy: lung cancer, lymphoma, leukaemiahe
• drugs: gold, penicillamine, NSAIDshe
• systemic lupus erythematous (class V disease)he
70-Which of the following types of renal tubular acidosis is most likely to cause renal
stones?ia
A.A Type 1 renal tubular acidosisia
• causes hyperkalaemiahe
• causes include hypoaldosteronism, diabeteshe
_____________________________________________________________________7
1-Which one of the following is least associated with minimal change
glomerulonephritis?ia
C.A Thymomaia
D.A Non-steroidal anti-inflammatory drugsia
• nephrotic syndromehe
• hypertensionhe
• highly selective proteinuriahe
• renal biopsy: electron microscopy shows fusion of podocytes he
_____________________________________________________________________7
2-Which one of the following types of glomerulonephritis is most characteristically
associated with cryoglobulinaemia?ia
A.A Bendrofluazideia
B.A Aminophyllineia
C.A Acetazolamideia
D.A Frusemideia
E.A Prednisoloneia
Bendrofluazide may help prevent the formation of calcium based renal stones. It may
however theoretically increase the risk of urate based stones
• dehydrationhe
• hypercalciuria, hyperparathyroidism, hypercalcaemiahe
• cystinuriahe
• high dietary oxalatehe
• renal tubular acidosishe
• medullary sponge kidney, polycystic kidney diseasehe
• beryllium or cadmium exposurehe
• gouthe
• ileostomy: loss of bicarbonate and fluid results in acidic urine, causing the
precipitation of uric acidhe
Drug causes
_____________________________________________________________________7
4-The albumin:creatinine excretion ratio (ACR) may be used to quantify the degree of
proteinuria in renal disease. A normal ACR may be defined as:ia
D.A 5 - 50ia
Proteinuria
Microalbuminuria
• post-streptococcal glomerulonephritishe
• subacute bacterial endocarditishe
• systemic lupus erythematoushe
• mesangiocapillary glomerulonephritishe
_____________________________________________________________________
76-Which of the following types of renal stones are radio-lucent?ia
• causes hyperkalaemiahe
• causes include hypoaldosteronism, diabeteshe
E.A Cryoglobulinaemiaia
Therapeutic benefit has not been found by controlled trials in patients with multiple
sclerosis
Plasma exchange
sqweqwesf erwrewfsdfs adasd dhe
Indications for plasma exchange
• Guillain-Barre syndromehe
• myasthenia gravishe
• Goodpasture's syndromehe
• ANCA positive vasculitis e.g. Wegener's, Churg-Strausshe
• TTP/HUShe
• cryoglobulinaemiahe
• hyperviscosity syndrome e.g. secondary to myelomahe
_____________________________________________________________________7
9-Which of the following factors would suggest that a patient has pre-renal uraemia
rather than established acute tubular necrosis?ia
_____________________________________________________________________8
0-Autosomal dominant polycystic kidney disease type 2 is associated with a gene defect
in:ia
ADPKD
sqweqwesf erwrewfsdfs adasd dhe
Autosomal dominant polycystic kidney disease (ADPKD) is the most common inherited
cause of kidney disease, affecting 1 in 1,000 Caucasians. Two disease loci have been
identified, PKD1 and PKD2, which code for polycystin-1 and polycystin-2 respectively
he earaer aeraer asdsadas eerw dssdfsselleds
Chromosome 16 Chromosome 4
_____________________________________________________________________8
1-A patient with type 1 diabetes mellitus is reviewed in the nephrology outpatient clinic.
He is known to have stage 2 diabetic nephropathy. Which of the following best
describes his degree of renal involvement?ia
A.A Microalbuminuriaia
D.A Hyperfiltrationia
_____________________________________________________________________8
2-A 14-year-old boy develops haematuria following an upper respiratory tract infection.
What is the likely diagnosis?ia
Glomerulonephritides
sqweqwesf erwrewfsdfs adasd dhe
Knowing a few key facts is the best way to approach the difficult subject of
glomerulonephritis:
he earaer aeraer asdsadas eerw dssdfsselleds
Membranous glomerulonephritis
_____________________________________________________________________8
3-Which one of the following statements regarding minimal change glomerulonephritis
is incorrect?ia
2. In patients with suspected idiopathic pulmonary fibrosis, the most valuable measure is:
a. Bronchoscopy
b. Sedimentation rate
c. Trial of steroids
d. Open lung biopsy xxx
1) All of the followings are useful for the assessment of the severity of patients with bronchial asthma
except :
a- Spirometry .
b- Methacholine tests
c- ABG (arterial blood gases)
d- Peak expiratory flow rate for variability.
e- Physical examination .
2) Which one of the following pulmonary function values indicates airflow limitation .
a-FEV1 of 60% of predicted .
b-FVC of 60% of predicted .
c-FEV1/FVC of 60% of predicted .
d-DLCO of 60% of predicted .
e- Residual volume of 60% of predicted .
3) All of the followings can be used as a challenge tests for patients with bronchial asthma except ;
a-Methacholine .
b-Histamine .
c-Normal saline .
d-Excersise .
e-Hypertonic saline .
4) Lung’s failure type respiratory failure is characterized by which one o the followings :
a-Normal chest X ray .
b-Hypocapnia or normocapnia .
c-Diffusion is the main mechanism of hypoxia .
d- Easy to correct hypoxia .
e- PEEP is contraindicated .
5) All of the following conditions typically can cause pump failure’s type respiratory failure except :
a-Myasthenia gravis .
b-Multiple rib fractures .
c-Bronchiolitis obliterans .
d-Severe chest pain .
e-Gullien-Barrie syndrome
6- Wide alveolar-arterial Po2 (PA-a O2) gradient can be increased in all of the following conditions
except :
a-Morphine over dose .
b-Severe pneumonia .
c-Acute Bronchial Asthma .
d-Acute Pulmonary edema .
e-ARDS (acute respiratory distress syndrome) .
8) – All of the followings are true regarding home monitoring with the PEFR except
a-Usefull in diagnosing asthma
b- Usefull in Identifying environmental triggers of asthma
c- Can detect early signs of deterioration before symptoms change.
d-Long term monitoring is useful for severe brittle asthma .
e- It is less effort dependent than spirometry .
9) The treatment of Bronchial asthma by Anti-inflammatory agents may cause all of the following except:
a-It reduces symptoms .
b-It improves lung function .
C-It decreases BHR (bronchial hyper reactivity).
d-It improves quality of life
e-It may cure the patient from the disease .
f. Hypercapnia.
g. alkalosis.
h. Increased work of breathing.
i. Increased deoxygenated Hb .
j. Hypoxia .
12) Noninvasive intermittent positive pressure ventilation (NIIPPV) in acute exacerbations of COPD
patients improves all of the followings except :
13) One of the following treatment for patients with advanced COPD may improve their survival
a. Exercise and rehabilitation.
b. Nocturnal O2 therapy .
c. Long term O2 therapy (more than 15Hrs) .
d. Prophylactic nebulized antibiotics .
e. Nebulized steroids (budesonide).
14) All of the followings are risk factors for pulmonary tuberculosis except :
a-Close contacts to TB patient
b-Immigration from an endemic area
c-Exposure to under treated cases
-Young adults
-Residence of high incidence location .
15) Regarding pleural effusion caused by TB
a- Fluid analysis Predominated by lymphocytes
b-Fluid positive for AFB stain in less than one third of patients .
c-Negative culture for AFB can’t exclude the disease.
d-Pleural biopsy increases the yield for AFB culture.
e-Usually it is difficult to treat ,and need treatment for 9-12 months .
16)All of the followings are associated with Worse prognosis in sarcoidosis except :
a- Incidious onset
b- Multiple extrathoracic lesion.
c- Blacks..
d- Erythema nodosum.
e- Lupus pernio.
17)All of the followings are true combination between a risk factors and pathgens causing pneumonia
except :
a-Alcoholism and klebsella pneumonia
a- old age and mycoplasma pneumonia
b Cigarette smokers and H .infleunza
c-Mechanical ventilation and pseudomonal pneumonia.
d-Abnormal level of consciousness and anaerobic bacteria
18) This is atypical part of 8 hours polysomnography for a 45 year old male patient .
19) ) Next is a typical part of 8 hours polysomnography for a 45 year old male patient .
What is the diagnosis of this patient ?
a-Narcolepsy.
b-Central apnea
c-Obstructive sleep apnea.
d-Mixed apnea .
e-Hypopnea .
20) 19year old man previously healthy presented to the emergency room coughing frish red blood , AFB
stain is negative and the AFB culture is pending , he was given three units of blood after which he remained
thermodynamically stable . a spiral CT scan of the chest using PE protocol showed airspace changes in the
Rt lower lobe ,a bronchoscopy showed large clot in the Rt lower lobe. with no other abnormalities seen
.Which one of the following is the most appropriate next step in the management of this patient ?
a - Start the patient on anticoagulation as therapeutic dose for PE .
b - Start the patent on anti TB treatment.
c - Ask for conventional CT scan.
d - Ask for conventional pulmonary and bronchial angiogram.
e - Sent the patient for Rt lower lobectomy.
21) This is a typical part of 8 hours polysomnography for a 45 year old male patient .
23) Which one of the following conditions is an absolute indication for thrombolytic therapy in
patients with PE ?
a- Contraindication for anticoagulation .
b- Large filling defect(s) on spiral CT scan .
c- Hypotention caused by PE .
d- Bilateral PE .
e- presence of ECG changes suggestive of PE .
24) All of the followings are known complications of non invasive ventilation NIV except.
a- Gastric insufflations .
b- Conjunctival Irritation ..
c- Hypotension.
d- Pulmonary edema .
e- Pneumothorax.
25) A 60 year old man , long standing smoker , has progressive dyspnea and prductive cough for the last 2
years .1 week ago had flue like symptoms followed by increase in his symptoms .His best Spirometry taken
2 months ago showed FEV1 1.1 Liter ,FVC 2.2 without changed after inhaled B2 agonist . The ABG taken
10 minuets ago while breathing room air was PH 7.29 , Pa Co2 56 HCO3 32 Po2 35 .
One of the followings is true regarding this patient except :
26) A24 year old female patient has 4 days history of fever .chills and left sided chest pain which increased by
inspiration .Chest x ray showed consolidation in the left lower zone with signs of pleural effusion on the same
side .All of the followings are indications for insertion of chest tube and intrapleural thrombolytic therapy
except ;
28) In patients with idiopathic pulmonary fibrosis (usual interstitial pneumonia) all of the followings are
expected physiological changes except :
f. Low DLCO .
g. Decreased FEV1/FVC .
h. Severe O2 desaturation on exercise.
i. Reduced vital capacity and total lung capacity .
j. Increased pulmonary artery pressure .
30) All of the followings may improve obstructive sleep apnea except :
a- Dental extraction .
b- Weight Reduction .
c- Decrease alcohol consumption .
d- Nasal CPAP
e- Tracheostomy .
31) Nasl CPAP/BiPAP can be used to treate all of the following conditions except :
a- Myasthnia gravis
b- Acute pulmonary edema .
c- Obstructive sleep apnea .
d- Respiratory failure due to severe kyphoscoliosis .
e- Narcolepsy .
32) All of the following statement regarding lung cancer are true except :
f. Smoking is a known risk factor for all types of bronchogenic carcinoma .
g. Adenocarcinoma usually is a peripheral lung tumor .
h. Adenocarcinoma in some cases is difficult to be differentiated from mesothelioma .
i. Thromboembolic disease can be the first manifestation of the disease.
j. Surgery can be curative for early diagnosed cases .
1. Which one of the following is LEAST useful in assessing patient with a poor prognosis in community-
acquired pneumonia?
a- mental confusion
b- urea of 11.4 mmol/l
c- positive C-reactive protein
d- respiratory rate of 35/ min.
e- age 75 years old.
f- 8.Bilateral hilar lymph nodes enlargement occurs commonly in all the following Except.
g- a- pulmonary Tuberculosis
h- b- chronic myeloid leukemia
i- c- non-Hodgkins lymphoma
j- d- Hodgkin lymphoma
k- e- sarcoidosis !! مش أكيد
14. ONE of the following drugs is LEAST used in treatment of acute sever asthma.
a- nebulized B2 agonist
b- i.v hydrocortisone
c- epinephrine (adrenaline)
d- oxygen
e- i.v . aminophylline
15. Hypoxia (decreased PaO2) and decreased Pa CO2 is found in all the following Except.
a- left ventricular failure
b- massive pulmonary embolism
c- acute sever asthma
d- acute exacerbation of COPD
e- pneumonia
ONE of the following drugs is most appropriate in treatment of pneumocystis carinii pneumonia.
a- clarithromycin
b- ethambutol
c- azithromycin
d- Trimethoprim-Sulphamethoxazole
e- INH and rifampicine
51. 20-year old woman presents with a week history of fever, rigor and productive rusty cough. The chest X-ray
shows left lower lobe consolidation.
Which ONE of the following is most appropriate treatment?
a- clarithramycin
b- gentamycin
c- Cotrimoxazole
d- Benzypenicillin
e- Flucloxacillin
69. A pleural effusion analysis results: ratio of concentration of total protein in pleural fluid to serum of 0. 38 ,
latate dehydrogenase LDH level of 125 IU, and ratio of LDH in pleural fluid to serum of 0. 45.
Which of the following ONE disease is the most likely the cause for this pleural effusion.
a- uremia
b- pulmonary embolism
c- sarcoidosis
d- SLE
e- Congestive heart failure
82. Which ONE of the following Arterial Blood Gases is most likely to be found in a 60-year-old heavy smoker
man, He has chronic bronchitis, peripheral odema and cyanosis?
a- PH 7.50, PO2 75, PCO2 28
b- PH 7.15, PO2 78, PCO2 92
c- PH 7.06, PO2 36, PCO2 95
d- PH 7.06, PO2 108, PCO2 13
f- PH 7.39, PO2 48, PCO2 54
87. All of the following associations between conditions and mechanisms of hypoxia are true, except:
a. COPD and V/Q mismatch (The principal contributor to hypoxemia in COPD patients is
ventilation/perfusion (V/Q) mismatch resulting from progressive airflow limitation)
b. ARDS and pulmonary shunt (edema in patients with ALI/ARDS is impaired gas exchange with
intrapulmonary shunt,)
c. Multiple rib fractures and hypoventilation
d. Hepatopulmonary syndrome and V/Q mismatch (The hepatopulmonary syndrome is characterized by a
defect in arterial oxygenation induced by pulmonary vascular dilatation in the setting of liver disease1)
(Dyspnea and hypoxemia are worse in the upright position (which is called platypnea and orthodeoxia,
respectively)
e. Motor neuron disease and hypoventilation
Answer: C
88. IN patients with sarcoidosis, all of the following are associated with good prognosis, except:
a. Fever
b. Erythema nodosum
c. Age less than 40 years
d. Black race
e. Presence of polyarthritis
Answer: D.
89. Regarding the pathogenesis of bronchial asthma, one of the following is specific for the disease:
a. Air flow limitation
b. Airway hyper-responsiveness
c. Inflammation of the mucosa
d. Peak flow variability
e. Brochioalevolar eosinophils
Answer: B.
90. The most common organism responsible for severe community pneumonia needing ICU care is:
a. Strep. pnuemonia
b. Legionella
c. H. influenza
d. Gram negative bacilli
e. Mycoplasama pneumonia
2. Which one of the following arterial blood gas sets on room air is compatable with
completely compensated metabolic acidosis?
A B C D E
PH 7.44 7.38 7.60 7.36 7.56
PaC02 mmHg 26 25 25 95 40
Bicarb. mEq 18 15 24 49 34
B. Excess -4.0 -10 +4 +15 +11
The pH must be normal. Therefore, exclude “E” and “C”. The correction will be respiratory in the form of
“washed-out” CO2 need to be low. Therefore, exclude D. Bicarbonate will be low. The remaining options are A
& B.
3. Lung’s failure type respiratory failure is characterized by which one o the followings :
f. Normal chest X ray .
g. Hypocapnia or normocapnia .
h. Diffusion is the main mechanism of hypoxia .
i. Easy to correct hypoxia .
j. PEEP is contraindicated .
Respiratory failure is a syndrome in which the respiratory system fails in one or both of its gas exchange
functions: oxygenation and carbon dioxide elimination. In practice, it may be classified as either hypoxemic or
hypercapnic.
Hypoxemic respiratory failure (type I) is characterized by an arterial oxygen tension (Pa O2) lower than 60 mm
Hg with a normal or low arterial carbon dioxide tension (Pa CO2). This is the most common form of respiratory
failure, and it can be associated with virtually all acute diseases of the lung, which generally involve fluid filling
or collapse of alveolar units. Some examples of type I respiratory failure are cardiogenic or noncardiogenic
pulmonary edema, pneumonia, and pulmonary hemorrhage.
Hypercapnic respiratory failure (type II) is characterized by a PaCO2 higher than 50 mm Hg. Hypoxemia is
common in patients with hypercapnic respiratory failure who are breathing room air. The pH depends on the
level of bicarbonate, which, in turn, is dependent on the duration of hypercapnia. Common etiologies include
drug overdose, neuromuscular disease, chest wall abnormalities, and severe airway disorders (eg, asthma
and chronic obstructive pulmonary disease [COPD]).
Answer: A.
4. In patients with idiopathic pulmonary fibrosis (usual interstitial pneumonia) all of the
followings are expected patho physiological changes EXCEPT :
k. Low DLCO .
l. Decreased FEV1/FVC .
m. Severe O2 desaturation on exercise.
n. Reduced vital capacity and total lung capacity .
o. Increased pulmonary artery pressure
Answer: B (increased FEV1/FVC ratio).
5. All of the followings may improve obstructive sleep apnea EXCEPT:
a. Dental extraction .
b. Weight Reduction .
c. Decrease alcohol consumption .
d. Nasal CPAP
e. Tracheostomy
Answer: A
9. All of the following statement regarding lung cancer are true EXCEPT :
k. Small cell lung carcinoma metastasis late in the course of the disease
l. Adenocarcinoma usually is a peripheral lung tumor .
m. Adenocarcinoma in some cases is difficult to be differentiated from mesothelioma .
n. Thromboembolic disease can be the first manifestation of the disease.
o. Surgery can be curative for early diagnosed cases .
Answer: “A. Compared to non-small cell lung cancer, small cell lung cancer is just bad disease. The tumor grows
fast and metastasizes early. Small cell is more often associated with paraneoplastic syndromes (e.g., Eaton-
Lambert) and ectopic hormonal syndromes (e.g., SIADH).
10. All of the followings are useful for the assessment of the severity of an attack of
bronchial asthma, EXCEPT :
a. Spirometry .
b. Methacholine test
c. ABG (arterial blood gases)
d. Peak expiratory flow rate
e. Physical examination.
Answer: B. Methacholine tes: methacholine challenge test: a test that involves the inhalation of increasing
concentrations of methacholine, a potent bronchoconstrictor, in patients with possible bronchial
hyperreactivity; usually performed when a diagnosis of asthma or bronchospastic lung disease is not clinically
obvious. Source: Stedman’s.
11. Which one of the following pulmonary function values indicates airflow limitation
a. FEV1 of 60% of predicted .
b. FVC of 60% of predicted .
c. FEV1/FVC of 60% of predicted .
d. DLCO of 60% of predicted.
e. Residual volume of 60% of predicted.
Answer: C. FEV1/FVC of 60%. Total lung capacity (TLC) is used to assess interstitial lung disease. Expiratory flow
rate (FEV1/FVC is used to assess obstructinve lung disease. Airway obstruction is diagnosed when the FEV1/FVC
is <0.7 (70%0). (Source: MedStudy Pulmonology 2013, p. 6)
9. Wide alveolar-arterial Po2 (PA-a O2) gradient can be increased in all of the following
conditions EXCEPT:
a. Morphine overdose .
b. Severe pneumonia .
c. Acute Bronchial Asthma .
d. Acute Pulmonary edema .
e. ARDS (acute respiratory distress syndrome)
Answer: In morphine overdose ➔ Hypoventilation ➔ No washout of alveolar CO2 and replacement with new
O2 ➔ Both arterial and alveolar O2 are decreased. Therefore, the PAa O2 gradient is decreased.
12. All of the followings are true combination between a risk factor and pathogens causing
pneumonia EXCEPT :
a. Alcoholism and klebsella pneumonia
b. Old age and mycoplasma pneumonia
c. Cigarette smoking and H .infleunza
d. Mechanical ventilation and pseudomonal pneumonia.
e. Abnormal level of consciousness and anaerobic bacteria
13. This is atypical part of 8 hours polysomnography for a 45 year old male patient